EMT Combine 1 to 30

Pataasin ang iyong marka sa homework at exams ngayon gamit ang Quizwiz!

The EMT correctly understands a​ "Physician Orders for​ Life-Sustaining Treatment" document when he​ states: A. ​"A POLST is designed to allow a patient to choose the desired type of care prior to the need for​ resuscitation." B. ​"A POLST legally empowers another person to make health care decisions for the patient in the event he or she cannot do​ so." C. ​"A POLST must be signed by the court system and the patient and is valid for a period of no more than 5​ years." D. ​"A POLST is the same as a​ 'Do Not​ Resuscitate' order, with the difference being that the physician does not have to sign​ it."

"A POLST is designed to allow a patient to choose the desired type of care prior to the need for​ resuscitation."

A woman calls the station and asks if you transported a patient named Patricia Gardner earlier in the day. She says that she is her friend and wants to know if she is okay. An appropriate response would​ be: A. ​"She was treated for chest​ pain, but I cannot release her diagnosis for​ admission." B. ​"I can tell you that she was transported to the​ hospital, but I cannot tell you​ why." C. ​"Although I do not doubt that you are her​ friend, I cannot release that​ information." D. ​"I cannot tell you​ that, but her family is at home and they can give you more​ information."

"Although I do not doubt that you are her​ friend, I cannot release that​ information."

An adult patient with the capacity to make his own decisions has​ cardiac-type chest pain and is refusing all treatment. What would be the​ EMT's most appropriate​ response? A. ​"Before you​ refuse, I need to tell you that this could be a heart attack and you could​ die." B. ​"Since you are oriented and​ rational, it is your right to refuse. Please sign this refusal of care​ form." C. ​"I am​ sorry, but since 911 was​ called, we must transport you to the hospital. You can refuse treatment​ there." D. ​"I will need to have you sign this refusal of care form and a family member sign as a​ witness."

"Before you​ refuse, I need to tell you that this could be a heart attack and you could​ die."

You have been dispatched to a residence where a woman lacerated her arm after falling while holding a drinking glass. She informs you that when she was first​ cut, blood spurted from the​ wound, and then within a second or​ two, it subsided to a mere trickle. She asks why this would happen. Which one of the following is the correct​ response?

"Blood vessels tend to constrict quickly once they are​ injured."

Which one of the following is an open-ended question?

"Can you describe the pain to me?"

You are returning from a call when you come across an apartment building with flames coming from several windows on the top story. You see several occupants trapped and hanging out of the windows screaming for help. Which one of the following radio transmissions is most appropriate to advise dispatch of the situation?

"Central communications from Unit 118, how do you copy?"

You have arrived by the side of an 82-year-old male complaining of generalized weakness. After introducing yourself, he states, "My name is Edward Burns, but everyone calls me Eddy." Which one of the following is your most appropriate response?

"Do you want us to call you Eddy as well?"

Which instruction is appropriate when helping a new EMT care for a male patient with a laceration to his left​ arm?

"Don't worry about removing embedded dirt from the​ wound, just the dirt around the​ laceration."

Which one of the following statements is most appropriate when communicating with the medical facility that will receive the patient?

"En route to your facility with a 73-year-old male complaining of dizziness."

Which of the following statements is NOT appropriate to document in the narrative section of a PCR? A. "The patient admits to smoking marijuana earlier in the day." B. "General impression revealed that the patient was intoxicated." C. "After oxygen was administered, the patient's breathing improved." D. "Significant damage was noted to the front end of the vehicle."

"General impression revealed that the patient was intoxicated."

Which of the following questions would you ask a patient to ascertain the "M" in the SAMPLE history?

"How much Tylenol do you take each day?

You have been called to a home for an alert and oriented female with terminal lung cancer. At the​ home, the​ daughter, who has her​ mother's durable power of​ attorney, states that she wants her mother transported to the hospital. The​ patient, who is alert and​ oriented, does not want to go. Which statement indicates that the EMT is acting both appropriately and legally in this​ conflict? A. ​"Since your daughter has the durable power of​ attorney, she has the legal right to make all​ decisions; therefore, we have to take you to the​ hospital." B. ​"Since it appears there is a conflict of opinion​ here, I will need to call the police so that they can determine what to​ do." C. ​"I know that you have the durable power of​ attorney, but your mother is alert and oriented so we cannot legally take her against her​ will." D. ​"As the person with your durable power of​ attorney, your daughter has the legally vested right to make your​ decisions; however, let us ask your husband what he​ thinks."

"I know that you have the durable power of​ attorney, but your mother is alert and oriented so we cannot legally take her against her​ will."

You are having a difficult time controlling bleeding from a small skin avulsion on a​ patient's ankle. Which one of the following statements made by the patient best explains why control has been difficult to​ achieve?

"I take Coumadin for my irregular​ heartbeat."

Which statement made by the EMT shows an understanding of bandaging an open​ wound?

"I use bandaging material to secure a dressing in​ place."

Which one of the following statements shows that the EMT understands the palm method of estimating the BSA burned?

"I use the patient's palm to estimate the size of the burn area."

You are by the side of a female patient who has overdosed on a drug. She is conscious but very combative and noncompliant with your requests. Which statement made by the EMT best suggests the EMT is committing​ assault? A. ​"You win. I cannot get your blood pressure and the doctor is going to yell at me. I am going to blame​ you." B. ​"If you throw up in the back of my​ ambulance, I will use your shirt to clean it up. Just think about that for a​ minute." C. ​"If that is the way you are going to​ behave, then I am no longer going to try to get your blood​ pressure." D. ​"If you do not let me get your blood​ pressure, I will take off the oxygen and you will​ suffocate."

"If you do not let me get your blood​ pressure, I will take off the oxygen and you will​ suffocate."

Which one of the following statements is appropriate when transferring a patient to the emergency department and giving an oral report?

"Mr. Benton called us today because he was having chest pain."

You are assessing a young male patient who was assaulted with a baseball bat and struck once on the left upper quadrant of the abdomen. You would recognize the patient as having​ Kehr's sign when he makes what​ statement?

"My left shoulder is killing​ me!"

How should the EMT respond when first making contact with an alert and oriented patient suffering from a medical emergency?

"My name is Charles Smith, and I am an EMT. Can I help you?"

When interviewing a patient with a medical emergency, the EMT is using the technique of summary when she says:

"So the nausea and vomiting started two days ago."

Which one of the following is the most appropriate way to contact a receiving hospital to alert them that you are en route with a patient?

"St. Margaret's Hospital, this is BLS ambulance 17-64. How do you copy?"

The EMT shows understanding of the standard of care when he​ states: A. ​"The standard of care is a measure of whether the right assessment and care have been provided to the​ patient." B. ​"The standard of care is a state law that describes which actions the EMT can and cannot legally​ perform." C. ​"The standard of care is a document that outlines the curriculum to which all EMT programs in the United States must​ subscribe." D. ​"The standard of care is state law that requires the EMS to respond and render care in emergency and nonemergency​ situations."

"The standard of care is a measure of whether the right assessment and care have been provided to the​ patient."

There has been a very serious motor vehicle collision involving a​ high-profile judge seeking reelection. The​ judge's car was traveling at a high rate of speed and crossed the center​ line, striking and killing a motorcyclist. Newspaper reporters are on​ location, asking you to describe what happened. Which statement is most​ appropriate? A. ​"The accident was not​ bad, so please leave the​ scene." B. ​"There has been a serious accident in which a person was​ killed." C. ​"The judge was​ involved, but I cannot tell you anything that​ happened." D. ​"The judge was in the car that hit and killed the​ motorcyclist."

"There has been a serious accident in which a person was​ killed."

A​ 20-year-old male has been stabbed in the abdomen. Assessment reveals him to be unresponsive with shallow breathing and a low blood pressure. Which instructions will you provide to your new EMT partner given that the patient has been involved in a​ crime? A. ​"We are not to move the patient to the stretcher until the police are done photographing the​ scene." B. ​"I am not worried about potential evidence. I am just worried about the​ patient." C. ​"When cutting his shirt​ off, make sure not to cut through the area where the knife​ penetrated." D. ​"Do not worry about documenting the crime scene in your report. The police report will reflect that​ information."

"When cutting his shirt​ off, make sure not to cut through the area where the knife​ penetrated."

You are assessing a patient complaining of shortness of breath. Which one of the following questions would best assist you in getting additional information about the patient's chief complaint?

"You appear to be short of breath. Are you also having chest pain?"

A 31-year-old female with a history of depression took some pills in a suicide attempt. Her husband is visibly upset and her three young children are crying as you place her into the ambulance. She is conscious and stable as you transport her to the hospital. During the transport, she tearfully states she did not mean to take the pills and asks if she is going to die. In regards to therapeutic communication, which one of the following is your best response

"You are stable right now and we are transporting you for definitive care."

A construction foreman was stabbed with a screwdriver in the right anterior chest by an angry employee. Prior to your​ arrival, he removed the screwdriver. He is alert and oriented and complaining of pain to the injury site. Your primary assessment reveals an open​ airway, adequate​ breathing, and strong radial pulse. Based on these​ findings, which of the following would be done​ first?

. Evaluate the patient SpO2 level.

From a mental health standpoint, an abnormal or disturbing pattern of behavior is a matter of concern if it lasts for at least:

1 month

The mastoid process is located approximately

1" posterior to the external opening of the ear.

Optimally, on-scene time for critically injured patients should be less than ____ minutes.

10

When administering epinephrine via auto-injector, you should hold the injector in place for:

10 seconds

At what age does separation anxiety typically peak in infants and small children?

10 to 18 months

10. You have been told that you must get your annual purified protein derivative (PPD) test for tuberculosis (TB). As acknowledgeable EMT, you recognize the the PPD will:

10. Determine if you have been exposed to TB and need to be repeated annually (new answer)

What is the minimum number of chest compressions that should be delivered per minute to a 4-month- old infant? a. 100b. 120c. 90d. 110

100

An infant with a total blood volume of 800 mL would start showing signs of shock when as little as ______ of blood is lost. Select one: A. 50 mL B. 100 mL C. 150 mL D. 200 mL

100 mL

The partial pressure of oxygen in the alveoli is _______ mm Hg, while the partial pressure of carbon dioxide in the alveoli is _______ mm Hg.

104, 40

The onset of menstruation usually occurs between the ages of:

11 and 16 years

11. Which one the following statements made by an EMT shows that he has an accurate understanding of hepatitis C?

11. " Wearing gloves and washing hands are the best means by which to prevent the transmission of hepatitis C to health care workers.

An adult at rest should have a respiratory rate that ranges between:

12 and 20 breaths/min.

The normal respiratory rate for an adult should range from:

12 to 20 breaths per minute.

12. While assessing a patient with abdominal pain, the patient informs you that he is a chronic carrier of hepatitis B. His skin is not jaundiced nor does he have a fever. As an EMT, you realize:

12. The patient's blood and body fluids are infectious

A 29-year-old male with a head injury opens his eyes when you speak to him, is confused as to the time and date, and is able to move all of his extremities on command. His Glasgow Coma Scale (GCS) score is:

13

13. You are treating a patient with active tuberculosis (TB). Which one of the following will afford you the best protection from the disease?

13. High-efficiency particulate aspirator mask

14. A patient has informed you that he was recently diagnosed with HIV but has yet to develop AIDS. Which one of the following statements is true?

14. Gloves will be enough protection unless body fluids are present

15. You are the first to arrive on the scene of a motor vehicle collision. The scene size-up reveals a tanker truck on its side with an unresponsive driver in the cab. Multiple bystanders are around the cab waving for you to come over and help the driver. Your immediate action would be to:

15. Order the bystanders away from the truck

17. As EMTs leave a house where the patient refused care, a panicked boy approaches them and states that some men just shot his friend in a house two doors up the street. Their immediate action would be:

17. Contact the police and move the ambulance from the immediate area

18. You are transporting a 58-year-old female suffering from terminal breast cancer. While in the ambulance, she becomes tearful and repeatedly asks, "Why me?" As a knowledgeable EMT, you recognize that she is in what stage of grieving?

18. Anger

The human body should be functioning at its optimal level between the ages of:

19 and 25 years.

19. A reaction to death or dying where the patient asserts "Not me!" is recognized as what stage of grieving?

19. Denial

When you assess capillary refill time (CRT) in an infant, normal color to the tested area should return within:

2 seconds

The pressure of gas in a full cylinder of oxygen is approximately _______ pounds per square inch (psi).

2,000

2. You are doing CPR on a male patient is is unresponsive, not breathing, and pulseless. Emergency Medical Responders (EMRs) were on scene first and applied an AED, which did not deliver a shock to the patient. The patient's wife is crying and asks how her husband is doing. Your response would be:

2. "Right now he is not breathing and his heart is not beating, but we are doing everything we possibly can for him."

Significant vital sign changes will occur if the typical adult acutely loses more than ______ of his or her total blood volume. Select one: A. 10% B. 20% C. 15% D. 5%

20%

20. Which one of the following statements should you expect from a patient who has accepted the fact that he is terminally ill?

20. "It's ok. I am not afraid"

Which patient has an isolated head injury that should be considered the MOST​ serious?

21-year-old male wearing a deformed motorcycle helmet who does not respond to verbal or painful stimuli

21. Organisms that cause diseases are correctly called:

21. Pathogens

22. A reference tool that should be placed in every emergency vehicle for use when responding to a possible "hazmat" incident is:

22. The Emergency Response Guidebook

23. For your safety and that of your crew, you should:

23. Evaluate every scene for dangers before entering

24. You have worked with the same partner for several years and know him well. Over the past three months the two of you have had horrific pediatric calls with death as the outcome of each. Which observation would make you the most suspicious that you partner is experiencing a possible stress reaction?

24. Excessive an uncharacteristic irritability

25. Changes in your lifestyle may help you manage stress include:

25. Practicing relaxation techniques such as meditation

26. You have just been instructed to stop emergency care on an unresponsive and pulseless 61-year-old man found by his wife in the bathroom. After telling his wife there is nothing more that can be done, she erupts in an episode of rage and starts screaming that there must be something else that can be done and you are not doing your job well enough. In this situation, which action is best?

26. Be tolerant of the angry reaction

A burn patient has full thickness burns to the fronts of both legs and the entire left arm. The front of the patient's right arm has superficial thickness burns. Using the rule of nines, the EMT should estimate what percentage of the patient's body has been burned

27%

27. After a particularly disturbing call the previous day, your partner is unusually angry and having a difficult time concentrating on his work. What type of stress reaction would you be suspicious of?

27. Acute stress reaction

28. A coworker is suffering from post-traumatic street. Which one of the following statements is most appropriate and beneficial to the coworker?

28. "I think that it would be best if you saw a mental health professional."

29. A prospective student is interested in becoming an EMT, but is unsure how it will affect his family. He sats that his wife is very worried about the danger associated with EMS and has never had to deal with him being gone for 24 hours at a time. Which one of the following is most appropriate response?

29. "It may help to bring your wife here so she can see what we do and the safety precautions we take."

3. The practice of protecting yourself from disease transmission through exposure to blood and other body fluids is referred to as:

3. Standard precautions

Most patients who die of anaphylaxis do so within the first __________ following exposure.

30 minutes

30. You and your partner were among several rescuers on a call involving a house fire in which a 4-year-old girls died. That night, your partner states that he did not sleep well and is extremely bothered by the little girl's death. The next day he calls you and states that he is so upset that he is now having chest pain. Your response would be:

30. " you really need yo get yourself to the emergency department to get checked out."

When testing a mechanical suctioning unit, you should turn on the device, clamp the tubing, and ensure that it generates a vacuum pressure of more than:

300 mm Hg

31. You would be concerned about cumulative stress when an EMS provider informs you that he:

31. Drinks a small amount of alcohol every night to help him sleep

32. A shooting at a public mall leaves three people dead and nine injured, several critically. As operations wind down, many rescuers are physically and emotionally upset. Which of the following is most appropriate at this time?

32. A gathering that allows the rescuers to vent their emotions

33. A tornado ripped through a small town, killing five and seriously injuring 21, including several young children. At a defusing session, which one of the following is most appropriate?

33. Allow the rescuers to vent their emotions

34. If you have questions regarding the steps to take when exposed to an infectious disease, your best resource would be:

34. Your service's written exposure control plan

35. You have just transferred care to an emergency department nurse. There is no blood, fluid, or dirt on your gloves. After taking them off, your priority action would be to:

35. Wash your hands with soap and water

36. A 38-year-old male is complaining of generalized weakness and fatigue. Assessment findings include abdominal pain, fever, and yellowing of the yes. Given these signs and symptoms, you would be suspicious of:

36. Hepatits

37. You are accidentally stuck with a needle used to administer a medication to a patient with a known history of hepatitis B. You have already completed your Hepatits B vaccine series. In this case, which of the following statements is true?

37. "You need to report the needle stick to your supervisor as soon as possible."

38. A patient with a history of Hepatitis C has called 911 because he has a headache. Given the complaint and medical history, which personal protective equipment is most appropriate?

38. Gloves

39. The major primary threat to a person with AIDS is:

39. Infection

4. An EMT is correct when he describes a communicable disease as:

4. A disease that can be spread from one person to another

40. In the field setting, the best way to avoid becoming infected with HIV is:

40. Use the appropriate protective personal equipment

41. You are by the side of a patient you suspect to be suffering from severe acute respiratory syndrome (SARS). Which one of the following is the most effective way to prevent transmission of the disease from the patient to you?

41. Place a surgical mask on the patient

42. You have been called to transport a patient from one medical facility to another. The patient has vancomycin-resistant enterococci (VRE), but does not have any open wounds and is medically stable. Which one of the following is most appropriate?

42. Use the appropriate PPE

43. Which one of the following organisms is classified as multi-drug resistant?

43. Methicillin/oxacillin-resistant Staphylococcus aureus

At a flow rate of 6 L/min, a nasal cannula can deliver an approximate oxygen concentration of up to

44%

With the flowmeter set at 6L/min, the nasal cannula will deliver up to________oxygen

44%

At a flow rate of 6 L/min, a nasal cannula can deliver an approximate oxygen concentration of up to:

44%.

45. Which one of the following actions taken by the EMT illustrates the best effort toward well-being in the field of EMS?

45. Creating a healthy balance between work and family life.

46. After 20 minutes of performing CPR on a 65-year-old female with end stage of multiple sclerosis, your medical director orders you to stop resuscitation and inform the family that the patient has died. The family is upset and crying. You notice the patient's daughter hollering her hand. Your immediate response would be:

46. "Its okay to hold her hand; you can also touch her face and her hair as well."

47. A young girl with end-stage cancer is unresponsive, both breathing, and pulseless. Her mother and father are at her side, both crying as you start emergency care, Which statement indicates the most appropriate action?

47. "If you feel comfortable, it's okay to stay and watch how we care for your daughter."

48. A patient with terminal lung cancer has called EMS for shortness of breath. AT request of your paramedic partner, you are getting the patient's blood pressure. While inflating and then deflating the blood pressure cuff, the patient angrily shouts, "That this is too tight... what are you trying to do, send me to the funeral home a couple of weeks early!" Your best reply would be:

48. "I Know that his is tight. I am just about done now."

For which patient would the palm method be best to estimate the body surface area affected by a burn?

49-year-old with a partial thickness burn to the inner thigh

49. As an EMT, you would recognize which one of the following as holding the greatest potential for causing emotional stress?

49. Caring for a child whose parents have appeared to have neglected him.

An unstable patient should be reassessed at least every:

5 minutes.

What is the minute volume of a patient with a tidal volume of 500 mL, a dead space volume of 150 mL, and a respiratory rate of 16 breaths/min?

5,600 mL

50. regarding "burnout' and the Emergency Medical Services, which statement is most accurate?

50. Burnout can diminish an EMT's ability to provide competent care

An oxygen cylinder should be taken out of service and refilled when the pressure inside it is less than:

500 psi.

51. Your supervisor has assigned you the task of designing a program that provides for stress management within the department. Which program would accomplish this task the best?

51. Complementary passes to a local fitness center

52. An EMT has an understanding of the different types of pathogens when she states:

52. "Viruses are resistant to antibiotics because they live within the body's cells."

53. You are bedside at a long-term care facility for an 89-year-old female with an infected area on her back. Staff states that they are fearful that it is a methicillin/oxacillin-resistant staphylococcus aureus (MRSA) infection so they have requested transport to the emergency department for further evaluation. To decrease your chance of acquiring MRSA, which of the following is critical?

53. Washing your hands following patient contact

54. A nurse has informed you that two days ago you transported a diabetic patient whose ulceration on his foot tested positive for MRSA. Given this information you would:

54. No further action is needed at this time

You are performing mouth-to-mask ventilations with oxygen connected and set at a flow rate of 15 L/min. What percentage of oxygen is your patient receiving?

55%

55. You are attempting to increase your cardiovascular endurance in an effort to better perform your job as an EMT. Which of the following would be most appropriate in working toward this goal?

55. Walking on a treadmill 3 to 4 times a week

56. A coworker is constantly tired and asks you for advise on how to better sleep after a busy night shift. Which of the following responses would best address this situation?

56. "Try to make your bedroom dark and keep the temperature cool."

6. The EMT is correctly performing routine hand washing when she:

6. Lathers and scrubs with plain soap for 10 to 15 seconds

For an adult, the normal resting pulse should be between:

60 and 100 beats/min.

Which of the following scenarios does NOT involve the presence of any symptoms?

61-year-old female who is unconscious with facial cyanosis

7. A patient with human immunodeficiency virus (HIV) is complaining of chest pain. In this situation, what personal protective equipment is most appropriate?

7. Gloves and eyewear

The average pulse rate of individuals between 19 and 60 years of age is typically:

70 beats/min.

The pulse rate of a child from ages 6 to 12 years is approximately:

70 to 120 beats/min.

At present, the average life expectancy is ________ years, while the maximum life expectancy is estimated at ________ years.

78, 120

8. The hospital staff has just returned the long spine board you used to immobilize a patient from a minor vehicle collision. There is no blood on the board, but your service policy states that all equipment must be disinfected any time that it is used. To properly disinfect the long spine board, what should you use?

8. Germicidal solution

The anterior fontanelle fuses together between the ages of:

9 and 18 months.

9. Which one of the immunizations should the EMT receive annually?

9. Influenza vaccine

With a good mask-to-face seal and an oxygen flow rate of 15 L/min, the nonrebreathing mask is capable of delivering up to ______% inspired oxygen.

90

The low normal systolic blood pressure for a 30-year-old is:

90 mm Hg.

reassure the mother and transport the patient.

911 was dialed by a frantic mother for her​ 3-year-old daughter who stuck a​ small, dried bean into her ear. Assessment shows the bean to be lodged firmly in the ear canal. When treating this​ patient, the EMT​ would:

Cold packs to the face and eyelid

A baseball player was struck in the side of the face by a baseball. Assessment findings include swelling and numbness to the left side of the face and left​ eyelid, as well as crepitus in the same area. Assessment of the eye reveals no obvious injury. The patient denies any problem with his​ eyesight, but is having difficulty visually tracking your finger upward. Which of the following would be appropriate care for this​ injury?

immobilize to a long spine board.

A conscious and alert​ 32-year-old was assaulted and struck in the jaw with a baseball bat. He complains of tremendous jaw pain and exhibits an unstable mandible. His airway is​ patent, breathing​ adequate, and radial pulse normal. In caring for this patient and his​ injury, you​ would:

​"I do not think that we will need to provide supplemental​ oxygen."

A construction worker complains of pain to both eyes after an extended period spent welding on a gas well. The light is bothering him and he rates the pain as a​ 10/10. He states that this occurred once before and was caused by the light of his welding tool. The primary assessment reveals no deficits and his vital signs​ are: pulse​ 80, respirations​ 14, blood pressure​ 132/68, and SpO2​ 98%. Although very​ painful, his vision is not compromised. Which of the following instructions indicate proper care of this​ patient

Suctioning the airway

A construction worker has been shot with a nail​ gun, resulting in a long nail going through his cheek and firmly embedding into the lower gum and jaw. There is considerable blood in his mouth and he is in excruciating pain. Given this​ scenario, which of the following would be your first​ priority?

Age-related changes in the renal system result in:

A decreased ability to clear wastes from the body and a decreased ability to conserve fluids when needed.

What does the "PID Shuffle" refer to?

A distinctive gait when the patient walks

Organic brain syndrome is defined as:

A dysfunction of the brain caused by abnormal physical or physiological function

suction blood from the airway as needed.

A female patient struck her face on the steering wheel in a​ head-on motor vehicle collision and has deformity and crepitus to the bridge of her nose. Further assessment reveals bleeding deep within the nasal passages. The patient has been fully immobilized to a long board. To best address the hemorrhage in the​ nasopharynx, the EMT​ would:

Which of the following statements regarding a patient refusal is correct? A. A patient who consumed a few beers will likely be able to refuse EMS treatment. B. A mentally competent adult has the legal right to refuse EMS care and transport. C. Advice given to a patient who refuses EMS treatment should not be documented. D. Documentation of proposed care is unnecessary if the patient refuses treatment.

A mentally competent adult has the legal right to refuse EMS care and transport.

Your EMS station has been designated a Baby​ Safe-Haven. As​ such, you know​ that: A. A mother or father can drop a baby off and leave with no questions asked B. You are legally required to provide basic supplies​ (e.g., bottles and​ diapers) to the mother or father C. You are legally required to provide the mother or father with resources to assist in the care of the baby D. A mother or father can bring a baby to the station for basic care issues

A mother or father can drop a baby off and leave with no questions asked

The clear structure that covers the pupil has been damaged.

A patient complaining of eye pain informs you that he was diagnosed with a corneal abrasion three days ago. He called 911 today because the discomfort has not yet resolved and he desires medical attention. Based on this​ information, which of the following is​ true?

covering both eyes with a bandage.

A patient complains of pain and discomfort to the left eye after being hit in the face with a cloud of dust while working at a manufacturing complex. As you specifically assess the​ eye, you note some redness to the globe but do not see any obvious foreign object. The most appropriate care you can provide to this patient would​ be:

Left lateral recumbent

A patient had an unknown chemical splashed into her left eye. You have been irrigating the eye on scene and are now getting ready to transfer her to the stretcher. Knowing that you will need to continue irrigation throughout​ transport, how will you position this patient on the​ stretcher?

after assessing the primary survey.

A patient has been splashed in the face with battery acid. He is complaining of being unable to see anything and severe burning to his face. You observe him holding both eyes tightly shut with significant burns on his cheeks and eyelids. In this​ situation, the EMT would assess the​ eyes:

upper jaw.

A patient with a maxillary fracture would have sustained injury to​ her:

blood in the eye chamber.

A patient with hyphema would​ exhibit:

Which of the following is an example of a psychiatric emergency?

A person violently attacking family members

The EMT shows that he understands the difference between a pneumothorax and a tension pneumothorax when he makes what​ statement?

A tension pneumothorax causes cardiac output to​ decrease; a simple pneumothorax does not affect cardiac​ output

iris.

A worker in a factory complex has a sliver of metal lodged in the colored portion of his eye. The EMT would recognize the foreign body as lying in​ the:

Difficulty speaking

A young boy riding an ATV did not see a chain stretched between two poles and hit it with his neck while traveling forward.​ Subsequently, he was thrown backward from the bike with significant force. He was not wearing a helmet. Emergency Medical Responders are with the patient and are holding manual​ in-line spinal immobilization as well as administering oxygen. When you are assessing this​ patient, which of the following signs would indicate the injury that must be addressed and managed​ first?

stabilizing the impaled object and providing immediate transport.

A young boy was running with a pencil and tripped. The pencil impaled the​ boy's left eye and remains lodged in place. He is in his​ mother's arms and both are crying hysterically. Appropriate care for the eye would​ include:

flush the eye with clean water from a nearby sink.

A young girl complains of eye pain after having sand thrown in her face by another girl at the park. When assessing her​ eyes, you note large particles of sand in the right eye. The sand appears to be on the​ conjunctiva, which also seems to be scratched and irritated. Your initial action would​ be:

He wipes blood from the face but not the eye

A young male golfer has been struck in the right eye with a golf club. The upper and lower lids are swollen shut and ecchymotic. There are also blood clots between the two eyelids. Which of the following indicates that the EMT is providing proper care to this​ patient?

Rolled gauze in the ear canal.

A young male patient has been involved in a motor vehicle accident. As an unrestrained​ driver, his head went through the​ driver's side​ window; in the process a portion of his ear was severed. Blood is now coming from the remaining portion of his left ear. In overseeing the care of this​ patient, which of the following would require corrective​ action?

17. A 19-year-old male complains of "not feeling right." His insulin and a syringe are on a nearby table. The patient says he thinks he took his insulin and cannot remember whether he ate. He is also unable to tell you the time or what day it is. The glucometer reads "error" after several attempts to assess his blood glucose level. In addition to administering oxygen, you should: A. contact medical control and administer oral glucose. B. assist him with his insulin injection and reassess him. C. request a paramedic ambulance to administer IV glucose. D. transport only with close, continuous monitoring en route.

A. contact medical control and administer oral glucose.

6. Diabetes is MOST accurately defined as a(n): A. disorder of carbohydrate metabolism. B. abnormally high blood glucose level. C. mass excretion of glucose by the kidneys. D. lack of insulin production in the pancreas.

A. disorder of carbohydrate metabolism.

20. When obtaining a SAMPLE history from a patient with diabetes, it would be MOST important to determine: A. if he or she has had any recent illnesses or excessive stress. B. approximately how much water the patient drank that day. C. if there is a family history of diabetes or related conditions. D. the name of the physician who prescribed his or her insulin.

A. if he or she has had any recent illnesses or excessive stress.

A 28-year-old female patient is found to be responsive to verbal stimuli only. Her roommate states that she was recently diagnosed with type 1 diabetes and has had difficulty controlling her blood sugar level. She further tells you that the patient has been urinating excessively and has progressively worsened over the last 24 to 36 hours. On the basis of this patient's clinical presentation, you should suspect that she: A. is significantly hyperglycemic. B. has a low blood glucose level. C. has a urinary tract infection. D. has overdosed on her insulin.

A. is significantly hyperglycemic.

8. Excessive eating caused by cellular "hunger" is called: A. polyphagia. B. polydipsia. C. dysphasia. D. dyspepsia.

A. polyphagia.

Insulin shock will MOST likely develop if a patient: A. takes too much of his or her prescribed insulin. B. markedly overeats and misses an insulin dose. C. eats a regular meal followed by mild exertion. D. misses one or two prescribed insulin injections.

A. takes too much of his or her prescribed insulin.

You are treating a 45-year-old woman who was stung by a hornet and has a rash. She tells you that she is allergic to hornets and has her own epinephrine auto-injector. She also tells you that she takes medication for hypertension. Her breath sounds do not reveal any wheezing, her breathing is unlabored, and her blood pressure is 154/94 mm Hg. What should you do if you are not able to make contact with medical control? A. Begin transport to the hospital and closely monitor her condition while en route B. Give her half the dose of her epinephrine in case her allergic reaction is delayed C. Administer her epinephrine, reassess her condition, and transport her promptly D. Begin immediate transport and request an intercept with a paramedic ambulance

A. Begin transport to the hospital and closely monitor her condition while en route

Which of the following statements regarding glucose is correct? A. Glucose is a simple sugar that is readily absorbed by the bloodstream B. Glucose is given to patients who are suspected of being hyperglycemic C. Glucose is usually administered by the EMT via the intravenous route D. Glucose is a complex sugar that rapidly absorbs into the bloodstream

A. Glucose is a simple sugar that is readily absorbed by the bloodstream

A 74-year-old woman complains of heaviness in her chest, nausea, and sweating that suddenly began about an hour ago. She is conscious and alert, but anxious. Her blood pressure is 144/84 mm Hg and her heart rate is 110 beats/min. She took two of her prescribed nitroglycerin (0.4-mg tablets) before your arrival but still feels heaviness in her chest. You should: A. Recall that geriatric patients often have slower absorption and elimination times, which may necessitate modification of the dosing of certain drugs B. Give her high-flow oxygen, avoid giving her any more nitroglycerin because it may cause a drop in her blood pressure, and transport C. Transport her at once and wait at least 20 minutes before you consider assisting her with a third dose of her prescribed nitroglycerin D. Assist her in taking one more of her nitroglycerin tablets, reassess her blood pressure, and contact medical control for further instructions

A. Recall that geriatric patients often have slower absorption and elimination times, which may necessitate modification and the dosing of certain drugs

How is nitroglycerin usually given by the EMT? A. Sublingually B. Injected C. Inhaled D. Orally

A. Sublingually

Which of the following patients is in decompensated shock? A. a 20-year-old female with absent radial pulses and dilated pupils B. a 23-year-old restless male with cool, clammy skin and tachycardia C. a 28-year-old female with pale skin and rapid, shallow respirations D. a 32-year-old male with anxiety and a systolic blood pressure of 110 mm Hg

A. a 20-year-old female with absent radial pulses and dilated pupils

While assisting a paramedic in the attempted resuscitation of a 55-year-old male in cardiac arrest, you should expect the paramedic to: A. administer drugs via the IV route to achieve the fastest effect B. give the patient activated charcoal to rule out a drug overdose C. withhold drug therapy until an intraosseous catheter is in place D. give the patient nitroglycerin to increase his blood pressure

A. administer drugs via the IV route to achieve the fastest effect

When perfusion to the core of the body decreases: A. blood is shunted away from the skin. B. decreased cardiac contractility occurs. C. blood is diverted to the gastrointestinal tract. D. the voluntary nervous system releases hormones.

A. blood is shunted away from the skin.

You are dispatched to a residence for a 40-year-old female who fainted. Upon your arrival, the patient is conscious and alert, and states that she is fine. Her husband tells you that she fainted after receiving news that her sister was killed in a car crash. You offer oxygen to the patient, but she refuses to accept it. At this point, your primary concern should be to: A. determine if she was injured when she fainted. B. provide emotional support regarding her sister. C. advise her that she needs to go to the hospital. D. obtain baseline vital signs and a medical history.

A. determine if she was injured when she fainted.

Neurogenic shock occurs when: A. failure of the nervous system causes widespread vasodilation. B. the spinal cord is severed and causes massive hemorrhaging. C. there is too much blood to fill a smaller vascular container. D. massive vasoconstriction occurs distal to a spinal cord injury.

A. failure of the nervous system causes widespread vasodilation.

Shock is the result of: A. hypoperfusion to the cells of the body. B. the body's maintenance of homeostasis. C. temporary dysfunction of a major organ. D. widespread constriction of the blood vessels.

A. hypoperfusion to the cells of the body.

Hypovolemic shock caused by severe burns is the result of a loss of: A. plasma. B. platelets. C. whole blood. D. red blood cells.

A. plasma.

Patients develop septic shock secondary to: A. poor vessel function and severe volume loss. B. an infection that weakens cardiac contractions. C. failure of the blood vessels to adequately dilate. D. weak vessel tone due to nervous system damage.

A. poor vessel function and severe volume loss.

A 70-year-old female was recently discharged from the hospital following a total hip replacement. Today, she presents with restlessness, tachycardia, and a blood pressure of 100/64 mm Hg. Her skin is warm and moist. You should be MOST suspicious that she is experiencing: A. septic shock. B. pump failure. C. a local infection. D. decompensated shock.

A. septic shock.

Which one of the following types of injuries is most likely to result in capillary bleeding only? a. Avulsion b. Penetration c. Abrasion d. Laceration

Abrasion

Which one of the types of injuries is MOST likely to result in capillary bleeding​ only?

Abrasion

A patient was assaulted and sustained various and multiple injuries. Which injury does the EMT recognize as presenting the greatest risk for infection? Select one: a. Contusion to the chin b. Abrasion to the right side of the face c. Hematoma to the left foot d. Closed crush injury to the knee

Abrasion to the right side of the face

A patient was assaulted and sustained various and multiple injuries. Which injury does the EMT recognize as presenting the greatest risk for​ infection?

Abrasion to the right side of the face

You have been called for a​ 96-year-old female with altered mental status. At the​ scene, you find that the patient is​ unresponsive, not​ breathing, and pulseless. The family provides you with a valid DNR order. What could you document as a presumptive sign of her​ death? A. Dilated pupils B. Closed eyes C. Absent blood pressure D. Poor or weak reflexes

Absent blood pressure

A patient was stabbed in the right anterior chest and is in obvious respiratory distress. As you perform the secondary​ assessment, which of the following signs​ and/or symptoms would concern you that she is suffering from a tension​ pneumothorax?

Absent breath sounds on the​ right, distended neck​ veins, tracheal deviation to the left

In pre-conventional reasoning, children:

Act almost purely to avoid punishment and to get what they want.

A​ middle-aged male's arm became caught in a grinding machine and was then immediately removed by coworkers. As you approach the​ patient, you note him to be holding a bloody towel over a severely deformed right arm. The patient is alert and​ anxious, and he has a patent airway. His breathing is​ adequate, and his left radial pulse is fast and weak. You note his skin to be cool and diaphoretic. Of the actions​ listed, which would be done first for this​ patient?

Administer oxygen and ensure the pulse oximeter reads​ > 95%.

A 37-year-old male is having a severe allergic reaction to penicillin. He does not have an epinephrine auto-injector and your protocols do not allow you to carry epinephrine on the ambulance. How should you proceed with the treatment of this patient?

Administer oxygen, transport at once, and request a paramedic intercept.

You have been dispatched to a residence for an​ 88-year-old female who is in and out of consciousness. The family provides you with a valid​ "Do Not​ Resuscitate" (DNR) order. Assessment of the patient shows her to be very confused with a low blood pressure and low oxygen level. Which action would be appropriate given the​ situation? A. Have the patient or family member sign a refusal of care form and clear the scene B. Provide care only if the​ patient's heart stops beating C. Provide transport but no additional care D. Administer supplemental oxygen

Administer supplemental oxygen

A patient presents with an arrow impaled in the right lower quadrant of his abdomen. He is in excruciating pain and states that he was accidentally shot by a friend while hunting. While there is no external bleeding coming from the​ injury, the surrounding skin is ecchymotic. Other assessment findings indicate early shock. Your partner manually provides stabilization to the arrow. Which action should the EMT perform​ first?

Administer supplemental oxygen.

Secondary sexual development begins during which age group?

Adolesence

The process of binding or sticking to a surface is called

Adsorption

When communicating with an older patient, it is important to remember that:

Age-related changes diminish the effectiveness of the eyes and ears.

When assessing an 80-year-old patient in shock, it is important to remember that:

Age-related changes in the cardiovascular system may make the patient less able to compensate for decreased perfusion.

What occurs when a patient is breathing very rapidly and shallowly?

Air moves primarily in the anatomic dead space and does not participate in pulmonary gas exchange.

Which of the following splinting devices would be MOST appropriate to use for a patient who has an open fracture of the forearm with external bleeding? Select one: A. Sling and swathe B. Vacuum splint C. Air splint D. Cardboard splint

Air splint

Which of the following conditions or situations would MOST likely make excited delirium worse?

Alcohol withdrawal

Which one of the following assessment findings indicates that a patient, who has suffered blood loss after being shot, is in an early stage of shock? a. Confused and anxious, pulse 144, BP 82/palpation, cool skin that is mottled b. Confused, pulse 44, BP 110/68 mmHg, cool and cyanotic skin c. Alert and anxious, pulse 96, BP 134/88 mmHg, pale and cool skin d. Slightly confused, pulse 116, BP 112/90 mmHg, warm skin that is flushed

Alert and anxious, pulse 96, BP 134/88 mmHg, pale and cool skin

A male patient has been shot in the chest with a​ small-caliber gun. What presentation indicates that he is in an early stage of​ shock?

Alert and​ anxious, pulse​ 96, BP​ 134/88 mmHg, pale and cool skin

If applying a dressing to control the bleeding of a patient's arm, the EMT should ________. Select one: A. apply direct pressure first B. use large or small gauze pads or dressings depending upon the size of the wound C. cover the entire wound, above and below, with the dressing D. All of these answers are correct.

All of these answers are correct

What mechanism(s) does the body use to control bleeding? Select one: A. Clotting B. Coagulation C. Vasoconstriction D. All of these answers are correct.

All of these answers are correct

A 23-year-old male experienced severe head trauma after his motorcycle collided with an oncoming truck. He is unconscious, has rapid and shallow breathing, and has copious bloody secretions in his mouth. How should you manage his airway?

Alternate 15 seconds of oral suctioning with 2 minutes of assisted ventilation.

Which of the following statements regarding the physical examination of a patient with a behavioral problem is correct?

Although the physical can be difficult, the patient's emotional state may be determined by noting facial expressions, pulse rate, and respirations

Which of the following is considered an organic brain syndrome?

Alzheimer dementia

Which of the following statements regarding the meter-does inhaler is correct

An MDI delivers the same amount of medication every time it is used

remove the lower dentures.

An elderly woman is suffering from a closed head injury after falling down five steps. She is responsive to painful stimuli and is breathing shallowly. When evaluating her​ airway, you note that she has both upper and lower dentures. Her upper dentures are​ secure, but her lower dentures are loose. Prior to providing positive pressure​ ventilation, you​ would:

Which patient may be treated under the guideline of implied​ consent? A. An adult patient with chest pain and shortness of breath who is refusing treatment and transport by EMS B. A​ 2-year-old boy who fell down a flight of stairs and whose parents want him taken to the hospital C. A​ 47-year-old female with severe abdominal pain who is refusing transport after placing the call to 911 for help D. An unresponsive​ 33-year-old female who is suspected of taking recreational drugs

An unresponsive​ 33-year-old female who is suspected of taking recreational drugs

Which of the following conditions or factors would be the LEAST likely to result in a change in behavior?

Antihypertensive medications

When a psychiatric emergency arises, a patient is most likely to exhibit which of the following behaviors?

Any behaviors that are in a violent threat to the patient, EMTs, or others

In which situation would the Good Samaritan law protect the​ EMT? A. The EMT forgets to provide oxygen to a patient with chest pain and the patient suffers a heart attack as a result B. A paramedic orders an EMT to administer a medication that the EMT legally may not​ give, and the patient dies as a result C. A patient is paralyzed from the waist down because an​ off-duty EMT did not take the appropriate measures in caring for him D. An​ off-duty EMT stops at the scene of an auto collision and provides proper​ care, but the patient dies anyway

An​ off-duty EMT stops at the scene of an auto collision and provides proper​ care, but the patient dies anyway

You are called for an alert and oriented female who had a cesarean section several days ago. Today she bent over and tore the surgical incision open. There is a moderate amount of bleeding coming from the site. What should the EMT do to control the​ bleeding?

Apply a large dressing over the incision.

A patient is bleeding severely from a severed femoral artery high in the groin region. Which of the following would MOST likely control the bleeding? Select one: A. Position the patient with his injured side down B. Apply a pelvic binder device to stabilize the pelvis C. Apply a topical hemostatic agent with direct pressure D. Apply chemical ice packs to the wound and transport

Apply a topical hemostatic agent with direct pressure

A construction worker has a metal rod impaled in his right forearm. Assessment of the injury indicates heavy bleeding from around the impaled object. What should you do​ first?

Apply direct pressure around the rod.

You have arrived on a scene where a young girl has pushed her arms through a plate glass window. She is responsive to painful stimuli and has blood spurting from a large laceration on her right upper arm. Your paramedic partner yells for you to immediately apply direct pressure to the laceration to prevent further blood​ loss, while he works to control the airway. You have yet to apply latex​ gloves, but are wearing goggles and a gown. What would you do​ next?

Apply gloves prior to applying direct pressure over the injury site.

You are assessing a 45-year-old female who is severely depressed. She states that is seems as though her entire world is crashing down around her. She further states that she has had frequent thoughts of suicide, but is not sure if she can actually go through with it. How should you manage this situation?

Ask the patient if she has developed a suicidal plan

A high school football player was hit from behind and is complaining of severe back pain and numbness to his right leg. Manual in-line spinal stabilization is being maintained by an assistant coach. Once at the patient's side, what should you do next? a. Apply a cervical spine immobilization device. b. Carefully remove the helmet. c. Assess the airway and breathing. d. Disconnect the face mask from the helmet

Asses the airway and breathing

You have been dispatched for a​ 15-year-old female with a headache. On​ scene, you find the patient holding her​ 6-month-old son in her arms. This young mother states that she has had a migraine headache for two days and would like to be transported to the hospital for further evaluation. As an​ EMT, you​ should: A. Treat the patient under the doctrine of implied consent B. Refuse transport since the patient does not have a true emergency C. Assess and transport the patient as she desires D. Obtain a​ parent's consent to treat the​ patient, given her age

Assess and transport the patient as she desires

You have just applied a pressure dressing to the leg of a woman who suffered a deep laceration to her calf from a piece of broken glass. What should you do​ next?

Assess for a pedal pulse.

A young female was involved in an altercation and was stabbed in the abdomen with an ice pick. After being​ stabbed, she quickly removed the ice pick. She states that there was not much bleeding but does say that it​ "stings" quite a bit. Assessment reveals a small puncture wound to the right upper quadrant with some dried blood around the site. What BEST describes the primary focus of the EMT in caring for this​ patient?

Assessing for internal blood loss or shock

You have been called to transfer a trauma patient from a rural hospital emergency department to the emergency department of a large urban medical center. Which action will best decrease your chance of becoming involved in an EMTALA​ violation? A. Assessing the patient for stability prior to transfer B. Determining the exact location to which the patient is to be transported C. Obtaining a copy of the​ patient's insurance and billing information D. Taking vital signs every 5 minutes during transport

Assessing the patient for stability prior to transfer

During your assessment of a patient with blunt chest trauma, you note that the patient has shallow breathing and paradoxical movement of the left chest wall. You should:Select one: a. request a paramedic to decompress the chest. b. apply high-flow oxygen via nonrebreathing mask. c. make note of it and continue your assessment. d. assist ventilations with a bag-valve mask.

Assist ventilations with a bag-valve mask

A 37 year old male is found unconscious in his car. His airway is patent and his respirations are rapid and labored. As you and your partner are assessing and treating the patient, a police officer hands you a medication named alupent, which he found in the backseat of the patients car. This medication suggests that the patient has a history of

Asthma

A general impression of the patient is formed by an EMT:

At a distance when the patient is first seen

The decline in cardiac function that commonly occurs in late adulthood is MOST often related to:

Atherosclerosis.

An EMT may administer aspirin to a patient if

Authorization from medical control has been obtained

You have been dispatched for a 61 year old female in cardiac arrest. Emergency Medical Responders are on scene. In your community, all EMS practitioners are trained according to the National EMS Scope of Practice Model. Given this, which type of care do you expect the EMRs to be providing?

Automated external defibrillation

Which of the following is a good guidelines for physical examination of any patient?

Avoid touching without permission

airway compromise.

A​ 15-year-old female was struck in the mouth with a baseball bat. She has lost her front teeth and is spitting blood. The​ EMT's priority concern when caring for this patient would​ be:

"When we get to the​ hospital, we can discuss this with the​ doctor."

A​ 19-year-old female was assaulted by her intoxicated boyfriend. After punching her in the​ chest, he slashed her face with a knife. Assessment reveals her to be stable with a large laceration from beneath her right eye to her right ear. She is very upset and crying and repeatedly asks you if this injury will leave a permanent scar on her face. Your best response would​ be:

"I need someone to place a gloved hand over the stab​ wound."

A​ 21-year-old male has a stab wound to the left side of his neck. Emergency Medical Responders are holding manual​ in-line spinal stabilization. As you approach the​ patient, you note blood on his shirt and​ pale-looking skin color. As team​ leader, what direction would you give​ first?

Unusual sensitivity to light

A​ 21-year-old-female has been struck in the left eye by a softball. Which finding is most concerning for an eye​ injury?

Apply a cervical collar.

A​ 22-year-old male attempted to hang himself in a garage using chains suspended from the roof trusses. He was quickly found by family members and taken down to the ground. Currently he is complaining of neck pain as well as mild difficulty breathing. Assessment shows his airway to be​ patent, breathing​ adequate, and skin warm and dry. His neck is​ red, but free of any subcutaneous air or swelling. Which of the following will you do first when caring for this​ patient?

Suctioning the airway to keep it free of blood.

A​ 23-year-old male struck his face on the steering wheel after he struck another car from behind. His nose is deformed and actively bleeding. In his​ care, which of the following would be a​ priority?

Involuntary consent would most likely apply​ to: A. A confused elderly woman who is in the presence of the person with her acting power of attorney B. A​ 23-year-old prisoner with a laceration to the right cheek and refusing care C. An alert and oriented​ 34-year-old male who is coughing up blood and refusing care D. A​ 3-year-old boy found wandering in the street with abrasions and cuts on his bare feet

A​ 23-year-old prisoner with a laceration to the right cheek and refusing care

Non-emergent transport to the hospital

A​ 3-year-old has stuck a crayon in his nose. Assessment reveals the crayon to be deeply embedded in the right nostril with some irritation and swelling noted. His vital signs​ are: pulse​ 124, respiration​ 20, and SpO2​ 100%. Which of the following would be most appropriate when caring for this​ child?

prevent air entry into the circulatory system.

A​ 36-year-old male patient was involved in an altercation with a knife and has suffered a​ 2-inch laceration to the right side of the neck. Bleeding was controlled prior to your arrival by Emergency Medical Responders. Your paramedic partner has asked you to apply an occlusive dressing to the wound. You recognize that the primary benefit of the action is​ to:

21. The normal blood glucose level, as measured by a glucometer, is between: A. 60 and 80 mg/dL. B. 80 and 120 mg/dL. C. 130 and 150 mg/dL. D. 160 and 200 mg/dL.

B. 80 and 120 mg/dL.

25. Which of the following statements regarding diabetic coma is correct? A. Diabetic coma can be prevented by taking smaller insulin doses. B. Diabetic coma typically develops over a period of hours or days. C. Patients with low blood glucose levels are prone to diabetic coma. D. Diabetic coma rapidly progresses once hyperglycemia develops.

B. Diabetic coma typically develops over a period of hours or days.

12. Glipizide, a non-insulin-type medication, is another name for: A. Glucophage. B. Glucotrol. C. Micronase. D. Diabinese.

B. Glucotrol.

Which of the following statements regarding sickle cell disease is correct? A. Sickle cell disease is an inherited blood disorder that causes the blood to clot too quickly. B. In sickle cell disease, the red blood cells are abnormally shaped and are less able to carry oxygen. C. The red blood cells of patients with sickle cell disease are round and contain hemoglobin A. D. Because of their abnormal shape, red blood cells in patients with sickle cell disease are less apt to lodge in a blood vessel.

B. In sickle cell disease, the red blood cells are abnormally shaped and are less able to carry oxygen.

18. A 37-year-old female with a history of diabetes presents with excessive urination and weakness of 2 days' duration. You apply 100% oxygen and assess her blood glucose level, which reads 320 mg/dL. If this patient's condition is not promptly treated, she will MOST likely develop: A. severe insulin shock. B. acidosis and dehydration. C. complete renal failure. D. hypoxia and overhydration.

B. acidosis and dehydration.

13. You respond to a movie theater for a 70-year-old male who is confused. His wife tells you he has type 2 diabetes but refuses to take his pills. Your assessment reveals that the patient is diaphoretic, tachycardic, and tachypneic. Initial management for this patient should include: A. administering one to two tubes of oral glucose. B. applying a nonrebreathing mask at 15 L/min. C. assisting the patient with his diabetic medication. D. performing a rapid exam and obtaining vital signs.

B. applying a nonrebreathing mask at 15 L/min.

19. Insulin functions in the body by: A. producing new glucose as needed. B. enabling glucose to enter the cells. C. increasing circulating blood glucose. D. metabolizing glucose to make energy.

B. enabling glucose to enter the cells.

4. Proper procedure for administering oral glucose to a patient includes all of the following, EXCEPT: A. assessing the patient's mental status. B. ensuring the absence of a gag reflex. C. checking the medication's expiration date. D. requesting permission from medical control.

B. ensuring the absence of a gag reflex.

15. Patients with uncontrolled diabetes experience polyuria because: A. they drink excess amounts of water due to dehydration. B. excess glucose in the blood is excreted by the kidneys. C. low blood glucose levels result in cellular dehydration. D. high blood sugar levels cause permanent kidney damage.

B. excess glucose in the blood is excreted by the kidneys.

10. Diabetic coma is a life-threatening condition that results from: A. hypoglycemia, excess insulin, and dehydration. B. hyperglycemia, ketoacidosis, and dehydration. C. hypoglycemia, dehydration, and ketoacidosis. D. hyperglycemia, excess insulin, and ketoacidosis.

B. hyperglycemia, ketoacidosis, and dehydration.

3. Glutose is a trade name for: A. glucotrol. B. oral glucose. C. micronase. D. glucophage.

B. oral glucose.

23. Hypoglycemic crisis tends to develop more often and more severely in children because: A. they have larger glucose stores than adults do. B. they do not always eat correctly and on schedule. C. their cells do not uptake glucose as fast as adults' do. D. their low activity levels cause rapid glucose depletion.

B. they do not always eat correctly and on schedule.

In infants and children, a capillary refill time (CRT) that is greater than ______ second(s) is a sign of poor peripheral perfusion. A. 1 B. 2 C. 3 D. 4

B. 2

Which of the following is required for an EMT to administer a drug to a patient? A. A patient with stable vital signs B. Medical direction approval C. A patient with an altered level of consciousness D. An alert adult patient

B. Medical direction approval

Clinical signs of compensated shock include all of the following, EXCEPT: A. cool and clammy skin. B. absent peripheral pulses. C. restlessness or anxiety. D. rapid, shallow breathing.

B. absent peripheral pulses.

You are dispatched to a state park for a young female experiencing an allergic reaction. Your assessment reveals that her breathing is severely labored and her blood pressure is very low. You carry epinephrine auto-injectors on your ambulance and have been trained and approved by your medical director to administer them. As your partner gives the patient supplemental oxygen, you attempt to contact medical control but do not have a signal from your cell phone. You should: A. immediately load the patient into the ambulance, begin transport, and reattempt to contact medical control when you receive a cell signal. B. administer epinephrine to the patient, begin immediate transport, and attempt to contact medical control en route to the hospital C. Place her in a supine position, keep her warm, begin transport to the hospital, and request a paramedic intercept en route D. notify dispatch and request that a paramedic unit respond to the scene so they can administer epinephrine to the patient

B. administer epinephrine to the patient, begin immediate transport, and attempt to contact medical control en route to the hospital

A construction worker fell approximately 30 feet. He is semiconscious with rapid, shallow respirations. Further assessment reveals deformity to the thoracic region of his spine. His blood pressure is 70/50 mm Hg, his pulse is 66 beats/min and weak, and his skin is warm and dry. In addition to spinal immobilization and rapid transport, the MOST appropriate treatment for this patient includes: A. oxygen via nonrebreathing mask, blankets for warmth, and elevation of his head. B. assisted ventilation, thermal management, and elevation of the lower extremities. C. oxygen via nonrebreathing mask, thermal management, and elevation of his legs. D. assisted ventilation, preventing hyperthermia, and elevating his lower extremities.

B. assisted ventilation, thermal management, and elevation of the lower extremities.

A 37-year-old male is found unresponsive in his car. His airway is patent and his respirations are rapid and labored. As you and your partner are assessing and treating the patient, a police officer hands you a medication named Alupent, which he found in the backseat of the patient's car. This medication suggests that the patient has a history of: A. allergic reactions B. asthma C. heart disease D. hypertension

B. asthma

When assessing a patient with signs and symptoms of shock, it is MOST important to remember that: A. the patient's respirations are deep during the early stages of shock. B. blood pressure may be the last measurable factor to change in shock. C. multiple fractures are the most common cause of hypovolemic shock. D. irreversible shock often responds well to a prompt blood transfusion.

B. blood pressure may be the last measurable factor to change in shock.

One of the primary waste products of normal cellular metabolism that must be removed from the body by the lungs is: A. lactic acid. B. carbon dioxide. C. pyruvic acid. D. carbon monoxide.

B. carbon dioxide.

Which of the following injuries would MOST likely cause obstructive shock? A. liver laceration B. cardiac tamponade C. simple pneumothorax D. spinal cord injury

B. cardiac tamponade

A 56-year-old male is found semiconscious by his wife. Your assessment reveals that his respirations are rapid and shallow, his pulse is rapid and irregular, and his blood pressure is low. The patient's wife states that he complained of left arm pain and nausea the day before, but would not allow her to call 9-1-1. The MOST likely cause of this patient's present condition is: A. acute myocardial infarction. B. cardiogenic hypoperfusion. C. severe septic hypoperfusion. D. a ruptured aortic aneurysm.

B. cardiogenic hypoperfusion.

Which of the following would MOST likely result in hemorrhagic shock? A. severe vomiting B. liver laceration C. excessive sweating D. repeated diarrhea

B. liver laceration

EMTs respond to a known heroin user who is unresponsive. If they give naloxone (Narcan) to this patient, the EMTs should recall that: A. naloxone should be administered in increments of 2mg B. naloxone administration could cause seizures in this patient C. naloxone should not be given if the patient's breathing is slow D. the effects of naloxone last longer than most opioid drugs

B. naloxone administration could cause seizures in this patient

When the body senses a state of hypoperfusion, the sympathetic nervous system releases epinephrine, the effects of which include: A. tachypnea. B. tachycardia. C. vasodilation. D. restlessness.

B. tachycardia.

Which of the following MOST accurately describes septic shock?

Bacterial damage to the vessel wall, leaking blood vessels, and vasodilation

Which statement about dressings and bandages is​ TRUE?

Bandages should not be applied until bleeding is controlled.

When assessing a patient with a behavioral crisis, you should:

Be direct and clearly state your intentions

In contrast to a behavioral crisis, a psychiatric emergency occurs when a person:

Becomes agitated or violent and is a threat to him or herself or others

Your patient is a​ 40-year-old male who fell 20 feet from a ledge while hiking. The park ranger is maintaining manual​ in-line spinal stabilization. The patient is combative and​ confused, but the airway is open and breathing is adequate. The patient has a radial pulse of​ 112, blood pressure of​ 96/76, and SpO2 of 91 percent. What should you do​ first?

Begin administering supplemental oxygen.

____________is what you can see of a person's response to the environment

Behavior

General guidelines for managing a patient with a behavioral emergency include:

Being prepared to spend extra time with the patient

Subcutaneous injections deliver the medication

Between the skin and the muscle

A medication with antagonistic properties is one that

Blocks receptor sites and prevents other chemicals from attaching to them

You have been called for a​ 42-year-old male complaining of back pain. On​ scene, the man tells you that he was lying on the floor when his​ 5-year-old son unexpectedly jumped feet first onto his back. He gives a medical history of high blood pressure and​ asthma, for which he takes medications. As you assess the​ patient, which one of the following signs or symptoms provides the greatest indication that the patient has sustained an abdominal​ injury?

Blood in his urine.

Your patient sustained blunt trauma to the chest in a motor-vehicle collision, and you suspect shock. While conducting the reassessment, which one of the following findings best indicates that the patient is deteriorating? a. Blood oozing from an abdominal laceration b. Restless and confused mental status c. Blood pressure 86/50 mmHg d. Heart rate 124 beats per minute

Blood pressure 86/50 mmHg

An infant's blood pressure typically increases with age because:

Blood pressure directly corresponds to body weight

Which of the following statements regarding the blood pressure is correct?

Blood pressure is usually not measured in children younger than 3 years of age.

When assessing a​ head-injured patient, what finding is most indicative of increasing pressure within the​ skull?

Blood pressure of​ 192/106 mmHg

A​ 24-year-old female was an unrestrained driver of a car that struck a​ guardrail, causing her to sustain blunt trauma to the chest and abdomen. Vitals obtained by Emergency Medical Responders​ (EMRs) are pulse​ 120, respirations​ 22, blood pressure​ 100/58 mmHg, and SpO2​ 96%. As you transport her to the​ hospital, what finding indicates that the patient is acutely​ deteriorating?

Blood pressure​ 86/50 mmHg

What statement regarding contusions is​ true?

Blood vessels in the dermal layer have ruptured.

Which of the following pupillary changes would indicate depressed brain function?

Both pupils dilate with introduction of a bright light.

Which of the following oxygen flowmeters is NOT affected by gravity and can be used in any position when attached to an oxygen cylinder?

Bourdon-gauge flowmeter

A patient with Cushing reflex is most likely incurring what​ pathophysiology?

Brain herniation

A patient suffered head trauma after falling off a ladder and is now unable to regulate the blood pressure and maintain a heart rate. What region of the brain that normally controls these functions may be​ injured?

Brainstem

A​ middle-aged male immediately died following a​ self-inflicted gunshot wound to the head. Which portion of the central nervous system was most likely damaged in order to cause such a rapid​ death?

Brainstem

While you are en route to a domestic dispute with possible​ injuries, the police contact you and report a male patient with arterial bleeding from a stab wound to the arm. Given this​ information, what would you expect to see on​ arrival?

Bright red blood that is spurting with each beat of the heart

A​ 68-year-old female patient is complaining of a headache and generalized weakness. Her husband informs you that she was with a friend yesterday and was involved in a motor vehicle collision. She did strike her face on the​ dashboard, but refused treatment on scene. Her past medical history includes a stroke with right arm weakness and high blood pressure for which she takes medications. When assessing this​ patient, what finding should concern the EMT​ most?

Bruising behind her left ear

Which finding is most consistent with a basilar skull​ fracture?

Bruising behind the ear

Everyone exhibits some signs and symptoms of mental illness at some point in life _________.

But that does not mean a person is mentally ill

How do poisons typically act to harm the body?

By changing the normal metabolism of cells or by destroying them

5. Which of the following statements regarding glucose is correct? A. Most cells will function normally without glucose. B. Blood glucose levels decrease in the absence of insulin. C. The brain requires glucose as much as it requires oxygen. D. The brain requires insulin to allow glucose to enter the cells.

C. The brain requires glucose as much as it requires oxygen.

26. In contrast to insulin shock, diabetic coma: A. is rapidly reversible if oral glucose is given. B. commonly results in excess water retention. C. can only be corrected in the hospital setting. D. is a rapidly developing metabolic disturbance.

C. can only be corrected in the hospital setting.

27. Ketone production is the result of: A. acidosis when blood glucose levels are low. B. blood glucose levels higher than 120 mg/dL. C. fat metabolization when glucose is unavailable. D. rapid entry of glucose across the cell membrane.

C. fat metabolization when glucose is unavailable.

7. Hemoglobin is: A. the fluid portion of the blood that transports cells throughout the body. B. essential for the formation of clots, such as when vessel damage occurs. C. found within the red blood cells and is responsible for carrying oxygen. D. a key component of the blood and is produced in response to an infection.

C. found within the red blood cells and is responsible for carrying oxygen.

28. You are treating a 40-year-old male with a documented blood sugar reading of 300 mg/dL. The patient is semiconscious and breathing shallowly, and is receiving assisted ventilation from your partner. You should recognize that definitive treatment for this patient includes: A. oxygen. B. glucagon. C. insulin. D. dextrose.

C. insulin.

22. A 29-year-old female presents with confusion and disorientation. Her respirations are rapid and shallow and her pulse is 120 beats/min and thready. She is markedly diaphoretic and has an oxygen saturation of 89%. You should: A. transport immediately. B. administer oral glucose. C. provide ventilatory support. D. treat her for hyperglycemia.

C. provide ventilatory support.

When assessing an elderly male who complains of nausea and generalized weakness, you find that he takes atorvastatin (Lipitor) and amlodipine (Norvasc). These medications suggest a history of: A. Reactive airway disease B. Bacterial infection C. Cardiovascular disease D. Non-insulin-dependent diabetes

C. Cardiovascular disease

Which of the following medication routes would be the MOST appropriate to use in an unresponsive patient when intravenous access cannot be obtained? A. Transcutaneous B. Intramuscular C. Intraosseous D. Subcutaneous

C. Intraosseous

Nitroglycerin, when given to patients with cardiac-related chest pain: A. Increases blood return to the right atrium B. Increases myocardial contraction force C. Relaxes the walls of the coronary arteries D. Constricts the veins throughout the body

C. Relaxes the walls of the coronary arteries

Which of the following is an example of a rules-based medication error? A. The EMT administers the correct drug, but gives it by the wrong route B. The EMT administers a drug that is contraindicated for the patient C. The EMT administers a drug that is not approved by the medical director D. The EMT accidentally gives a higher drug dose than what is indicated

C. The EMT administers a drug that is not approved by the medical director

Which of the following medication routes delivers a drug through the skin over an extended period of time, such as a nitroglycerin or nicotine patch? A. Sublingual B. Subcutaneous C. Transcutaneous D. Intraosseous

C. Transcutaneous

Which of the following MOST accurately describes septic shock? A. bacterial infection of the nervous system with widespread vasodilation B. widespread vasoconstriction and plasma loss due to a severe viral infection C. bacterial damage to the vessel wall, leaking blood vessels, and vasodilation D. viral infection of the blood vessels, vascular damage, and vasoconstriction

C. bacterial damage to the vessel wall, leaking blood vessels, and vasodilation

Pulmonary edema and impaired ventilation occur during: A. septic shock. B. neurogenic shock. C. cardiogenic shock. D. anaphylactic shock.

C. cardiogenic shock.

A 27-year-old male was stabbed in the chest during a disagreement at a poker game. As you approach him, you see that a knife is impaled in his chest. Before you make physical contact with the patient, it is MOST important to: A. form a general impression. B. call for an ALS ambulance. C. follow standard precautions. D. ask bystanders what happened.

C. follow standard precautions.

Cardiogenic shock may result from all of the following, EXCEPT: A. heart attack. B. increased afterload. C. increased preload. D. poor contractility.

C. increased preload.

Hypotension in a child with blunt or penetrating trauma is particularly significant because: A. it typically develops earlier in children than it does in adults. B. the most likely cause of the hypotension is respiratory failure. C. it often indicates the loss of half of his or her blood volume. D. most children with hypotension die in the prehospital setting.

C. it often indicates the loss of half of his or her blood volume.

Shortly after assisting a 65-year-old female with her prescribed nitroglycerin, she begins complaining of dizziness and experiences a drop of 30 mm Hg in her systolic blood pressure. The patient remains conscious and her breathing is adequate. You should: A. wait 5 minutes and reassess her blood pressure B. transport her in a sitting position C. place her in a supine position D. assist ventilations with a bag-valve mask

C. place her in a supine position

Temporary, widespread vasodilation and syncope caused by a sudden nervous system reaction MOST accurately describes: A. vasovagal shock. B. neurogenic shock. C. psychogenic shock. D. neurologic shock.

C. psychogenic shock.

You are transporting a 33-year-old male who was involved in a motor vehicle crash. You have addressed all immediate and potentially life-threatening conditions and have stabilized his condition with the appropriate treatment. With an estimated time of arrival at the hospital of 20 minutes, you should: A. take his vital signs in 15 minutes. B. arrange for an ALS rendezvous. C. reassess his condition in 5 minutes. D. repeat your secondary assessment.

C. reassess his condition in 5 minutes.

Which of the following clinical signs is unique to anaphylactic shock? A. pallor B. dizziness C. wheezing D. hypotension

C. wheezing

A 40-year-old male intentionally cut his wrist out of anger after losing his job. Law enforcement has secured the scene prior to your arrival. As you enter the residence and visualize the patient, you can see that he has a towel around his wrist and a moderate amount of blood has soaked through it. You should:

Calmly identify yourself to the patient

Medications encased in a gelatin shell taken by mouth are called

Capsules

Which of the following incidents does NOT require a report to be filed with local authorities? A. Spousal abuse B. Animal bites C. Gunshot wounds D. Cardiac arrest

Cardiac arrest

An unrestrained female driver hit a utility pole at a moderate rate of speed and struck the steering wheel with her chest. Her airway is​ open, and she states that it is painful to breathe. Her pulse is moderate in strength and irregular. Breath sounds are equal​ bilaterally, and there is no jugular venous distention noted. Assessment of her chest reveals bruising and instability to the sternum. When​ asked, she denies any past medical history. Based on this mechanism and assessment​ findings, the EMT should be suspicious for what​ condition?

Cardiac contusion

When assessing an elderly male who complains of nausea and generalized weakness, you find that he takes simvastatin (vytorin) and clopidoherl(plavix). This medication regimen suggests a history of

Cardiovascular disease

Irregular respirations characterized by an increasing rate and depth of breathing followed by periods of apnea are called:

Cheyne-Stokes respirations.

Which of the following is a severe burn in a 35-year- old patient?

Circumferential partial-thickness burn to the chest

If an individual is bringing a lawsuit against your EMS system for perceived improper​ care, which type of liability action will it probably​ be? A. Criminal B. State C. Federal D. Civil

Civil

You suspect that an unhelmeted male patient who was ejected from a motorcycle may have a basilar skull fracture. As you perform the secondary​ assessment, which finding would reinforce this​ suspicion?

Clear fluid coming from the right ear and left nostril

You have arrived on the scene of a motor vehicle collision. A car has gone off the road and into a utility pole. The pole is broken and wires are hanging just above the car. You do not see any sparking or arcing of the wires. The driver of the car is slumped over the steering wheel and not moving. Several bystanders around the car are yelling for you to help. Which steps represent the correct order of your response?

Clear the bystanders, allow the fire department to secure the wires, access the patient, extricate the patient, transport

A baseball player was struck in the side of the face by a baseball. Assessment findings include swelling and numbness to the left side of the face and left eyelid, as well as crepitus to the same area. Assessment of the eye reveals no obvious injury. The patient denies any problem with his eyesight, but is having difficulty visually tracking your finger upward. Which of the following would be appropriate care measures for this injury?

Cold pack to the face and eye

A​ 14-year-old male was struck in the chest by a baseball. On your​ arrival, he is in cardiac arrest. What is the most likely explanation for the sudden cardiac​ arrest?

Commotio cordis

Which one of the following best explains the reason why emergency services communications are recorded?

Communications can become part of a legal record if need be.

An elderly patient has fallen down a flight of stairs and is complaining of neck and back pain as well as weakness to both legs. The primary assessment reveals no life threats to the airway, breathing, or circulation. Manual in-line spinal stabilization is being maintained. Which one of the following should the EMT do next?

Complete the secondary assessment looking for injuries.

Your best protection when an alert and oriented patient with the capacity to make rational decisions refuses EMS care or transport is​ to: A. Inform the​ patient's family physician of what happened B. Completely and thoroughly document the incident C. Contact the patient at a later time to see how she is doing D. Have your partner sign the refusal form as a witness

Completely and thoroughly document the incident

A male soccer player was struck in the head with a soccer ball. Players state that he was dazed for several seconds following the impact and then asked the same questions over and over.​ Presently, he is conscious and oriented to​ person, but confused to place and time. He also has a reddened area to the side of his head and face. As you proceed with your​ assessment, his memory continues to improve. Based on these​ findings, the EMT should suspect which of the following​ injuries?

Concussion

You are treating a middle aged man with chest discomfort. He has a history of three previous heart attacks and takes nitroglycerin as needed for for Chest pain. You have standing orders to administer aspirin to patients with suspected cardiac related chest pain or discomfort. While your partner is preparing to give oxygen to the patient, you should

Confirm that the patient is not allergic to aspirin, give him the appropriate dose of aspirin, and document the time and the dose

A patient who was involved in an altercation has been struck on the side of the head with a baseball bat in addition to suffering several blows to the arms and legs. In assessing this​ patient, which signs or symptoms BEST indicate that the patient has suffered a brain​ injury?

Confusion and combativeness

The inner surface of the eyelids and the exposed surface of the eye itself are covered by a delicate membrane called the:

Conjunctiva

Your patient is a young adult male who is in police custody after he crashed his car into a utility pole. There is minor​ front-end damage to the​ vehicle, and the air bags deployed. The patient was not wearing a seat belt and has an abrasion on his forehead. He is awake and oriented to​ person, time, and place. His speech is​ slurred, and he has an odor of alcohol on his breath. The arresting officer states that she just wants you to check the patient out before she transports him to jail. Which action is MOST​ appropriate?

Contact medical command.

An intoxicated​ 72-year-old male with an alcoholic history has fallen at home and has a laceration on the back of his head. He states that he does not want treatment and becomes combative when you try to talk to or assess him. What is your next best course of​ action? A. Contact medical direction B. Transport him against his​ will, but do not provide treatment C. Have him sign a refusal of care form D. Transport him by permission of family

Contact medical direction

A patient who was transported by EMS for a laceration on her arm is suing the service and the hospital because her laceration became​ infected, resulting in loss of the extremity.​ However, it becomes known that the patient was appropriately treated by EMS and the emergency​ department, but failed to seek​ follow-up medical care at a health clinic and did not take the antibiotics that were prescribed to her. Which element would most likely be used by the medical community as a defense in this​ case? A. Res ipsa loquitor B. Intentional tort C. Proximate cause D. Contributory negligence

Contributory negligence

An alert and oriented patient has had a portion of his hand amputated by a table saw. Assessment reveals the patient to be in great pain with a moderate amount of dark red blood still coming from the injury site. His airway is​ open, and his breathing is​ fast, but adequate. The radial pulse is rapid and​ strong, and his skin is warm and dry. The​ patient's hand has been placed in a bag of ice by coworkers. What should the EMT do​ immediately?

Control the bleeding with direct pressure.

Assessment of an​ 88-year-old female who fell reveals bruising to her right buttock. The skin is intact and the patient complains of tenderness to the area when you palpate. As a knowledgeable​ EMT, you would recognize what type of​ injury?

Contusion

A window washer has fallen 20 feet from scaffolding while washing the windows on an office building. He is alert and oriented, but states that he cannot move or feel his legs. Additionally, because his blood pressure is 82/48 mmHg, you suspect spinal shock. What other assessment finding reinforces your suspicion of spinal shock?

Cool and clammy skin

If the patient has sustained an abdominal evisceration the EMT​ should:

Cover the wound with a moist dressing followed by an occlusive dressing.

A patient was building a bomb in his garage when it accidentally detonated. Assessment findings from the rapid secondary assessment reveal intestine and a portion of the liver protruding from the right side of the abdomen. Proper care of this injury would include which of the​ following?

Covering the exposed organs with a large sterile dressing soaked with sterile water

14. To which of the following diabetic patients should you administer oral glucose? A. an unconscious 33-year-old male with cool, clammy skin B. a conscious 37-year-old female with nausea and vomiting C. a semiconscious 40-year-old female without a gag reflex D. a confused 55-year-old male with tachycardia and pallor

D. a confused 55-year-old male with tachycardia and pallor

2. Assessment of a patient with hypoglycemia will MOST likely reveal: A. sunken eyes. B. hyperactivity. C. warm, dry skin. D. combativeness.

D. combativeness.

30. Diabetic ketoacidosis occurs when: A. blood glucose levels rapidly fall. B. the cells rapidly metabolize glucose. C. the pancreas produces excess insulin. D. insulin is not available in the body.

D. insulin is not available in the body.

11. When assessing an unconscious diabetic patient, the primary visible difference between hyperglycemia and hypoglycemia is the: A. patient's mental status. B. rate of the patient's pulse. C. presence of a medical identification tag. D. rate and depth of breathing.

D. rate and depth of breathing.

29. You respond to a residence for a patient who is "not acting right." As you approach the door, the patient, a 35-year-old male, begins shouting profanities at you and your partner while holding a baseball bat. The man is confused and diaphoretic, and is wearing a medical identification bracelet. You should: A. calm him down so you can assess him. B. be assertive and talk the patient down. C. contact medical control for instructions. D. retreat at once and call law enforcement.

D. retreat at once and call law enforcement.

24. A 42-year-old male is found unresponsive on his couch by a neighbor. During your assessment, you find no signs of trauma and the patient's blood glucose level is 75 mg/dL. His blood pressure is 168/98 mm Hg, his heart rate is 45 beats/min and bounding, and his respirations are 8 breaths/min and irregular. The patient is wearing a medical alert bracelet that states he has hemophilia. You should: A. administer oxygen via a nonrebreathing mask, apply oral glucose in between his cheek and gum, and transport. B. suspect that he has internal bleeding and is in shock, administer high-flow oxygen, and transport at once. C. administer high-flow oxygen, perform a detailed secondary assessment at the scene, and transport promptly. D. suspect that he has intracranial bleeding, assist his ventilations, and transport rapidly to an appropriate hospital.

D. suspect that he has intracranial bleeding, assist his ventilations, and transport rapidly to an appropriate hospital.

A 62-year-old male is seen with crushing chest pain, which he describes as being the same kind of pain that he had with a previous heart attack. He has prescribed nitroglycerin but states that he has not taken any. After administering supplemental oxygen if needed and contacting medical control, you should: A. Administer the nitroglycerin unless he has taken Viagra within the past 72 hours B. Begin immediate transport and request a rendezvous with a paramedic unit C. Administer up to three doses of nitroglycerin before assessing his blood pressure D. Assist him with his nitroglycerin unless his systolic blood pressure is less than 100 mm Hg

D. Assist him with his nitroglycerin unless his systolic blood pressure is less than 100 mm Hg

The amount of medication that is given is known as the ______. A. Contraindication B. Indication C. Side effect D. Dose

D. Dose

Which of the following statements regarding anaphylactic shock is MOST correct? A. Anaphylactic shock occurs immediately after a person is sensitized to an allergen. B. Sensitized people will experience less severe reactions upon subsequent exposure. C. Anaphylactic shock is the result of immune system failure due to a toxic exposure. D. Each subsequent exposure following sensitization often produces a more severe reaction.

D. Each subsequent exposure following sensitization often produces a more severe reaction.

What is the route of administration for the EpiPen auto-injector? A. Intravenous B. Sublingual C. Intraosseous D. Intramuscular

D. Intramuscular

Which of the following statements regarding parenteral medications is correct? A. Parenteral medications are absorbed by the body through the digestive system B. Compared to enteral medications, parenteral medications have fewer side effects C. Tylenol is an example of a parenteral medication because it is taken orally D. Parenteral medications are absorbed more quickly than enteral medications

D. Parenteral medications are absorbed more quickly than enteral medications

Which of the following clinical signs would necessitate the administration of naloxone (Narcan)? A. Tachycardia B. Extreme agitation C. Hypertension D. Slow respirations

D. Slow respirations

The term "pharmacology" is MOST accurately defined as: A. The study of drug excretion from the human body B. The study of drugs that are produced illegally C. The study of how medications affect the brain D. The study of drugs and their actions on the body

D. The study of drugs and their actions on the body

An EMT may administer aspirin to a patient if: A. the patient is believed to be experiencing an acute stroke B. the patient is currently experiencing hypotension C. transport time to the hospital is greater than 30 minutes D. authorization from medical control has been obtained

D. authorization from medical control has been obtained

When treating an 80-year-old patient who is in shock, it is important to remember that: A. compensation from the respiratory system usually manifests with increased tidal volume. B. the older patient's central nervous system usually reacts more briskly to compensate for shock. C. medications older patients take for hypertension often cause an unusually fast heart rate. D. changes in gastric motility may delay gastric emptying, which increases the risk for vomiting.

D. changes in gastric motility may delay gastric emptying, which increases the risk for vomiting.

Capillary sphincters are: A. under complete control of the voluntary portion of the nervous system. B. capable of dilating in order to increase perfusion to crucial body organs. C. responsible for constricting to compensate for decreased cell perfusion. D. circular muscular walls that regulate blood flow through the capillaries.

D. circular muscular walls that regulate blood flow through the capillaries.

In an acute injury setting, neurogenic shock is commonly accompanied by: A. hypovolemia. B. tachycardia. C. diaphoresis. D. hypothermia.

D. hypothermia.

A 59-year-old male presents with severe vomiting and diarrhea of 3 days' duration. He is confused and diaphoretic, and his radial pulses are absent. His blood pressure is 78/50 mm Hg. After applying 100% supplemental oxygen, you should: A. perform a head-to-toe exam. B. allow him to drink plain water. C. obtain a repeat blood pressure in 5 minutes. D. prepare for immediate transport.

D. prepare for immediate transport.

Aspirin is beneficial to patients suspected of having a heart attack because it: A. causes direct coronary vasodilation B. reduces associated chest pain C. dissolves the coronary artery clot D. prevents the aggregation of platelets

D. prevents the aggregation of platelets

A 25-year-old unrestrained female struck the steering wheel with her chest when her car hit a tree while traveling at a high rate of speed. She has signs and symptoms of shock, which you suspect are the result of intrathoracic bleeding. Which of the following interventions will provide this patient with the greatest chance for survival? A. 100% oxygen administration B. full immobilization of her spine C. application of the pneumatic antishock garment (PASG) D. rapid transport to a trauma center

D. rapid transport to a trauma center

A 31-year-old female is experiencing an acute asthma attack. She is conscious and alert, but in obvious respiratory distress. After assisting her with her prescribed MDI, you should: A. administer another treatment in 30 seconds if she is still in distress B. check the drug's expiration date to ensure it is still current C. contact medical control and apprise him or her of what you did D. reassess the patient and document her response to the medication

D. reassess the patient and document her response to the medication

All of the following conditions would make you suspect shock, EXCEPT: A. anaphylaxis. B. heart attack. C. severe infection. D. tachycardia.

D. tachycardia.

Distributive shock occurs when: A. an injury causes restriction of the heart muscle and impairs its pumping function. B. severe bleeding causes tachycardia in order to distribute blood to the organs faster. C. temporary but severe vasodilation causes a decrease in blood supply to the brain. D. widespread dilation of the blood vessels causes blood to pool in the vascular beds.

D. widespread dilation of the blood vessels causes blood to pool in the vascular beds.

Your patient is a young adult female who was struck by a vehicle while she was riding her bicycle. She was not wearing a helmet. She is unresponsive and has a deformity with a deep depression to the temporal area of the head. You note no​ bleeding, and the scalp is intact. What is the greatest threat associated with this​ injury?

Damage to the brain

What is a primary purpose of the modern day EMS system?

Decrease the incidence of death and disability related to injury and illness.

Which signs and symptoms of shock will be observed​ last?

Decreased blood pressure

You have been called to a New​ Year's party for a male patient who was stabbed in the anterior chest. According to​ partygoers, the patient went outside with another male and was stabbed. Fifteen minutes​ later, he was found in the snow and 911 was called. When assessing this​ patient, what finding would you consider most serious and warrant immediate​ attention?

Decreased breath sounds to the left lung

Why does the incidence of diabetes mellitus increase with age?

Decreased physical activity, increased weight gain, and decreased insulin production

Which of the following is not a characteristic of epinephrine

Decreases heart rate and blood pressure

Which of the following would MOST likely occur as the direct result of the second collision in a motor vehicle crash?

Deformed steering wheel

A 40-year-old man is in cardiac arrest. Your partner is performing CPR. You are attaching the AED when the patient's wife tells you that he has an implanted pacemaker. The AED advises that a shock is indicated. What should you do?

Deliver the shock followed by immediate resumption of CPR.

The most significant factor that contributes to suicide is:

Depression

You have been asked to sit on your service's quality improvement (QI) committee. As a knowledgeable EMT, you recognize that in this role you will be involved in projects that are primarily designed to:

Develop ways in which the delivery of emergency care can be improved.

Signs of excited delirium include:

Diaphoresis, tachycardia, and hallucination

In addition to ensuring his or her own safety, the EMT's responsibility when caring for a patient with a behavioral emergency is to:

Diffuse and control the situation and safely transport the patient

If the abdomen appears larger than​ normal, this is​ called:

Distension.

The EMT gains the greatest protection from​ consent-related legal issues​ by: A. Notifying the emergency physician or medical direction of the​ patient's consent B. Asking the patient twice if it is okay to assess and treat him or her C. Having his or her partner serve as a witness to the consent D. Documenting the​ patient's consent for treatment on the prehospital care report

Documenting the​ patient's consent for treatment on the prehospital care report

Which one of the following best describes a topical hemostatic agent? a. Dressing especially designed to stop bleeding correct b. Rapid acting pill that promotes clotting in the body c. Topical medication that reduces chances of infection d. Liquid that stops bleeding by "gluing" the edges of a laceration together

Dressing especially designed to stop bleeding correct

What BEST describes a topical hemostatic​ agent?

Dressing specially designed to stop bleeding

cover the eye with a sterile dressing soaked in saline.

During an​ altercation, an intoxicated male was cut with a sharp knife. Assessment reveals a​ laceration, oozing dark red​ blood, across the right eyelid down to the right cheek. It also appears that the​ patient's eye was cut with the​ knife, reinforced by the complaint of eye pain and collection of blood in the sclera. The EMT​ would:

Which of the following components are needed to prove negligence?

Duty to act, breach of duty, injury/damages, and causation

For negligence to be​ proven, what must be​ present? A. Harm to the​ patient, false​ imprisonment, duty to act B. ​Assault, breach of​ duty, damage, and then abandonment C. Duty to​ act, breach of​ duty, battery, foreseeability D. Duty to​ act, breach of​ duty, injury, proximate cause

Duty to​ act, breach of​ duty, injury, proximate cause

A pregnant woman is choking inside a local restaurant. Who has the most compelling legal duty to​ act? A. EMTs from another county eating lunch after dropping off a patient at a local hospital B. An EMS supervisor who just came into the restaurant and is on a​ week's vacation C. An​ off-duty EMT who just finished eating and is headed to the bathroom D. A retired paramedic who just finished eating and is paying his bill

EMTs from another county eating lunch after dropping off a patient at a local hospital

Why might EMTs encounter a larger proportion of violent patients than the population at large?

EMTs respond to patients who, by definition, are having an emergency

What important piece of information has been omitted from the following report: "University Hospital, we are en route with a 61-year-old male complaining of generalized weakness. He states that he was cutting the grass when he suddenly became hot and weak, forcing him to sit down. He has a history of diabetes and prostate cancer, but takes no medications. Right now he is alert and oriented and has the following vital signs: pulse 96, respirations 18, and blood pressure 156/82 mmHg. We have placed him on two liters of oxygen by nasal cannula and are transporting in a semi-Fowler's position. He states that the weakness is starting to resolve. Do you have questions or orders?"

ETA to the hospital

Which statement about emergency medical services in the United States is true?

Each state has governmental control of its own EMS system, independent of the federal government

An Increase in fatty tissue, and weight gain begins to manifest in which age group?

Early Adult

________ pregnancy and spontaneous abortion are two conditions that can cause vaginal bleeding in women who do not appear to be pregnant and who may not realize they are pregnant.

Ectopic

Breathing is often more labor intensive in older adults because the:

Elasticity of the lungs decreases.

Following a stab wound to the left anterior chest, a 25 year old male presents with a decreased level of consciousness and signs of shock. Which of the following additional assessment findings should increase your index of suspicion for a cardiac tamponade?

Engorged jugular veins

The first step in assessing a patient with a behavioral emergency is to:

Ensure your safety

The highest priority for the EMT working on a patient when approaching the scene of a crime​ is: A. Administering quality patient care B. Ensuring personal safety C. Providing medical information to law enforcement D. Preserving potential evidence

Ensuring personal safety

Which of the following statements regarding electrical burns is correct?

Entrance wounds are small relative to the amount of internal tissue damage

What layer of the skin forms a watertight, protective seal for the body

Epidermis

You have a patient who was trapped beneath a truck along the freeway when he was trying to repair the brakes. By the time you​ arrive, the local fire department has already extricated the patient from underneath the​ truck, and he is currently immobilized. They advise you that the patient has a crushed​ pelvis, an open fracture to the left​ femur, a distended​ abdomen, and vomit in the airway. The patient suddenly became unresponsive when the weight of the truck was lifted from him. What should be your FIRST priority of​ management?

Establish an open airway.

What is the primary difference between ethics and​ morals? A. Ethics are what the EMT should​ do, whereas morals are what the EMT will do B. Morals define the expectations for professional​ occupations, whereas ethics are regulations written into state law defining what those expectations are C. Ethics is a branch of philosophy that studies​ morality, whereas morals are concepts of​ "right and​ wrong" D. Morals are what the EMT should​ do, whereas ethics are what the EMT will do

Ethics is a branch of philosophy that studies​ morality, whereas morals are concepts of​ "right and​ wrong"

While you are transporting a stable patient with a temporal hematoma from getting hit with a puck during a hockey​ game, he suddenly becomes unresponsive. You are still 20 minutes out from the hospital. At a​ minimum, how often should the EMT reassess the patient during the rest of the​ transport?

Every 5 minutes

You have arrived on the scene of a stabbing. As you approach the​ patient, you note that he has removed his shirt and has a single stab wound to the right lower quadrant of the abdomen. At the​ patient's side, your first action should be​ to:

Examine the​ patient's airway.

Activated charcoal is frequently suspended in sorbitol, a complex sugar that:

Facilitates moment through the digestive system

What best illustrates the​ EMT's failure to meet the standard of care when treating a patient with chest​ pain? A. Failing to notify the​ patient's primary care physician B. Failing to start an IV on the patient C. Failing to assist the patient in taking her heart palpitation medication D. Failing to give the patient oxygen

Failing to give the patient oxygen

A homeless patient is sick and is refusing transport. The patient is alert and clearly has the capacity to understand her situation and make a rational​ decision, but the temperature tonight will be near zero. The EMT picks up the patient and puts her on the cot. The patient continues to refuse to allow treatment and​ transport, despite the​ EMT's explanation that it will be warm in the hospital and she will be fed. The patient is transported. What could the EMT be charged​ with? A. Abandonment B. Assault C. False imprisonment D. Negligence

False imprisonment

On​ scene, you encounter a restless patient who has been stabbed in the right upper quadrant of the abdomen and complaining of abdominal pain and a dry mouth. Assessment reveals no immediate life threats to the airway or breathing. The radial pulse is rapid and weak and the skin cool and diaphoretic. Vital signs​ are: pulse​ 124, respirations​ 20, blood pressure​ 122/88, and SpO2​ 97%. The injury is isolated with no involvement of the​ head, neck, or back. In managing the scene and​ patient, what observation would cause you to immediately​ intervene?

Family giving the patient water to drink.

Your partner accidentally uses a profane word when talking to dispatch. Which agency has the power to fine him and the ambulance service?

Federal Communications Commission

What is the most common misconception surrounding mental illness?

Feeling "bad" or "depressed" means that you must be "sick"

Which of the following statements regarding fire ants is correct?

Fire ants often bite a person repeatedly.

Reduced tidal volume due to shifting of expiratory air from the uninjured side to the injured side is a common result of which​ injury?

Flail segment

A​ 21-year-old male has suffered a gunshot wound to the head. As you assess​ him, what finding would you recognize as a nonpurposeful response to​ pain?

Flexing his arms across his chest when you pinch his shoulder

Your legal right to function as an EMT is contingent​ upon: A. Avoiding civil liability B. Gaining paid employment by a public EMS system C. Completing an approved EMT program D. Following standing orders and​ protocols, as approved by medical direction

Following standing orders and​ protocols, as approved by medical direction

reddened membrane over the sclera.

For a patient suffering from​ conjunctivitis, the EMT would expect to observe​ a(n):

The _____ of a medication usually dictates the route by which it will be administered

Form

What GCS score would you document on your PCR​ if, after application of painful​ stimuli, the patient is able to localize pain and displays eye opening and incomprehensible sounds after the same painful​ stimuli?

GCS​ = 9

Which organ or organ system has the greatest tolerance for lack of perfusion (shock)? Select one: A. Gastrointestinal system B. Kidneys C. Brain D. Skeletal muscle

Gastrointestinal system

What medication form does oral glucose come in

Gel

You have arrived at the scene of a medical emergency. As you enter the room where the patient is located, you note him to be sitting up and talking without difficulty. Emergency Medical Responders are on scene and providing basic care for the patient. Which one of the following will you do first?

Get a report from the Emergency Medical Responder

Which of the following statements regarding glucose is correct

Glucose is given to patients who are suspected of being hyperglycemic

You arrive in the parking lot of a retail store and find a middle-age male on the ground, seemingly unresponsive with blood coming from a laceration on his head. Several shoppers have gathered around the patient and have covered him with a blanket. What should you do first?

Go to the patient and begin providing care

You have arrived at the emergency department with a patient complaining of nausea. The department is extremely busy and you are waiting to give a verbal report to the nurse or doctor. While​ waiting, dispatch contacts you over the radio and states that you are needed for a critically injured child that was hit by a car. Your best action would be​ to: A. Provide the certified​ nurse's aide with a report of the​ patient's condition B. Take the call and come back to give the report at a later time C. Go to the​ nurse's station and provide the registered nurse with a quick oral report D. Leave the patient in the emergency department and respond to the call

Go to the​ nurse's station and provide the registered nurse with a quick oral report

Painful urination associated with burning and a yellowish discharge is associated with:

Gonorrhea

In providing prehospital care to a patient with a​ soft-tissue injury, which statement is​ TRUE?

Hands must be washed even if gloves were worn during care.

A patient who has the capacity to make rational​ decisions, although he is​ dizzy, complains of a​ headache, and then​ vomits, declines your care and will not sign the refusal of care form. What is your best​ action? A. Have your partner sign on behalf of the patient B. Document that the patient refuses to sign and leave C. Inform the patient that by law he must sign the form D. Have a family member sign as a witness to the refusal

Have a family member sign as a witness to the refusal

You are transporting a 54-year-old male in respiratory arrest. An EMR is driving the ambulance as you and your partner are caring for the patient. Which of the following is the MOST logical way of notifying the hospital? A. Call the receiving hospital with your cell phone while providing patient care. B. Have the driver contact dispatch and relay the patient information to the hospital. C. Wait until you arrive at the hospital and then quickly apprise the staff of the situation. D. Request that a police officer respond to the hospital to apprise the staff of your arrival.

Have the driver contact dispatch and relay the patient information to the hospital.

You are treating a two-year-old girl who fell off her bike. She is crying and will not answer any of your questions. Which one of the following would be appropriate to find out where the girl hurts?

Have the girl's mother ask your questions for you

What should you suspect if you discern an altered mental status in a patient who wrecked his motorcycle and was not wearing a​ helmet?

Head injury

Which of the following is the most common cause of death from a blast injury?

Head trauma

You are transporting a female patient who was sexually assaulted by having a foreign body inserted deep into her vagina. You are attempting to control hemorrhage by the appropriate means. As you reassess the patient, which one of the following is of most concern? a. Pain increases from a 7/10 to 10/10. b. Patient informs you she has a sexually transmitted disease. c. Heart rate increases from 116 to 140 per minute. d. Large blood clots are forming at the vaginal entrance.

Heart rate increases from 116 to 140 per minute.

Your patient is confused and anxious after falling 20 feet from a ladder onto the grass below. What sign is most suggestive of​ shock?

Heart rate of 112

You are called for an​ 8-year-old boy who has fallen. On scene the​ boy's mother states that he was running while flying a kite and​ tripped, striking his face on a rock. Assessment reveals a large​ reddish-blue lump with intact skin under his left eye. When alerting the hospital emergency department of your​ arrival, you should inform them that the patient has what type of​ injury?

Hematoma

A patient has referred pain to her left shoulder. Which one of the following should the EMT suspect? Select one: a. Damage to the gall bladder b. Trauma to the right kidney c. Hemorrhage from the spleen d. Rupture of the small intestine

Hemorrhage from the spleen

A 70-year-old man presents with a severe nosebleed. His medical history includes COPD, depression, and a hemorrhagic stroke 3 years ago. His BP is 190/110 mm Hg, his pulse is 100 beats/min, and his respirations are 24 breaths/min. His medications include albuterol, sertraline (Zoloft), and multivitamins. Which of the following is MOST likely causing his nosebleed today? Select one: A. His prescribed albuterol B. Diabetic complications C. High blood pressure D. Hemorrhagic stroke

High blood pressure

You suspect that a 75-year-old man has internal injuries after he fell and struck his ribs and abdomen on the corner of a table. When assessing and treating an injured patient of this age, you must recall that:

His ability to physiologically compensate for his injury may be impaired due to an inability to increase cardiac output.

You receive a call for a domestic dispute. When you arrive at the scene, you find a young male standing on the front porch of his house. You notice that an adjacent window is broken. The patient has a large body, is clenching his fists, and is yelling obscenities at you. Which of the following findings is LEAST predictive of this patient's potential for violence?

His large body size

When taking a history on a patient experiencing a gynecologic emergency, you should consider asking all the following EXCEPT:

How many sexual partners have you had in the past?

The medical term for an extremely low blood glucose level is:

Hypoglycemia

A confused​ 62-year-old female fell at​ home, hitting the side of a table with her chest. Assessment reveals instability to the left lateral chest accompanied by minor bruising in the same area. She complains of intense pain every time she breathes. Her airway is​ patent, breathing is rapid and​ shallow, and skin warm and nondiaphoretic. Vital signs are pulse​ 112, respirations​ 24, and blood pressure​ 132/64 mmHg with SpO2 at​ 90%. Breath sounds are shallow but equal to both lungs. Based on those​ findings, you should treat this patient for what​ life-threatening condition?

Hypoxia

Which one of the following shows that the EMT is correctly assessing motor function in the arms of a patient with potential spine injuries? Select one: a. "I am going to move your arm; tell me if it hurts." b. "I am going to feel for a pulse in your wrist." c. "I need you to flex both arms across your chest." d. "Can you tell me what finger I am touching?"

I need you to flex both arms across your chest.

Which of the following patient responses would establish the "E" in the SAMPLE history?

I was mowing the lawn when the pain began."

In the public health continuum, what is the first responsibility that public health officials must satisfy to improve the health of a community?

Identify problems that affect the health of the population in question

In discussing an EMS response that involved potential​ negligence, your medical director asks you to determine whether the statute of limitations is active. As a knowledgeable​ EMT, you understand that you will need to determine which of​ these? A. If there is a proximate cause that can be linked to the bad outcome B. The exact action committed that is the basis for a negligence charge C. The amount of money that can be awarded to the patient and family D. If the allowed time for the patient to file a lawsuit has passed

If the allowed time for the patient to file a lawsuit has passed

A plaintiff is suing an EMT in civil court for alleged improper care and resultant injury. In this​ situation, which is​ true? A. The EMT is facing a very serious criminal charge B. If the suit is​ successful, the plaintiff will most likely be awarded money C. If found​ guilty, the EMT will face jail time or probation D. The government is the lead agency that is prosecuting the EMT

If the suit is​ successful, the plaintiff will most likely be awarded money

A pregnant​ 31-year-old has been shot in the neck.​ Currently, she is responsive to verbal​ stimuli, with an open airway and​ rapid, but adequate respirations. Her radial pulse is weak and fast. Vital signs are as​ follows: pulse​ 152, respirations​ 22, blood pressure​ 92/76, and SpO2 at​ 100% on​ high-concentration oxygen. When caring for this​ patient, what other intervention will most benefit this​ patient?

Immobilization and immediate transport

A young male patient was running through a park and struck his head on a​ low-hanging branch of a tree. Emergency Medical Responders have already immobilized the patient to a long board and are providing supplemental oxygen through a nonrebreather face mask. During​ transport, which assessment​ finding, obtained during your​ reassessment, would BEST indicate that the patient has suffered a​ concussion?

Improving memory

Which of the following statements regarding oxygenation and ventilation is correct?

In mines or confined places, where oxygen levels are low, ventilation may continue despite adequate oxygenation.

You are transporting a male patient who was stabbed in the left upper quadrant of the abdomen. On​ scene, the patient was alert and oriented with a patent airway and adequate respirations. There was minimal hemorrhage from the stab​ wound, which was covered with a sterile dressing. Oxygen was​ applied, and transport was initiated. During the​ reassessment, which of the findings is of MOST​ concern?

Increase in heart rate and unexplained restlessness

In late adults, the amount of air left in the lungs after expiration of the maximum amount of air:

Increases, which hampers diffusion of gases because of stagnant air that remains in the alveoli.

A​ 74-year-old female is complaining of abdominal pain after falling down five stairs and hitting her abdomen and left hand on a mailbox at the bottom. She also states pain to her left wrist and right​ ankle, both of which show obvious deformity. She is alert and​ oriented, and her abdomen is free of bruising but is tender to the left upper and lower quadrants. Her vital signs​ are: pulse​ 132, respirations​ 22, blood pressure​ 106/86, and SpO2 at​ 93%. Oxygen has been​ applied, and she is fully immobilized. Advanced life support has been requested and is 12 minutes away. What is your next​ action?

Initiate immediate transport.

A 14 year old boy has fallen through a plate glass window and is bleeding heavily from a laceration to his arm. His panicked mother has called 911 for help. Since the 911 operators are also emergency medical dispatchers, what sort of assistance will they be able to provide?

Instructions on how to control bleeding.

A mucosal atomizer device (MAD) is used to deliver certain medication via the

Intranasal route

Which of the following medications routes would be the most appropriate to use in an unconscious patient when intravenous access cannot be obtained

Intraosseous (IO)

Which of the following is the most rapidly acting medication administration route?

Intravenous(IV)

You are transporting a patient who was shocked by an electrical current while installing a residential electrical service panel. He has partial thickness burns to the hand and foot where the electricity entered and exited his body respectively. During the reassessment, which one of the following signs or symptoms would be of most concern to the EMT?

Irregular heartbeat

Which of the following statements regarding the pain associated with AMI is correct?

It can occur during exertion or when the patient is at rest.

How does CPAP improve oxygenation and ventilation in patients with certain respiratory problems?

It forces the alveoli open and pushes more oxygen across the alveolar membrane.

What BEST describes the purpose of determining a Glasgow Coma Scale​ (GCS) score in a patient with a head​ injury?

It helps determine whether a​ patient's mental status is improving or deteriorating.

How does positive-pressure ventilation affect cardiac output?

It increases intrathoracic pressure, which decreases venous return to the heart and causes a decrease in cardiac output.

Which of the following statements regarding a "dedicated line" is correct? A. It is a constantly open line of communication that cannot be accessed by outside users. B. It is a designated frequency on a portable radio that provides direct access to medical control. C. It is a frequency that is used exclusively by EMTs to communicate with one another in the field. D. It is a constantly open line of communication that is under exclusive control of a single user.

It is a constantly open line of communication that cannot be accessed by outside users.

Which of the following statements regarding stridor is MOST correct?

It is a high-pitched, crowing upper airway sound

You are assessing the abdomen of a​ 16-year-old female that was kicked by a horse earlier in the day. You notice a bluish discoloration around the belly button. What does this finding​ mean?

It is a late sign of intraabdominal bleeding.

What is the function of the left atrium?

It receives oxygenated blood from the lungs.

You have been called for an elderly female found by family members on the floor of her bathroom. She is unresponsive and has a bruise to the right side of her face. Her respirations are snoring and shallow. Her skin is cool and dry, and she is incontinent of urine. Her daughter tells you that the patient has a past medical history of stroke, COPD, and seizures. Which of the following would be most appropriate when managing this patient's airway?

Jaw-thrust maneuver

A​ 62-year-old male was the restrained driver of a car that was hit on the​ driver's side. Emergency Medical Responders have extricated and fully immobilized him with a cervical collar and long board. When​ asked, he complains of dizziness as well as leg and arm pain. Assessment reveals multiple contusions and deformity to his left forearm and a laceration with minor bleeding to the left side of his face. His airway is open and his breathing is labored. A rapid radial pulse is felt. Skin is cool and dry. What assessment finding would the EMT address​ first?

Labored breathing

A​ 49-year-old male was climbing on a truck at a construction site when he fell backward to the ground. He presents with a​ 2-inch linear wound to the top of his head. Bleeding has been controlled and the skull can be seen through the wound. How should you document this injury on the prehospital care​ report?

Laceration

____ impacts are commonly referred to as T-bone crashes

Lateral

You are called to a home and find a 56-year-old woman supine in her bed. She appears alert, but has slurred speech. Her family tells you she has a history of TIAs and hypertension. You ask the patient, "What day is it today?" Her reply is, "Butterfly." Which area of the brain is likely affected?

Left hemisphere

A patient had an unknown chemical splashed into her left eye. You have been irrigating the eye on scene and are now getting ready to transfer her to the stretcher. Knowing that you will need to continue irrigation throughout​ transport, how will you position this patient on the​ stretcher?

Left lateral recumbent

"A comprehensive regional resource capable of providing every aspect of trauma care from prevention through rehabilitation" is the definition of a _____ trauma center.

Level I

Which statement is true regarding linear skull​ fractures?

Linear skull fractures cannot typically be identified with palpation.

Which of the following abdominal organs would be characterized as being solid despite containing a large amount of​ blood?

Liver.

A​ 77-year-old female is​ unresponsive, but breathing and with a pulse. On​ scene, a family member hands you an official document stating that the patient does not want feeding​ tubes, ventilators, or other​ long-term life support equipment to keep her alive. There is no mention of whether to administer lifesaving drugs or withhold cardiopulmonary resuscitation. The EMT would recognize this document as​ a(n): A. ​"Do Not​ Resuscitate" (DNR) order B. Living will document C. Durable power of attorney D. ​"Involuntary Resuscitation" order

Living will document

A patient has been shot in the chest with a rifle. Assessment reveals a deteriorating level of consciousness and inadequate breathing. Positive pressure ventilation is being​ administered, and the entrance​ wound, located midclavicular at the second intercostal space on the left side of the​ chest, has been covered. Your next action would be​ to:

Look for an exit wound

Prior to your arrival on the scene of a motorcycle​ crash, an Emergency Medical Responder contacts you via radio and reports that there is one patient who has an avulsion to her left arm. As a knowledgeable​ EMT, you should anticipate which one of the​ following?

Loose flap of skin torn on her left arm

What is the most common presenting sign or symptom of PID?

Lower abdominal pain (and PID Shuffle)

Which of the following clinical presentations is MOST consistent with pelvic inflammatory disease (PID)?

Lower abdominal pain, fever, general malaise, and foul-smelling vaginal discharge

Which of the following organs or structures does NOT reside within the mediastinum?Select one: a. Trachea b. Venacavae c. Lungs d. Esophagu

Lungs

Which one of the following is an effective way to improve communication with most patients?

Maintain eye contact.

Which patient would the EMT recognize as suffering from an evisceration​ injury?

Male patient with a loop of intestine protruding from an open surgical wound

People at higher risk for suicide include all of the following, EXCEPT:

Married males older than 30 years

A drug is contraindicated for a patient when it

May cause harm or have no positive effect

According to the terminal drop hypothesis:

Mental function is presumed to decline in the 5 years preceding death.

An alert and oriented young male fell 5 feet from a​ stage, impacting a metal railing with the right side of his​ chest, just under his armpit. The primary assessment is negative for life​ threats, although he does complain of very painful breathing and has remarkable tenderness and crepitus over the 5th rib laterally in this area. Throughout​ care, what action is essential to​ perform?

Monitor breath sounds.

What would be a logical project to be undertaken and developed by a quality improvement (QI) committee?

Monthly continuing education programs in which seldom used skills are practiced.

When gathering a patients medications, you find the following: Isordil, Lasix,Mortion,and digoxin. Which of these medications can be obtained over the counter(OCT)

Motrin

Which of the following statements concerning trauma is true? a. A "spider web" or "star" pattern of cracks on the windshield means the patient impacted the windshield with his head. b. Multi-system trauma has a higher mortality rate than single-system injuries. c. Falls are the leading cause of trauma deaths. d. Mechanism of injury is the best predictor of patient outcome.

Multi-system trauma has a higher mortality rate than single-system injuries.

"For every action, there is an equal and opposite reaction" is:

Newton's third law

Which of the following statements regarding nitroglycerin is correct?

Nitroglycerin usually relieves anginal chest pain within 5 minutes.

Which of these actions constitutes a breach of the​ EMT's duty, placing him or her at risk for the charge of​ negligence? A. Obtaining a refusal of care from a patient without obtaining vital signs B. Applying oxygen to a patient who is not complaining of shortness of breath C. Transporting an alert and oriented patient who does not want transport to the hospital D. Obtaining an informed and signed refusal of care form from an alert and oriented patient with abdominal pain

Obtaining a refusal of care from a patient without obtaining vital signs

A patient who fell out of a tree has an open skull injury. Which findings would fit this​ condition?

Obvious deformity to the right side of the skull with a laceration to the overlying scalp

A 54-year-old male patient has been involved in a car crash. Window glass has caused an open wound to his upper neck. Which one of the following dressings is best for this injury? a. Sterile gauze b. Self-adhering roller gauze c. Occlusive dressing d. Moist sterile gauze

Occlusive dressing

A​ 54-year-old male patient has been involved in a car crash. Window glass has caused an open wound to his upper neck. Which one of the following dressings is best for this​ injury?

Occlusive dressing

An industrial worker has sustained a laceration to his abdomen. On closer​ inspection, you note what appears to be fat tissue and a portion of intestine protruding through the wound. Which one of the following is most appropriate for treating this​ injury?

Occlusive dressing applied over​ saline-soaked gauze.

If a patient with chest pain takes a daily aspirin, your medical director wants the EMT to contact an emergency department physician first, before administering additional aspirin. The act of contacting the emergency department physician by phone or radio for permission to administer additional aspirin is an example of:

On line medical direction

You are by the side of a confused patient who has diabetes and a low blood sugar level. You know that the patient needs orally administered sugar, so following your service's guidelines, you call the physician on duty at the hospital emergency department and request permission to administer it. This request is an example of:

On-line medical direction

Regarding an​ EMT's duty to​ act, the relationship between an EMT and a patient​ starts: A. Once the EMT begins to provide physical care B. Once the EMT makes physical contact with the patient C. Once the patient consents to be treated D. Once the EMT is dispatched to the call

Once the EMT makes physical contact with the patient

You are transporting a young and healthy female patient who was involved in a motor vehicle collision and struck her chest on the steering column. Although the primary assessment reveals no life​ threats, she does have redness to her​ sternum, as well as pain and tenderness. Breath sounds are clear and present bilaterally and vital signs are within normal limits. As you transport and reassess​ her, what sign or symptom would be most​ concerning?

Onset of an irregular heartbeat

An unhelmeted patient who was thrown from a motorcycle has a large scalp avulsion with obvious depression to the top of his head. Which injury do these findings​ suggest?

Open head injury

Your patient was shot in the right upper chest. Upon inspecting the​ wound, you notice that is bubbles each time the patient exhales. Which injury is MOST​ likely?

Open pneumothorax

A 22-year-old male with a history of clinical depression called 911 and stated that he has attempted to kill himself. Your unit and law enforcement officers arrive at the scene simultaneously. You find the patient lying supine on the living room floor. He is unresponsive and cyanotic. An empty bottle of hydromorphone (Dilaudid) is found on an adjacent table. You should:

Open the patient's airway and assess his respirations

Which of the following types of questions allow for the most detailed response? A. Multiple questions asked at once B. Yes or no questions C. Closed-ended questions D. Open-ended questions

Open-ended questions

Which of the following medication routes has the slowest rate of absorption

Oral

Which of the following statements regarding normal gas exchange in the lungs is correct?

Oxygen and carbon dioxide diffuse across the alveolar walls and capillaries

Which of the following statements regarding oxygen is correct?

Oxygen supports the combustion process and may cause a fire.

You believe that a patient who has been shot in the lower abdomen is bleeding internally and is in an early stage of shock. Which treatment shows appropriate prehospital care of this​ patient?

Oxygen therapy and rapid transport to the hospital

Which of the following statements regarding gonorrhea is correct?

Painful urination is a common symptom of gonorrhea in both men and women.

A​ 23-year-old female has been involved in a serious motor vehicle collision. Which assessment finding best indicates that she has a flail​ segment?

Paradoxical chest wall movement

A​ 46-year-old man who is conscious and has the capacity to make a rational decision is refusing treatment despite crushing chest pain and shortness of breath. He states that the hospital will not do anything for him and he does not want you to treat or transport him. The EMT should recognize that​ the: A. Police will need to be called so the patient can be transported in custody B. Patient will need to be transported against his will C. ​Patient's family can give consent to transport the patient D. Patient has a right to refuse treatment and transport

Patient has a right to refuse treatment and transport

Which patient would most benefit from calling an enhanced 911 system from the home phone?

Patient with sudden onset of confusion

Which of the following statements regarding breathing adequacy is correct?

Patients breathing shallowly may require assisted ventilation despite a normal respiratory rate.

Which of the following statements regarding hemophilia is correct? Select one: A. Patients with hemophilia may bleed spontaneously. B. Approximately 25% of the population has hemophilia. C. Hemophilia is defined as a total lack of platelets. D. Hemophiliacs take aspirin to enhance blood clotting.

Patients with hemophilia may bleed spontaneously.

What age bracket of patients is MOST likely to suffer injury to the liver and spleen from blunt abdominal​ trauma?

Pediatric.

In _____ administration, you are administering medication to yourself or your partner

Peer-assisted

Your patient is a​ 48-year-old male who is in shock from injuries sustained in a motor vehicle collision. Which assessment finding is an indication for the use of the pneumatic​ anti-shock garment​ (PASG)?

Pelvic instability with a BP of​ 78/48 mmHg

You have just arrived on the scene of a very serious motor-vehicle collision. Emergency Medical Responders (EMRs) have rapidly extricated an unresponsive female from the driver's seat. The EMRs report that she was unrestrained and struck the steering wheel with her chest and abdomen. As you start the primary assessment, you note that she has snoring respirations and is breathing shallowly at a rate of 24 breaths per minute. Which one of the following should you do next? a. Obtain her vital signs. b. Immobilize her to the backboard. c. Perform a jaw-thrust maneuver. d. Start positive pressure ventilation.

Perform a jaw-thrust maneuver.

A​ 40-year-old homeless male was found lying in the street. On physical​ examination, you notice that the patient has a stab wound to the​ mid-chest. He is​ unresponsive, and his skin is cool to the touch. His respirations are snoring at 40 per​ minute, and his pulse is 120. What care would you provide​ first?

Perform a​ jaw-thrust maneuver.

While cleaning a​ gun, a​ 44-year-old man accidentally shot himself in the abdomen. On​ arrival, the patient is responsive to painful stimuli and lying on his side with his legs drawn to his chest. Blood is evident on his shirt and pants. What is the correct sequence of events in caring for this​ patient?

Perform the primary​ assessment, administer supplemental​ oxygen, perform the secondary​ assessment, transfer to the​ stretcher, and provide rapid transport.

A female patient has been involved in a motorcycle crash. Your scene​ size-up reveals her to be unresponsive and lying in the roadway. It also appears that her left leg has been amputated at the knee. What should you do​ immediately?

Perform the​ jaw-thrust maneuver.

A​ 52-year-old male is unresponsive and has bruises and lacerations to the​ head, chest, and abdomen. Bystanders state that the patient was intoxicated and got into a fight with several patrons of a bar. They state that he was beaten with fists and not other objects or weapons. The patient has snoring​ respirations, shallow​ breathing, and a strong radial pulse. His skin is warm and dry. After applying manual​ in-line spinal​ stabilization, what should your next action​ be?

Perform the​ jaw-thrust maneuver.

Your patient was stabbed in the left anterior chest wall at the midclavicular line and fourth rib. His breath sounds are​ normal, but he is in severe distress and obvious shock. What do you suspect has happened to​ him?

Pericardial tamponade

Which of the following causes​ immediate, excruciating​ pain?

Peritonitis.

What mechanism of injury produces the conditions for traumatic​ asphyxia?

Person pinned between a truck and wall of a building

EMS providers must report which of these suspected events to law​ enforcement? A. Alcohol abuse and child abuse B. Drug addiction and​ drug-related crimes C. Physical and mental abuse D. Refusal of EMS services

Physical and mental abuse

The two basic categories of diagnosis that a physician will use for behavioral crises or psychiatric emergencies are _______________.

Physical and psychological

A functional disorder is a:

Physiological disorder in the absence of a structural abnormality

A young male was riding a motorcycle when he lost control and was ejected. The patient is unresponsive and receiving positive pressure ventilation. The rapid secondary assessment reveals bright blood flowing from both nostrils. Management of this bleeding would include which one of the following interventions? a. Careful place a nasopharyngeal airway. b. Lightly pinch the patient's nostrils shut. c. Gently place rolled sterile gauze into the nostrils. d. Place a loose dressing under the nose.

Place a loose dressing under the nose.

A​ 14-year-old male fell 10 feet from a retaining wall and hit his head on a metal post. He is responsive to verbal stimuli with incomprehensible speech. The secondary assessment indicates blood and fluid coming from inside his left ear. What treatment should you​ provide?

Place a piece of sterile gauze over the ear.

A 16-year-old male has been struck in the genitalia by a baseball. Assessment indicates a tremendous amount of edema as well as hematoma formation and bruising to his scrotum. The primary assessment reveals no life-threatening conditions; however, he is crying and rates the pain as a 10/10. Which one of the following is most appropriate for this patient? a. Application of warm packs to the scrotum b. Place cool compresses to the testicles c. Apply direct pressure to the site of injury d. Rapid transport using lights and sirens

Place cool compresses to the testicles

Shortly after assisting a 65 yet old female with her prescribed nitroglycerin, she begins complaining of dizziness and experiences a drop of 30 in her systolic blood pressure. The patient remains conscious and her breathing is adequate. You should:

Place her supine and elevate her legs

Which of the following is the MOST accurate guide to palpating a pulse?

Place the tips of your index and long fingers over the pulse point

What are noticeable characteristics of a 9-month-old infant?

Places objects in the mouth, pulls himself or herself up

A​ 16-year-old male has been struck in the genitalia by a baseball. Assessment indicates a tremendous amount of edema as well as hematoma formation and bruising to his scrotum. The primary assessment reveals no​ life-threatening conditions;​ however, he is crying and rates the pain as a​ 10/10. Which one of the following is most appropriate for this​ patient?

Placing cool compresses to the testicles.

Which of the following occurs after tissues are injured? Select one: A. Local blood vessels begin to dilate B. Red blood cells separate from plasma C. Platelets collect at the injury site D. Red blood cells become less sticky

Platelets collect at the injury site

Damage to tissues of what thoracic​ structure(s) will cause impairment of ventilation and a​ pneumothorax?

Pleural membranes

Which of the following is a metabolic cause of a seizure?

Poisoning

You are assessing a patient who had a previous head injury in which a portion of the cerebellum was destroyed. Which condition would relate to this specific​ injury?

Poor coordination when signing his name to the prehospital care report

Your assessment of a patient who was involved in a motor vehicle collision reveals a deformity to the left side of the head underneath the hair. On further​ inspection, you note that the skin overlying the deformity is still intact. On the basis of these assessment​ findings, which condition is your greatest​ concern?

Possible brain injury

By what mechanism is a person injured when he or she falls from a significant height?

Potential energy is converted to kinetic energy; the kinetic energy is then converted into the work of bringing the body to a stop.

Aspirin is beneficial to patients suspected of having a heat attack because it

Prevents the aggregation of platelets

Damage to the body that resulted from a pressure wave generated by an explosion is found in what type of blast:

Primary

Severe external bleeding should be controlled during what phase of the patient​ assessment?

Primary assessment

Which one of the following statements best describes the technique to be used when hyperventilating a​ head-injured patient with signs that the brain is​ herniating?

Provide 1 ventilation every 3 seconds.

An alert and oriented patient complaining of a severe headache is refusing all treatment as well as transport by the EMTs. After having signed the​ refusal, the patient suddenly seizes and becomes unresponsive prior to EMS providers leaving the home. The EMT​ should: A. Provide care under the guideline of implied consent B. Contact medical direction for permission to treat C. Place the patient into the​ family's car for transport to the hospital D. Honor the​ patient's wishes and leave the scene

Provide care under the guideline of implied consent

An intoxicated teenage male with a diabetic history fell down five stairs and is now responding to a shoulder pinch with garbled speech. Manual stabilization of the cervical spine is being maintained. The primary assessment reveals the airway is open, breathing is adequate, and the radial pulse is strong. His skin is warm and dry. Which one of the following should you do next?

Provide high concentration oxygen.

A young male patient has suffered a gunshot wound to the head. Realizing that there is little chance for​ survival, the EMT notes that the patient is a potential organ donor. The patient is unresponsive and breathing 4 times per minute on scene. The EMT initiates​ positive-pressure ventilation. En route to the​ hospital, the patient stops breathing and loses a pulse. Since the patient is a potential organ​ donor, the EMT​ should: A. Ventilate the patient but not provide chest compressions B. Stop any and all resuscitation measures since this is a trauma arrest C. Provide the same resuscitation as for a person who is not an organ donor D. Perform CPR but not use the automated external defibrillator

Provide the same resuscitation as for a person who is not an organ donor

An infant or small toddler would MOST likely gain trust in an individual who:

Provides an organized, routine environment.

While transferring a patient from her house to the ambulance on the wheeled​ cot, one of the EMTs slips on a patch of ice and falls. The stretcher​ overturns, and the patient suffers a broken wrist as a result. She files a lawsuit in civil court alleging negligence. For the​ patient, what will be the most difficult component of negligence to​ prove? A. Damage B. Proximate cause C. Duty to act D. Implied consent

Proximate cause

Which of the following terms applies to a state of delusion in which the patient is out of touch with reality?

Psychosis

Assessment findings of a patient ejected from a motorcycle indicate that he has a flail chest wall segment to his right anterior chest. He exhibits labored breathing and an SpO2 at​ 94%. Breath sounds are clear and equal bilaterally. The segment has been​ stabilized, and you are prepared to start positive pressure ventilation. Given these assessment​ findings, what type of injury underlying the flail segment is your primary​ concern?

Pulmonary contusion

Which term is used for bleeding that occurs in and around the​ alveoli, reducing oxygen​ exchange?

Pulmonary contusion

You are assessing a conscious 55-year-old male with a sudden change in behavior. Which of the following clinical findings would be MOST suggestive of dysfunction of this patient's central nervous system?

Rapid eye movement

A young male patient is​ self-extricated after hitting a telephone pole and rolling his car several times at a high rate of speed. He is spitting blood and has a large area of tenderness and instability to the left side of his chest. On scene you did not observe any paradoxical motion of the chest wall. The patient remains alert and oriented with stable vital​ signs, but continually complains of painful breathing. During​ transport, what is it essential that you​ do?

Reassess for paradoxical motion of the chest wall.

Immediately after physically restraining a violent patient, the EMT should:

Reassess the patient's airway and breathing

A 31 year old female is experiencing an acute asthma attack. She is conscious and alert, but in obvious respiratory distress. After assisting her with her prescribed MDI, you should

Reassess the patients and document her response to the medication

You are pulling into the ambulance bay at a local hospital. On​ board, you have a patient with behavioral problems who continually seeks treatment at this particular hospital. Before the patient can be​ unloaded, the physician comes out and states that the patient must be taken to another hospital. As an​ EMT, you​ should: A. Recognize a violation of the EMTALA statute B. Take the patient to the facility specified by the doctor C. Ask the patient which hospital he wants to be taken to D. Call the supervisor to report a HIPAA violation

Recognize a violation of the EMTALA statute

Which portion of the blood carries oxygen to and wastes away from body tissues? Select one: A. Plasma B. Red blood cells C. White blood cells D. Platelets

Red blood cells

A technique used to gain insight into a patient's thinking, which involves repeating in question form what the patient has said, is called:

Reflective listening

When given to patients with cardiac related chest pain nitroglycerin

Relaxes the walls of the coronary arteries

A patient with a history of schizophrenia called EMS, because he was experiencing abdominal pain. When law enforcement arrived, the patient became violent, necessitating the placement of handcuffs. When you assess the patient, he tells you that killing someone will make his abdominal pain go away. His vital signs are stable. How should you manage this situation?

Request a police officer to accompany you in the ambulance

You suspect a trauma patient to be suffering from a hemothorax to the left lung. What assessment finding would reinforce this​ suspicion?

Respiratory distress and the signs and symptoms of shock

A 38-year-old male with a history of schizophrenia is reported by neighbors to be screaming and throwing things in his house. You are familiar with the patient and have cared for him in the past for unrelated problems. Law enforcement officers escort you into the residence when you arrive. The patient tells you that he sees vampires and is attempting to ward them off by screaming and throwing things at them. He has several large lacerations to his forearms that are actively bleeding. The MOST appropriate way to mange this situation is to:

Restrain the patient with appropriate force in order to treat his injuries

Which section of the heart receives deoxygenated blood? Select one: A. Atria B. Ventricles C. Left D. Right

Right

While running in a parking​ lot, a young boy tripped and fell. He hit his face and mouth on a​ curb, knocking one of his top front teeth from its socket. Bleeding has been controlled and the tooth found. What instructions would you give your partner when caring for the​ tooth?

Rinse the tooth first then put it in a cup of saline for​ transport.

A man has been bitten in his arm by his dog. He states that the bite occurred several hours earlier when he accidentally stepped on the​ dog's paw. When​ asked, he tells you that the dog is​ up-to-date on all of her​ shots, including the rabies vaccine. Assessment reveals two small puncture wounds to the hand with some bruising in the surrounding tissue. The patient wants to refuse treatment and transport. Given these assessment​ findings, which one of the following indicates the primary reason the patient should be seen in the emergency​ department?

Risk for infection

Which of the following is considered a type of motor vehicle collision?

Rollover

Your service's medical director would like to explore the idea of EMTs using special transport ventilators when caring for patients who require assistance with their breathing. When using an evidence based medicine approach, what should you do first?

Search the medical literature for research related to this topic.

It has been some time since you were involved in the care of a pediatric patient. You decide to review the assessment and treatment basics that your medical director requires. To review this information, which resource should you consult first?

Service protocols.

A woman has fallen from a​ second-story window onto the concrete sidewalk below. She is unresponsive and has a large depression to the back and top of her skull. Additional findings include abdominal bruising and an angulated left ankle. Your partner reports that the​ patient's vital signs are pulse​ 68, respirations 14 and​ irregular, blood pressure​ 198/110 mmHg, and SpO2 91 percent. On the basis of these assessment​ findings, what should you​ recognize?

Severe head injury with increasing pressure within the skull

The EMT recognizes that the Health Insurance Portability and Accountability Act​ (HIPAA) limits the​ EMT's ability​ to: A. Bill for services provided by the Emergency Medical Services system B. Obtain refusals of care from patients who do not have medical insurance C. Transport patients to the hospitals of their choice D. Share​ patient-specific medical information with others

Share​ patient-specific medical information with others

In contrast to secure attachment, anxious-avoidant attachment occurs when a child:

Shows little emotional response to a parent or caregiver following repeated rejection.

The risk of bleeding in the skull, which increases with age, is MOST directly related to:

Shrinkage of the brain.

After taking Benadryl for an allergic reaction a person begins experiencing drowsiness and a dry mouth. These findings are an example of an

Side effect

Which of the following organs can tolerate inadequate perfusion for 2 to 3 hours? Select one: A. Brain B. Heart C. Skeletal muscle D. Kidneys

Skeletal muscle

When performing the primary​ assessment, which of the following signs or symptoms best suggests that the patient is in​ shock?

Skin that is cool and diaphoretic

A local politician has been involved in a motor vehicle collision. Witnesses state that they saw this man leave a​ bar, and repeatedly stumbled until he got to his vehicle.​ Then, just two miles down the​ road, he struck a utility pole head on. On scene and throughout​ transport, the patient is confused and combative. When you are leaving the​ hospital, a reporter asks you what happened. You state that it looks as though the politician may be​ drunk, but you are not sure. Your statement is printed in the newspaper.​ Later, it is determined that the man was not​ drunk, but rather has diabetes and had low blood sugar at the time of the accident. Which charge could the politician levy against​ you? A. Litigation B. Assault C. Battery D. Slander

Slander

You are dispatched to a call for a 4-month-old infant with respiratory distress. While you prepare to take care of this child, you must remember that:

Small infants are nose breathers and require clear nasal passages at all times.

The​ liver, spleen, and pancreas are examples of what type of abdominal​ organs?

Solid.

You are treating an 82-year-old female who is having trouble hearing some of your questions. What technique would be best in helping you get her past medical history?

Speak clearly and slowly.

When you are assessing the abdomen of a​ 25-year-old male you note that he has a large piece of glass sticking out of his right upper quadrant. What should you do about​ this?

Stabilize it and administer oxygen.

An object impaled in the abdomen should​ be:

Stabilized in place.

Before the EMT contacts medical direction for authorization to assist a patient with shortness of breath in using her metered dose inhaler, protocols state that the EMT must first attempt to provide relief by administering oxygen if the pulse oximeter reading is less than 94%. You recognize that the administration of oxygen is best described as a (n):

Standing order

A patient with a terminal disease is​ unresponsive, not​ breathing, and without a pulse. The family states that the patient has a​ "Do Not​ Resuscitate" (DNR) order signed by her medical​ doctor, but they cannot find the current one. All they can find is a previous DNR that expired 8 months ago. The EMT​ should: A. Wait for family to locate the DNR B. Start cardiopulmonary resuscitation C. Have the family sign a refusal of care form D. Pronounce the patient deceased

Start cardiopulmonary resuscitation

A​ 61-year-old male has fallen off a roof. Your primary assessment findings include​ unresponsiveness, agonal breathing and a slow and weak radial pulse. His skin is cool and dry. Emergency Medical Responders are maintaining manual​ in-line spinal stabilization. Which action would you perform​ immediately?

Start positive pressure ventilation.

You have moved to another state and wish to work as an EMT. In your previous state of​ employment, EMTs were allowed to administer a specific set of drugs. To determine whether EMTs can administer drugs in your new state of​ residence, you should review​ the: A. National EMS Scope of Practice Model B. National EMS Education Standards C. State Emergency Medical Services Act D. State Emergency Medical​ Technicians' Scope of Practice

State Emergency Medical​ Technicians' Scope of Practice

A​ 22-year-old female has been sexually assaulted and taken to the emergency department. With regard to the release of confidential medical​ information, which individual would most likely be allowed to receive information related to the​ patient's injuries without her​ consent? A. ​Patient's mother B. State police C. ​Patient's boyfriend D. Crisis counselor

State police

You are attending to a 27-year-old patient with a history of depression. The patient's family tells you that she has been openly talking about harming herself and suicide, and they got scared when she tried to overdose on some medications. The patient did not take the pills and is alert and oriented. Despite all of your best efforts to convince her, the patient refuses to go to the hospital for treatment. Based on this information, you should:

Stay with the patient while you arrange for other transport options as a potentially life-threatening emergency exists

You have been called by family members for their mother who is​ "not acting​ right." On scene the family informs you that they are concerned because their​ 68-year-old mother has been complaining of a headache for two days and is now very confused. When asking about a bruise on the right side of her​ forehead, the family states she fell in church a week ago and hit her head. Since all other aspects of the assessment are​ unremarkable, you suspect a head injury. What condition would you suspect given the findings and​ history?

Subdural hematoma

How is nitroglycerin usually given by the emt

Sublingually

Which of the following statements regarding anaphylactic shock is correct?

Subsequent exposure after sensitization often produces a more severe reaction.

A​ 66-year-old female patient has been struck by a car. Your assessment reveals gurgling​ respirations, rapid​ breathing, and​ cool, diaphoretic skin. You also observe bruising to the chest and abdomen. What should you do​ immediately?

Suction the airway.

A construction worker has been shot with a nail​ gun, resulting in a long nail going through his cheek and firmly embedding into the lower gum and jaw. There is considerable blood in his mouth and he is in excruciating pain. Given this​ scenario, which of the following would be your first​ priority?

Suctioning the airway

When caring for a patient experiencing excited delirium, the EMT should remember that:

Sudden death can occur if the patient's violence is not controlled

What will be the MOST likely position of transport for a patient with a traumatic brain injury who is currently hypotensive and being ventilated by the​ EMT?

Supine

Activated charcoal is an example of

Suspension

Common signs and symptoms of acute hyperventilation syndrome include:

Tachypnea an tingling in the extremities.

You arrive on the scene of a motor-vehicle collision. Walking toward you is the unrestrained driver of the vehicle that sustained moderate front-end damage. The patient complains of some back pain, but was walking around after the crash and does not have any neurological deficits. What is your initial action for managing this patient?

Take manual in-line spinal stabilization.

A​ 41-year-old male patient has been struck in the head with a metal pipe. As you approach the​ patient, you note that he is combative and has blood on the left side of his face and head and on his shirt. His breathing appears to be​ labored, and he is incontinent of urine. Which action would you perform​ immediately?

Take manual​ in-line spinal stabilization.

You are called for a​ 78-year-old female who fell earlier in the day. As you enter her​ kitchen, you observe her sitting in a chair with ecchymotic areas to her left cheek and left arm. There also appears to be blood on her pants. She is on oxygen and is breathing in a moderately labored manner. She states that she fell a few hours ago and now the pain and swelling in her arm are unbearable. Which one of the following would you do​ first?

Take manual​ in-line spinal stabilization.

In which thoracic injury is a mechanical collapse of the vena cava​ involved?

Tension pneumothorax

Burns from hot gases and respiratory injuries from inhaling toxic gas are associated with which type of blast injury?

Tertiary or Miscellaneous

Which of the following statements regarding the AED and defibrillation is correct?

The AED will not analyze the rhythm of a moving patient.

You have been asked to review a legal case in which a patient intentionally overdosed on heroin and then died after EMS responders arrived. Those responders did not have a​ bag-valve mask to ventilate the nonbreathing​ patient, so the patient was not ventilated until a second EMS unit arrived. The evidence indicates that the EMT and paramedics both signed their morning equipment check indicating that they had inspected the ambulance and had all the necessary equipment to do their job. In this​ case, you would​ realize: A. The EMT and paramedics can seek protection under the Good Samaritan law B. The EMT and paramedics will most likely be charged with contributory negligence C. The EMT and paramedics are not responsible because the patient intentionally overdosed D. The EMT and paramedics were negligent in caring for the patient

The EMT and paramedics were negligent in caring for the patient

A​ patient's spouse, who is a​ doctor, states that an EMT is negligent because he put his wife on​ low-concentration oxygen when she should have gotten​ high-concentration oxygen. The patient was having chest pain that was later determined to be caused by anxiety. She was released from the emergency department later in the day. Which statement is​ true? A. Negligence cannot be proven since the EMT responded to the call and did render care B. Negligence can be proven because the protocol states​ high-concentration oxygen should be given C. Negligence is a distinct possibility since the patient was transported and treated in the emergency department D. The EMT cannot be proven negligent because that patient suffered no harm or damage

The EMT cannot be proven negligent because that patient suffered no harm or damage

While off​ duty, an EMT happens upon the scene of a motor vehicle collision in which a car rolled over the side of an embankment. A young man has been seriously​ injured; he appears​ unresponsive, and is bleeding from the head and face. The EMT is alone with his infant baby daughter in the car and cannot leave her to provide help.​ Therefore, the EMT alerts 911 of the emergency and remains in his car with his daughter until EMS arrives. Later in the​ week, he finds out that the patient died. Which statement is true regarding this​ incident? A. The EMT violated the state standard of care by not providing some form of care B. The EMT must go to court and explain that he did not help because of his daughter C. The EMT had a legal obligation to help and can be held liable for the death D. The EMT had no duty to act and cannot be held liable for the​ man's death

The EMT had no duty to act and cannot be held liable for the​ man's death

A​ 62-year-old male is short of breath. The EMT asks the patient if it would be okay to assess​ him, including taking his vital signs. The patient agrees by nodding the head​ "yes." As​ such, which is​ true? A. The patient can be transported without further permission B. The EMT has obtained expressed consent C. The EMT has gained implied consent D. Informed consent has been obtained by the EMT

The EMT has obtained expressed consent

Which of the following statements regarding the mechanism of injury (MOI) is correct?

The MOI may allow you to predict the severity of a patient's injuries

Which of the following explains why the basilar skull is commonly the site of fractures due to​ trauma?

The area is permeated with openings.

A young boy was riding his bicycle down the street when he hit a parked car. What was the second collision?

The bike rider hitting his bike or the car

With increasing age, the heart must work harder to move the blood effectively because:

The blood vessels become stiff.

In the human​ body, what occurs when a person is in​ shock?

The cells are not getting enough​ oxygen, and waste products are accumulating.

An intoxicated patient has suffered a burn to his left lateral thigh after passing out with his leg touching the side of a kerosene heater. Close examination of the burn reveals tough leathery tissue in the center of the burn, with red skin with blisters surrounding it. Regarding the burn, what would the EMT agree to be true?

The central portion of the burn is a full-thickness burn and the surrounding portion is a partial-thickness burn

Which of the following is NOT considered appropriate use of air medical services?

The closest trauma center is 10 minutes away by ground transport

Which of the following statements regarding the dermis is correct?

The dermis contains hair follicles, sweat glands, and nerve endings

Which of the following statements regarding the epinephrine auto injector is correct?

The epinephrine auto injector delivers a preset amount of the drug

Which of the following body systems or components is the LEAST critical for supplying and maintaining adequate blood flow to the body? Select one: A. Adequate blood in the vasculature B. An intact system of blood vessels C. The filtering of blood cells in the spleen D. An effectively pumping heart

The filtering of blood cells in the spleen

focus light onto the retina.

The function of the ocular lens is​ to:

You are assessing the pupils of a patient who hit his head after falling from the top of a tractor trailer. What pupillary finding suggests a closed head​ injury?

The left pupil constricts to light but the right pupil does not.

A male EMT was discussing a​ patient's medical condition with a female EMT from a different service. The woman then repeated this information to​ others, which eventually made its way back to the patient. Regarding issues of​ confidentiality, which of the statements would be is​ true? A. The female EMT committed an EMTALA violation B. The male EMT could be changed with slander C. The male EMT violated the HIPAA law D. Both EMTs violated EMTALA regulations

The male EMT violated the HIPAA law

What is the Moro reflex?

The neonate opens his or her arms wide, spreads his or her fingers, and seems to grasp at something after being startled.

An infant or small child's airway can be occluded if it is overextended or overflexed because:

The occiput is proportionately large and the trachea is flexible.

Which of the following statements made by an EMT indicates an understanding of facial injuries?

The only movable bone making up the face is the​ mandible

What action best indicates that informed consent has been​ obtained? A. The patient agrees to oxygen therapy after being told it may help decrease his chest pain but may dry his​ nose, causing discomfort B. The EMT tells the patient that he will need oxygen because he is having chest​ pain, which could be a sign of a heart attack C. The EMT administers oxygen because the patient is having chest pain and appears short of breath D. The patient is given oxygen because he is having chest pain and was the one who placed the 911 call for help

The patient agrees to oxygen therapy after being told it may help decrease his chest pain but may dry his​ nose, causing discomfort

Despite a​ patient's protest that he does not want his blood pressure​ taken, the EMT places a BP cuff on his arm and takes his blood pressure. The EMT​ states: "See? It is just as I suspecteddash-your blood pressure is sky high. You really need to be seen in the emergency​ department." The patient states he did not realize that his blood pressure was that high. In terms of​ battery, which statement is​ true? A. The patient could charge the EMT with​ assault, not battery B. Since the BP was elevated and treatment must be​ provided, battery cannot be charged C. Because the patient knows the result of the​ BP, the EMT is safe from battery D. The patient could charge the EMT with​ battery, regardless of his blood pressure reading

The patient could charge the EMT with​ battery, regardless of his blood pressure reading

Another crew has called for your assistance in extricating an obese male with a nosebleed from a​ third-floor bedroom. As you enter the​ room, which observation indicates proper management of the​ patient's condition?

The patient is sitting​ upright, leaning​ forward, with nostrils pinched shut and an ice pack on the nose.

You have extricated a male patient who was entrapped in a rollover car wreck. During your immediate​ on-scene management, you removed the​ patient's clothing and started CPR. During this​ time, a police officer retrieved the​ driver's license from the​ patient's pants and advises you that he is an organ donor. Given this​ information, which statement is​ true? A. The patient should be hyperventilated and hyperoxygenated along with receiving CPR to increase the longevity of his organs B. The patient should be treated in the same way en route to the hospital as a nondonor C. The EMTs should initiate cooling measures to increase the longevity of the​ patient's organs D. The patient cannot be an organ donor now since he arrested prior to reaching the hospital

The patient should be treated in the same way en route to the hospital as a nondonor

Which of the following findings would be the MOST significant when assessing a patient with possible internal bleeding? Select one: A. The patient has not eaten in 24 hours. B. The patient has a history of hypertension. C. The patient had a stroke 5 years prior. D. The patient takes rivaroxaban (Xeralto).

The patient takes rivaroxaban (Xeralto).

You work in a community that has an "enhanced" 911 system. Given this, which is true about such a system?

The physical location of the landline used to make the 911 call is displayed to the call taker.

Which finding is considered normal in assessing a patient who is suffering from a head​ injury?

The right pupil constricts when light is shined into it.

What is the primary difference between the​ EMT's scope of practice and the​ EMT's standard of​ care? A. The scope of practice is decided by the medical​ director, while the standard of care is defined by state regulations B. The scope includes which actions the EMT can legally​ take, while the standard of care is what a EMT should do in an EMS system with similar training and protocols C. The scope is what the EMT should​ do; the standard is what the EMT is legally bound to do D. There is no clinically relevant difference between the scope of practice and the standard of​ care, as both define what the EMT should do in the exact same situation

The scope includes which actions the EMT can legally​ take, while the standard of care is what a EMT should do in an EMS system with similar training and protocols

Which of the following statements regarding the secondary assessment is correct?

The secondary assessment should focus on a certain area or region of the body as determined by the chief complaint.

Which observations direct the EMT to classify a​ patient's soft tissue injury as an​ abrasion?

The skin is scraped and​ red; blood oozing from the injury site

The term pharmacology is most accurately defined as

The study of drugs and their actions on the body

What occurs when the diaphragm and intercostal muscles​ contract?

The thoracic cage​ enlarges, causing a decrease in intrathoracic and intrapulmonary pressures.

44. Which one of the following statements best characterizes the reason for wearing high-visibility safety vest at the scene of a motor vehicle collision?

The use of a vest can increase your visibility to passing drivers at the scene of an incident.

Most authorities agree that the modern-day EMS system evolved after the release of which document?

The white paper "Accidental Death and Disability: The Neglected Disease of Modern Society": in 1966

Infants are often referred to as "belly breathers" because:

Their rib cage is less rigid and the ribs sit horizontally.

Which of the following statements regarding HIV is correct? A. There is no vaccine against HIV infection. B. HIV is easily transmittable in the EMS field. C. HIV is far more contagious than hepatitis B. D. HIV is transmitted exclusively via blood.

There is no vaccine against HIV infection.

Why do middle adults commonly experience financial concerns?

They are preparing for retirement but must still manage everyday financial demands.

A paramedic was late in reregistering and​ now, according to the state EMS​ agency, must function as an EMT until all of the reregistration requirements are satisfied. As an​ EMT, she is on the scene of a motor vehicle accident and taking care of a critically injured teenager. While waiting for a paramedic to​ arrive, she starts an​ IV, but does not hang the IV fluid that will be used to help stabilize the patient. This is immediately done by a paramedic once he arrives. As​ such, which of these is​ true? A. The​ paramedic/EMT violated the scope of practice and can be held accountable even though the patient survived B. The​ paramedic/EMT did not violated the scope of practice since the patient was critically injured and required lifesaving care C. The​ paramedic/EMT violated the scope of practice but cannot be held accountable since the patient was stabilized by her actions D. The​ paramedic/EMT did not violate the scope of practice since she is experienced with IV therapy

The​ paramedic/EMT violated the scope of practice and can be held accountable even though the patient survived

A patient involved in a motor-vehicle collision has suffered the separation of a rib from a spinal vertebrae. Based on the anatomy of the spine, where has this injury occurred? a. Middle spine b. Lumbar spine c. Thoracic spine d. Cervical spine

Thoracic Spine

A patient, who is coughing, has sustained a superficial burn to the chest, arms, and face after pouring gasoline on a smoldering campfire. Which one of the following statements indicates that the EMT is properly caring for him?

Throughout transport, I am going to listen to your lungs quite frequently

Which one of the following is a function of providing the physician or nurse with an oral report when transferring care to the hospital emergency department (ED)?

To summarize patient assessment and treatment information

You have been called to a local emergency department to transfer a​ 67-year-old male to another facility. At the​ hospital, the emergency physician informs you that the patient and his family have requested the transfer because his doctor is affiliated with the other hospital. At the​ patient's bedside, you see that the patient is receiving an antibiotic through an IV. Although your state does allow EMTs to perform nonemergent transports of patients receiving normal saline through an​ IV, it does not allow transport of patients receiving IV medications. When informed of this​ restriction, the physician states that the antibiotic will be done in 10 minutes and that he will give written permission to start the transport while the medication is infusing. Which EMT action would be most appropriate at this​ time? A. Stop the IV infusion and transport the patient B. Recognize a HIPAA violation and refuse the transport C. Transfer the patient once all the medication is infused D. Follow the​ doctor's order as long as he writes and signs it

Transfer the patient once all the medication is infused

What actions place the patient at greatest risk for a medical mistake?

Transferring care from one EMT to another.

Victims of inhaled poisoning will require which of the following?

Transport to an emergency department for evaluation

In a community park near his​ home, a​ 10-year-old boy has fallen from the top of a slide. His right wrist is deformed and he is in pain. He states that he lives with his mother and she is working. He does not know her work number. The​ EMT's best action would be​ to: A. Obtain permission from the park director B. Do not treat the patient until his mother can be contacted C. Treat the patient under the guidelines of implied consent D. Have the police take custody of the boy and give permission to treat

Treat the patient under the guidelines of implied consent

Which action best illustrates an EMT upholding a high ethical​ standard? A. Providing care that goes beyond the scope of practice B. Informing other health care providers of patients in the community with AIDS C. Administering oxygen to patients who are short of breath D. Treating alcoholics and drug addicts with respect

Treating alcoholics and drug addicts with respect

Which of the following is an example of a brand(trade). Name of a drug

Tylenol

A 21 year old female has been struck in the left eye by a softball. Which of the following assessment findings is most concerning for an eye injury?

Unusual sensitivity to light.

In managing a patient with a​ soft-tissue injury, it is essential that the EMT perform which​ action?

Use the appropriate personal protective equipment.

Injuries to the genitalia​ are:

Usually embarrassing for the patient.

_________can cause significant blood loss and lead to hypovolemia.

Vaginal bleeding

Which​ organ, if seriously​ injured, would cause the most rapid​ death?

Vena cava.

Which of the following will help improve radio communications? A. Answer questions with "yes" or "no." B. Use codes to speed communication. C. Hold the radio at least 6 inches from your mouth. D. Wait 1 second after pressing the transmit button before speaking.

Wait 1 second after pressing the transmit button before speaking.

You are attending to a 32-year-old male patient. The patient's wife tells you that he returned from Afghanistan last year. While he initially seemed fine, lately he has become withdrawn and distanced himself from his family and friends. He does not talk about it, but she knows that he has been having terrible nightmares that wake him up. The most appropriate question to ask regarding his experience in Afghanistan is:

Were you shot at or under fire?

"We will need to cover both eyes with patches before we transport​ him."

What instructions would you provide to the Emergency Medical Responders when treating a patient who was punched in the eye and has blood visible in the anterior​ chamber?

Which of the following questions is used to determine a patient's chief complaint?

What seems to be the matter?

A​ state's Good Samaritan law would NOT protect the health care​ provider(s) in which of these​ situations? A. When an EMT is working for a volunteer EMS system and performs an act of gross negligence B. When a nurse or physician who is off duty stops to assist at an EMS call where paid EMTs are​ present, but the patient still dies C. When an EMT is off​ duty, but stops to assist at a motor vehicle collision and functions in good faith D. When the EMT stops to help a collapsed person at a park while off duty by providing​ CPR, but the patient still dies

When an EMT is working for a volunteer EMS system and performs an act of gross negligence

Sclera

When assessing a male patient who was punched in the face by another​ man, you detect blood in the white part of his eye. As​ such, you would recognize injury to what optical​ structure?

Remove if the eye has been injured by a chemical burn.

Which of the following is a general guideline to use when determining whether or not to remove a​ patient's contact​ lenses?

"The only movable bone making up the face is the​ mandible."

Which of the following statements made by an EMT indicates an understanding of facial anatomy and​ injuries?

"Rinse the tooth first then put it in a cup of saline for​ transport."

While running in a parking​ lot, a young boy tripped and fell. He hit his face and mouth on a​ curb, knocking one of his top front teeth from its socket. Bleeding has been controlled and the tooth found. What instructions would you give your partner when caring for the​ tooth?

establish manual​ in-line spinal stabilization.

While working to build a​ house, a construction worker was hit in the head by a​ 2" x​ 4" plank that slid from the roof and fell to the ground below. The patient has a sizable laceration to the left side of his face and is spitting blood. He also lost several​ teeth, two of which he is holding. Your initial action when caring for this patient would be​ to:

Which of the following questions would be LEAST pertinent during the initial questioning of a patient who ingested a substance?

Why was the substance ingested?

Which of the following statements regarding positive-pressure ventilation is correct?

With positive-pressure ventilation, more volume is required to have the same effects as normal breathing.

During​ orientation, an EMT is told that owing to the nature of her​ employment, she is covered by sovereign immunity. Based on this​ statement, the EMT​ must: A. Work for a public EMS agency B. Provide services for a volunteer EMS agency C. Be immune from claims of negligence D. Work for a​ nonprofit, privately owned hospital

Work for a public EMS agency

You are an​ off-duty EMT and have come across a motor vehicle collision. A car has crashed​ head-on into a utility pole. There is moderate damage to the vehicle and the driver is walking about the scene. You stop and assess the patient. He has a large bruise on his arm and is complaining of abdominal pain where the seat belt contacted his body. Emergency Medical Responders​ (EMRs) are on​ scene, and a responding ambulance has a​ 2-minute ETA. At this​ point, you realize​ that: A. You can leave the scene since the patient is stable and EMS is 2 minutes away B. You can only transfer care to a paramedic once he or she arrives on scene C. You cannot leave until another EMT or paramedic arrives to assume care D. You can transfer care to EMRs since EMS has been notified and an ambulance is 2 minutes from the scene

You cannot leave until another EMT or paramedic arrives to assume care

"Are you wearing contact​ lenses?"

You have arrived on the scene of an industrial plant where a female worker had a chemical splashed into her eye. She is complaining of severe pain in the eye and is flushing it with tap water over a sink. Which of the following questions is it most important that the EMT ask​ first?

Jaw-thrust maneuver

You have been called for an elderly female found by family members on the floor of her bathroom. She is unresponsive and has a bruise to the right side of her face. Her respirations are snoring and shallow. Her skin is cool and​ dry, and she is incontinent of urine. Her daughter tells you that the patient has a past medical history of​ stroke, COPD, and seizures. Which of the following would be most appropriate when managing this​ patient's airway?

hold the​ patient's eye open and begin flushing with large amounts of tap water and continue throughout transport.

You have been called to a high school science lab where a male student was splashed in the eye with an alkaline solution. He is complaining of burning to the eye as well as pain. After performing the primary assessment and finding no​ life-threatening conditions, you​ would:

Which of the following statements is FALSE regarding assessment and treatment of a woman who was the victim of sexual assault

You should question the victim thoroughly about the assaulter in case the police missed any details

​"I am seeing two of​ everything."

You suspect that a patient has a fracture of the left orbit. What statement made by the patient would reinforce this​ suspicion?

Which one of the following patients requires the EMT to remove an impaled object in the field? a. Female with a knife to the right side of her chest; she is short of breath and coughing up blood b. Male patient who fell on a screwdriver, which is impaled through his hand; the patient is in pain and requesting that you remove the screwdriver c. Young female with a broken pencil through her cheek and into the oropharynx; blood from the injury is trickling into her airway d. Male patient with a six-inch knife to his left upper quadrant; there is active bleeding around the injury site

Young female with a broken pencil through her cheek and into the oropharynx; blood from the injury is trickling into her airway

Which one of the following patients requires the EMT to remove an impaled object in the​ field?

Young female with a broken pencil through her cheek and into the​ oropharynx; blood from the injury is flowing into her throat

A patient with diabetes has an elevated blood sugar​ (545 mg/dL)​ and, based on your assessment of his​ condition, will require an injection of insulin. Your EMT partner also has diabetes and administers insulin injections to himself throughout the day as needed. Although the Scope of Practice lists nothing about EMTs giving​ insulin, your partner administers the insulin injection to the patient. The​ patient's blood sugar comes down and her condition improves. Given this​ information, which statement is​ true? A. Since the patient benefited from the​ insulin, your partner acted appropriately B. Because your partner used the​ patient's insulin rather than his​ own, he cannot be reprimanded C. It was permissible for your partner to give the insulin since he has diabetes and knows how to give the injections D. Your partner violated the Scope of Practice and may have his certification revoked

Your partner violated the Scope of Practice and may have his certification revoked

Your ambulance was dispatched for a patient complaining of shortness of breath at 1512. You arrived on scene at 1523 and left at 1538. You arrived at the hospital with the patient at 1557 and were back in service at 1622. Which one of the following is true?

Your scene time was 15 minutes.

You are approached by a college student who states that his college is working with a physician on a project that examines the types of violent trauma occurring within your jurisdiction. He then asks you if he may see copies of all patient care reports that involved patients who were either stabbed or shot. Who would be the best person to contact regarding this​ request? A. The state department of EMS B. Your​ service's privacy officer C. The physician overseeing the research project D. Your​ service's medical director

Your​ service's privacy officer

On which of the following patients would it be MOST appropriate to use the flow-restricted, oxygen-powered ventilation device?

a 21-year-old male with traumatic cardiac arrest

Which of the following patients should you place in the recovery position?

a 31-year-old semiconscious male with low blood sugar and adequate breathing

In which of the following patients would the head tilt-chin lift maneuver be the MOST appropriate method of opening the airway?

a 37-year-old female who is found unconscious in her bed

Which of the following patients would MOST likely require insertion of an oropharyngeal airway?

a 40-year-old unconscious patient with slow, shallow respirations

Which of the following patients would MOST likely have a delayed onset of an allergic reaction?

a 45-year-old male who ingested penicillin

Which of the following patients is breathing adequately?

a conscious male with respirations of 19 breaths/min and pink skin

A patient who is complaining of seeing flashing lights, specks, or "floaters" in his or her field of vision has MOST likely experienced:

a detached retina

A ventilation/perfusion (V/Q ratio) mismatch occurs when:

a disruption in blood flow inhibits the exchange of oxygen and carbon dioxide in the lungs, even though the alveoli are filled with fresh oxygen.

Acute coronary syndrome (ACS) is a term used to describe:

a group of symptoms that are caused by myocardial ischemia.

A decrease in the blood pressure may indicate:

a loss of vascular tone

Epistaxis​ is:

a nosebleed.

Which of the following patients does NOT have an altered mental status?

a patient with an acute allergic reaction and dizziness

A trauma surgeon informs you that the patient you transported to the emergency department earlier was diagnosed with an epidural hematoma. The patient was taken into surgery and the hematoma was removed. Right now the patient is in critical condition. Based on this​ information, the EMT should recognize that the patient suffered​ from:

a pocket of arterial blood that collected between the skull and dura mater.

Which of the following is NOT a common sign or symptom associated with malfunction of an implanted cardiac pacemaker?

a rapid heart rate

A flail chest occurs when:

a segment of the chest wall is detached from the thoracic cage.

A transient ischemic attack (TIA) occurs when:

a small clot in a cerebral artery causes temporary symptoms.

If direct pressure fails to immediately stop severe bleeding from an extremity, you should apply: Select one: A. a splint and elevate the extremity. B. additional sterile dressings. C. a tourniquet proximal to the injury. D. digital pressure to a proximal artery.

a tourniquet proximal to the injury.

A raised, swollen, well-defined area on the skin that is the result of an insect bite or sting is called:

a wheal.

The study of how projectiles affect the body is called: Select one: a. Ballistics b. None of the above c. Kinematics d. Velocity

a. Ballistics

The phenomenon of pressure waves emanating from the bullet, causing damage remote from its path, is known as: Select one: a. Cavitation waves b. Ballistic pressure c. Body cavity dissociation d. Ballistics

a. Cavitation waves

Which of the following would MOST likely occur as the direct result of the second collision in a motor vehicle crash? Select one: a. Deformed steering wheel b. Collapsed dashboard c. Caved-in passenger door d. Intrathoracic hemorrhage

a. Deformed steering wheel

Air bags are designed to: Select one: a. decrease the severity of deceleration injuries. b. prevent the driver from sustaining head trauma. c. be used with or without a shoulder harness. d. prevent a second collision inside the car.

a. decrease the severity of deceleration injuries.

The cervical spine is MOST protected from whiplash-type injuries when the: Select one: a. headrest is appropriately positioned. b. rear end of the vehicle is initially struck. c. air bag correctly deploys upon impact. d. patient tenses up at the time of impact.

a. headrest is appropriately positioned.

The presence of tachycardia following a significant abdominal injury: a. should be assumed to be a sign of shock. b. is most commonly caused by severe pain. c. is always accompanied by hypotension. d. indicates a state of decompensated shock.

a. should be assumed to be a sign of shock.

Blood stasis, changes in the vessel wall, and certain medications affect the: Select one: A. ability of red blood cells to carry oxygen. B. ability of the blood to effectively clot. C. systolic blood pressure exclusively. D. white blood cells' ability to fight infection.

ability of the blood to effectively clot.

An advantage to asking the patient close-ended questions is a(n):

ability to get medical information very quickly

Clinical signs of compensated shock include all of the following, EXCEPT:

absent peripheral pulses.

Gas exchange in the lungs is facilitated by:

adequate amounts of surfactant.

Which of the following is the MOST reliable indicator of adequately performed bag-mask ventilations in an apneic adult with a pulse?

adequate rise of the chest when squeezing the bag

A 51-year-old female presents with a sudden onset of difficulty breathing. She is conscious and alert and able to speak in complete sentences. Her respirations are 22 breaths/min and regular. You should:

administer 100% oxygen via a nonrebreathing mask.

While jogging, a 19 year old male experienced an acute onset of breath and pleuritic chest pain. He is conscious and alert with stable vital signs. Your assessment reveals that he has diminished breath sounds over the left side of the chest. You should:

administer oxygen and transport to the hospital

Your primary assessment of an elderly woman reveals that she is conscious and alert but is experiencing difficulty breathing. She has a history of emphysema, hypertension, and congestive heart failure. As you assess the patient's circulatory status, you should direct your partner to: . perform a head-to-toe secondary assessment. B. assess her oxygen saturation and blood pressure. C. retrieve the stretcher and prepare for transport. D. administer oxygen with the appropriate device.

administer oxygen with the appropriate device

A 66-year-old female with a history of hypertension and diabetes presents with substernal chest pressure of 2 hours' duration. Her blood pressure is 140/90 mm Hg, her pulse is 100 beats/min and irregular, and her respirations are 22 breaths/min with adequate depth. The patient does not have prescribed nitroglycerin, but her husband does. You should

administer oxygen, give her 2 (81mg each tablet) chewable aspirin, and assess her further.

Your patient is a young man who fell down a flight of stairs and is now restless and confused. His airway is​ open, and he is breathing adequately at 18 times per minute. He has a radial pulse of 92 beats per minute and his skin is cool and diaphoretic. Your next action would be​ to:

administer supplemental oxygen.

The MOST appropriate treatment for a patient with a mild upper airway obstruction includes:

administering oxygen and transporting immediately

While you are returning from lunch, a frantic woman flags you down and states that she just found a young child on the roadside who appears to have been hit by a car. She is not sure if the child is breathing. You should immediately:

advise dispatch that you have been flagged down for a possible emergency

Medical control gives you an order that seems inappropriate for the patient's condition. After confirming that you heard the physician correctly, you should: A. obtain consent from the patient and then carry out the order as usual. B. carry out the order and then carefully document it on the run form. C. state that you will not carry out the order because it is inappropriate. D. advise the physician that the order is unclear and ask for clarification.

advise the physician that the order is unclear and ask for clarification.

In the presence of oxygen, the mitochondria of the cells convert glucose into energy through a process called:

aerobic metabolism.

A patient has been splashed in the face with battery acid. He is complaining of being unable to see anything and severe burning to his face. You observe him holding both eyes tightly shut with significant burns on his cheeks and eyelids. In this​ situation, the EMT would assess the​ eyes:

after assessing the primary survey.

A 37-year-old male was pinned between a flatbed truck and a loading dock. On exam, you find bruising to the chest, distended neck veins, bilaterally diminished breath sounds, and bilateral scleral hemorrhaging. a. suspect a severe hemopneumothorax. b. perform a secondary assessment. c. aggressively manage his airway. d. request a paramedic ambulance.

aggressively manage his airway

An Emergency Medical Responder asks you to explain a pneumothorax. You would explain that a pneumothorax occurs​ when:

air accumulates around the​ lung, causing a portion of it to collapse.

An open pneumothorax occurs when:

air enters the pleural space from outside the body.

The three collisions in a frontal impact include all of the following EXCEPT:

airbag striking passenger

A​ 15-year-old female was struck in the mouth with a baseball bat. She has lost her front teeth and is spitting blood. The​ EMT's priority concern when caring for this patient would​ be:

airway compromise

An elderly patient has fallen and hit her head. You assess her level of consciousness as unresponsive using the AVPU scale. Your initial care should focus on:

airway, breathing, and circulation

The foreign substance responsible for causing an allergic reaction is called a(n):

allergen.

Palliating factors regarding a patient's pain involve those that:

alleviate the pain

During your assessment of a 20-year-old man with a severe headache and nausea, you ask him when his headache began, but he does not answer your question immediately. You should: A. tell him that you cannot help him unless he answers your questions. B. ask him if he frequently experiences severe headaches and nausea. C. allow him time to think about the question and respond to it. D. repeat your question because he probably did not hear you.

allow him time to think about the question and respond to it.

What should be one of the first findings that can alert the EMT to a possible head​ injury?

altered mental status.

The actual exchange of oxygen and carbon dioxide occurs in the:

alveolar sacs.

Which of the following MOST accurately defines an allergic reaction?

an exaggerated immune system response to any substance

When blood flow to a particular part of the brain is cut off by a blockage inside a blood vessel, the result is:

an ischemic stroke

Which of the following is a major difference between angina pectoris and AMI?

angina pain usually subsides with rest

When using the pulse oximeter as part of your assessment of a patient, it is important to remember that:

any situation that causes vasoconstriction or loss of red blood cells, such as anemia or bleeding, may result in an inaccurate or misleading value

An unrestrained patient is sitting in his car after an automobile crash. He is conscious and alert, has no visible trauma, and is complaining of neck and back pain. Before removing him from his car, you should:

apply a cervical collar and immobilize him with a vest-style device

When you are treating a patient who sustained a rib​ fracture, you should administer oxygen if the patient is in respiratory distress​ and:

apply a sling and swathe to the patient.

A 20-year-old male has a large laceration to his wrist. He is holding a blood-soaked towel over the wound, but it continues to bleed rapidly. You should:

apply a tourniquet proximal to the wrist.

Direct pressure has failed to control an arterial bleed on a​ patient's leg. The​ EMT's next action would​ be:

apply a tourniquet.

A 17-year-old male was shot in the right anterior chest during an altercation. As your partner is applying oxygen, you perform a rapid assessment and find an open chest wound with a small amount of blood bubbling from it. You should

apply an occlusive dressing to the wound and continue your assessment

You arrive at a meat packing plant to find a​ 30-year-old male cut in the wrist with a sharp knife. The patient appears pale and blood is spurting from the laceration. Your immediate action would be​ to:

apply direct pressure to the laceration.

A 39-year-old male sustained a stab wound to the groin during an altercation at a bar. As you approach the patient, you note that he is conscious, is screaming in pain, and is attempting to control the bleeding, which is bright red and spurting from his groin area. You should:

apply direct pressure to the wound

You respond to a residence for a patient with a severe leg injury following an accident with a chainsaw. When you arrive, you find the patient, a 44-year-old male, lying supine in the backyard. He has a partial amputation of his right lower leg that is actively bleeding. The patient is conscious and breathing adequately; however, he is restless and his skin is diaphoretic. You should:

apply direct pressure to the wound.

You are assessing a young male who was stung on the leg by a scorpion. He is conscious and alert, his breathing is regular and unlabored, and his blood pressure is 122/64 mm Hg. Assessment of his leg reveals a wheal surrounded by an area of redness. He states that he had a "bad reaction" the last time he was stung by a scorpion, and carries his own epinephrine auto-injector. You should:

apply oxygen as needed, clean the area with soap and water or a mild antiseptic, and transport him to the hospital.

Most cases of external bleeding from an extremity can be controlled by: Select one: A. elevating the injured extremity. B. compressing a pressure point. C. applying local direct pressure. D. packing the wound with gauze.

applying local direct pressure.

The MOST common reason that many people experiencing AMI do not seek immediate medical attention is because they

are in denial

The smaller vessels that carry blood away from the heart and connect the arteries to the capillaries are called the: Select one: A. vena cavae. B. capillary arteries. C. arterioles. D. venules.

arterioles

Poor peripheral circulation will cause the skin to appear:

ashen

You respond to the residence of a 55-year-old female with a possible allergic reaction to peanuts that she ate approximately 30 minutes ago. The patient is conscious and alert, but has diffuse urticaria and the feeling that she has a lump in her throat. As your partner applies oxygen to the patient, you should:

ask her if she has prescribed epinephrine.

A 66-year-old male presents with bizarre behavior. His daughter states that he did not seem to recognize her and was very rude to her. The patient is conscious, and has a patent airway and adequate breathing. You should _________________:

ask the daughter how her father normally behaves

A 4-year-old boy had an apparent seizure. He is conscious and calm and is sitting on his mother's lap. His father is sitting in a nearby chair. The child's mother suddenly begins crying uncontrollably, which causes the child to start crying. You should: A. reassure the child's mother that seizures in children are very common and that there is nothing to worry about. B. attempt to calm the child's mother, but avoid separating her from her child because this will increase her anxiety. C. ask the father to hold the child so you can assess him while your partner tries to calm the mother. D. give the child a favorite toy or blanket to hold onto and perform your assessment to the best of your ability.

ask the father to hold the child so you can assess him while your partner tries to calm the mother.

The EMT is properly assessing for a spinal cord injury when she: Select one: a. checks for distal pulses in all four extremities. b. asks the patient if he has pain anywhere along his spinal column. c. palpates the entire spinal column for tenderness. d. asks the patient to spread his fingers apart on both hands.

asks the patient to spread his fingers apart on both hands.

Which of the following medications is commonly given to patients with chest pain to prevent blood clots from forming or getting bigger?

aspirin

Activated charcoal may be indicated for a patient who ingested:

aspirin.

Which of the following is an example of a drugs generic name

asprin

You receive a call to a local daycare center for an unresponsive 8-month-old infant. Upon arrival, you perform an assessment and determine that the infant is not breathing. After delivering two rescue breaths, you should:

assess for the presence of a brachial pulse

A 49-year-old male presents with an acute onset of crushing chest pain and diaphoresis. You should:

assess the adequacy of his respirations.

You are summoned to a patient who was attacked. As you approach a young male lying on the​ sidewalk, you observe an area of dark blood stains on his left pant leg. He appears​ lethargic, pale, and​ diaphoretic, and you can hear sonorous breathing. Your first action in caring for this patient would be​ to:

assess the airway.

Upon arriving at the residence of a patient with a possible cardiac problem, it is MOST important to:

assess the scene for potential hazards.

A 38-year-old female was bitten by fire ants while at the park with her kids. Your primary assessment reveals that she is semiconscious, has profoundly labored breathing, and has a rapid, thready pulse. She has a red rash on her entire body and her face is very swollen. You should:

assist her ventilations with 100% oxygen.

You and your partner are treating a 66-year-old man who experienced a sudden onset of respiratory distress. He is conscious but is unable to follow simple verbal commands. Further assessment reveals that his breathing is severely labored and his oxygen saturation is 80%. You should:

assist his ventilations with a bag-mask device.

A young male was shot in the abdomen by an unknown type of gun. He is semiconscious, has shallow breathing, and is bleeding externally from the wound. As you control the external bleeding, your partner should: Select one: A. assist the patient's ventilations. B. obtain baseline vital signs. C. perform a secondary assessment. D. apply a nonrebreathing mask.

assist the patient's ventilations.

You respond to a college campus for a young male who is acting strangely. After law enforcement has secured the scene, you enter the patient's dorm room and find him sitting on the edge of the bed; he appears agitated. As you approach him, you note that he has dried blood around both nostrils. He is breathing adequately, his pulse is rapid and irregular, and his blood pressure is 200/110 mm Hg. Treatment for this patient includes:

attempting to calm him and giving him oxygen if tolerated.

Kussmaul respirations are an indication that the body is:

attempting to eliminate acids from the blood.

Which of the following is the MOST effective method of assessing the quality of air movement in the lungs?

auscultating breath sounds with a stethoscope

The ability of cardiac muscle cells to contract spontaneously without a stimulus from a nerve source is called:

automaticity.

The electrical stimulus that originates in the heart's primary pacemaker is controlled by impulses from the brain that arrive by way of the:

autonomic nervous system.

A young female experienced a laceration to her left eyeball from flying glass when her boyfriend broke a soda bottle against a wall. There is moderate bleeding and the patient states that she cannot see out of the injured eye. You should:

avoid applying pressure to the globe when you are covering the eye.

Your priority in caring for a patient with a surface contact poisoning is to:

avoid contaminating yourself

An injury that separates various layers of soft tissue, resulting in complete detachment or a flap of skin, is called

avulsion

Bruising to the right upper quadrant of the abdomen following blunt trauma is MOST suggestive of injury to the: a. stomach. b. liver. c. kidney. d. spleen.

b. liver.

When a motor vehicle strikes a tree while traveling at 40 mph, the unrestrained occupant: Select one: a. will decelerate at the same rate as the motor vehicle. b. remains in motion until acted upon by an external force. c. will most likely be thrown over the steering column. d. is thrust under the steering column onto the floorboard.

b. remains in motion until acted upon by an external force.

If one or more occupants in the same vehicle are killed in a crash, the EMT should: Select one: a. transport the survivors only if they have injuries or complain of pain. b. suspect that all living occupants experienced the same serious trauma. c. allow the survivors to refuse transport if they have no obvious injuries. d. rapidly assess only the survivors who have experienced obvious trauma.

b. suspect that all living occupants experienced the same serious trauma.

Left untreated, ____ can lead to premature birth or low birth weight in pregnant women.

bacterial vaginosis

A 38-year-old male was electrocuted while attempting to wire a house. Your assessment reveals that he is unresponsive, pulseless, and apneic. A coworker has shut off the power to the house. You should

begin CPR and apply the AED

You and your partner arrive at the scene of a middle-aged man who collapsed about 5 minutes ago. He is unresponsive, apneic, and pulseless. Bystanders are present, but have not provided any care. You should:

begin high-quality CPR and apply the AED without delay.

You and your partner are caring for a critically injured patient. Your partner is controlling severe bleeding from the patient's lower extremities as you attempt ventilations with a bag-mask device. After repositioning the mask several times, you are unable to effectively ventilate the patient. You should:

begin ventilations using the mouth-to-mask technique

You are dispatched to a residence for a 56-year-old male with an altered mental status. Upon arrival at the scene, the patient's wife tells you that he complained of chest pain the day before, but would not allow her to call EMS. The patient is semiconscious; has rapid, shallow respirations; and has a thready pulse. You should:

begin ventilatory assistance.

Your head trauma patient is​ unconscious, has a patent​ airway, is breathing 18 times a​ minute, and is on O2 via NRB 15 lpm. The oxygen saturation reading is 88 percent. Your next intervention should be​ to:

begin​ bag-valve-mask ventilations.

The posterior tibial pulse can be palpated

behind the medial malleolus.

Which of the following factors would MOST likely cause a patient's pulse rate to be slower than normal?

beta-blockers medications

Intrapulmonary shunting occurs when:

blood coming from the right side of the heart bypasses nonfunctional alveoli and returns to the left side of the heart in an unoxygenated state.

When assessing a patient with signs and symptoms of shock, it is important to remember that:

blood pressure may be the last measurable factor to change in shock.

Closed chest injuries are typically caused by ________?

blunt trauma

An organ or tissue may be better able to resist damage from hypoperfusion if the: Select one: A. body's demand for oxygen is markedly increased. B. body's temperature is considerably less than 98.6°F (37.0°C). C. heart rate is maintained at more than 100 beats/min. D. systolic arterial blood pressure is at least 60 mm Hg.

body's temperature is considerably less than 98.6°F (37.0°C).

When auscultating the blood pressure in a patient's upper extremity, you should place the diaphragm (head) of the stethoscope over the _________ artery.

brachial artery

A contusion to the patient's forehead along with a spider-webbed windshield suggests possible injury to the:

brain

The inferior vena cava returns deoxygenated blood to the right side of the heart from all of the following areas, EXCEPT the:

brain

When relaying patient information via radio, communications should be: A. spoken in a loud voice. B. brief and easily understood. C. coded and scripted. D. lengthy and complete.

brief and easily understood.

Which of the following structures is NOT found in the upper airway?

bronchus

You have been called to the local community hospital for a​ 48-year-old male involved in a motor vehicle collision 2 hours prior. He has been diagnosed with a cerebral contusion and must be transported to a trauma center for specialty care. As a knowledgeable​ EMT, you recognize that a cerebral contusion​ is:

bruising and swelling of the brain tissue.

When performing a full-body scan on a trauma patient, you note the presence of Battle's sign. This is defined as:

bruising behind the ear.

The mesentery is: a. a complex network of blood vessels that supply blood to the liver. b. a layer of thick skeletal muscles that protects the abdominal organs. c. a membranous fold that attaches the intestines to the walls of the body. d. the point of attachment between the small and large intestines.

c. a membranous fold that attaches the intestines to the walls of the body.

When worn properly, a seatbelt should lie: a. above the anterior posterior iliac spines of the pelvis and below the hip joints. b. across the abdominal wall at the level of the umbilicus and against the hip joints. c. below the anterior superior iliac spines of the pelvis and against the hip joints. d. across the abdominal wall at the level of the diaphragm and below the hip joints.

c. below the anterior superior iliac spines of the pelvis and against the hip joints.

Accumulation of blood in the abdominal cavity will MOST likely cause: a. nausea or vomiting. b. referred pain. c. distention. d. diffuse bruising.

c. distention.

Contraction or tensing of the abdominal muscles in an effort to ease pain is called: a. flexing. b. referring. c. guarding. d. withdrawing.

c. guarding.

A driver involved in a rollover motor vehicle crash will MOST likely experience serious injuries or death if he or she: Select one: a. experiences multiple impacts. b. remains within the vehicle. c. is ejected or partially ejected. d. is wearing only a lap belt.

c. is ejected or partially ejected.

A patient who was complaining of a headache is being transported to the hospital. After providing a report to the emergency department, the patient begins to seize. The seizure lasts for one minute and then stops. At this time, you are five minutes from the hospital. After providing care for the seizure, your next action should be to:

call the hospital and inform them of the seizure

Pulmonary contusions are serious injuries because​ they:

can interfere with oxygen exchange.

The nasopharyngeal airway is MOST beneficial because it:

can maintain a patent airway in a semiconscious patient with a gag reflex.

A patient exhibits retrograde amnesia when​ she:

cannot remember falling and hitting her head.

The primary waste product of aerobic metabolism is:

carbon dioxide

A 56-year-old male is found semiconscious by his wife. Your assessment reveals that his respirations are rapid and shallow, his pulse is rapid and irregular, and his blood pressure is low. The patient's wife states that he complained of left arm pain and nausea the day before, but would not allow her to call 9-1-1. The MOST likely cause of this patient's present condition is:

cardiogenic hypo perfusion.

Your patient has a decreased cardiac output and poor myocardial contractility. This will likely lead to ___________.

cardiogenic shock

The head and brain receive their supply of oxygenated blood from the:

carotid arteries.

Emanation of pressure waves that can damage nearby structures

cavitation

The phenomenon of pressure waves emanating from the bullet, causing damage remote from its path, is known as:

cavitation

The jaw-thrust maneuver is used to open the airway of patients with suspected:

cervical spine injuries.

When treating an 80-year-old patient who is in shock, it is important to remember that:

changes in gastric motility may delay gastric emptying, which increases the risk for vomiting.

Your assessment of an unresponsive patient reveals that her breathing is inadequate. Your MOST immediate action should be to:

check her airway for obstructions.

When would it be MOST appropriate for a patient to take his or her prescribed nitroglycerin?

chest pain that does not immediately subside with rest

With regard to the assessment of a patient's cardiovascular status, capillary refill time is MOST reliable in:

children who are younger than 6 years of age.

In rare cases, ____ causes arthritis that may be accompanied with skin lesions and inflammation of the eyes and urethra

chlamydia

A 39-year-old female experienced a severe closed head injury. She is unresponsive with her eyes slightly open; her pupils are bilaterally dilated and slow to react, In addition to managing problems with airway, breathing, and circulation, you should

close her eyes and cover them with a moist dressing

Often the most important intervention for a sexual assault patient is to _______ and transport to a facility with a staff specially trained to deal with this scenario.

comforting reassurance

Your _____ is the best tool to gain the patient's confidence to seek medical help.

compassion

Your patient is in shock, but the body's defense mechanisms are currently able to maintain adequate circulation. This is called ___________.

compensated shock

In a motor vehicle collision, as the passenger's head hits the windshield, the brain continues to move forward until it strikes the inside of the skull, resulting in a _____ injury.

compression

While en route to the scene of a shooting, the dispatcher advises you that the caller states that the perpetrator has fled the scene. You should:

confirm this information with law enforcement personnel at the scene

Your patient denies difficulty breathing, but displays signs of respiratory distress. You state, "I know that you said you are breathing fine, but you cannot speak more than two to three words at a time without gasping for air." Your response is an example of

confrontation.

Rapid, labored breathing in a patient with signs and symptoms of AMI should make you suspicious for:

congestive heart failure.

The cerebrum is the largest part of the​ brain, and is responsible​ for:

consciousness and sensory​ functions, emotions, and personality.

The normal response of the pupils while examined with a penlight is described​ as:

consensual.

Ethnocentrism is defined as: A. subconsciously forcing your cultural values onto a patient because you feel that your own values are more acceptable. B. considering your own cultural values to be more important when interacting with people of a different culture. C. suspecting that a person has an ulterior motive based on the tone of his or her voice when answering a question. D. understanding that people from different cultural backgrounds respond to pain and stress differently.

considering your own cultural values to be more important when interacting with people of a different culture.

A 73-year-old man presents with a generalized rash, which he thinks may have been caused by an antibiotic that he recently began taking. He has a history of coronary artery disease, hypertension, and emphysema. He is conscious and alert, his blood pressure is 144/94 mm Hg, and his pulse is 64 beats/min and regular. You auscultate his breath sounds and hear scattered wheezing, although he is not experiencing respiratory distress. In addition to administering oxygen, you should:

contact medical control if needed, transport the patient, and monitor him for signs of deterioration.

The cerebellum is described as an area of the brain​ that:

controls reflexes and assists in maintaining body posture.

Energy can be:

converted

Common signs and symptoms of severe hyperglycemia include all of the following, EXCEPT:

cool, clammy skin

To select the proper size oropharyngeal airway, you should measure from the:

corner of the mouth to the earlobe.

The myocardium receives oxygenated blood from the __________, which originate(s) from the __________.

coronary arteries, aorta

Signs of a pulmonary blast injury include:

coughing up blood

A compression injury to the anterior portion of the brain and stretching of the posterior portion is called a(n) ________ brain injury.

coup-contrecoup

You are on the scene of a shooting. Your assessment reveals a​ 23-year-old male who has been shot twice. The first wound is to the left lower quadrant of the abdomen and is actively bleeding. The second wound is to the left lateral chest and bubbles every time the patient takes a breath. Your immediate action would​ be:

cover the chest wound with a gloved hand.

A man grinding metal took his safety goggles off and was struck in the face with a shower of tiny metal fragments. He is complaining of pain and visual disturbance in his right eye. When assessing the​ eye, you note that the globe is red with several pieces of metal embedded in the tissue. Proper care of this patient would​ include:

covering both eyes with a bandage.

The __________ cartilage is a firm ring that forms the inferior part of the larynx.

cricoid

By what mechanism is a person injured when he or she falls from a significant height? Select one: a. Potential energy is created as the person is falling; the potential energy is then converted into kinetic energy upon impact. b. Kinetic energy is converted to potential energy; the potential energy is then converted into the work of bringing the body to a stop. c. As the person falls, the amount of kinetic energy is converted into work; work is then converted to kinetic energy upon impact. d. Potential energy is converted to kinetic energy; the kinetic energy is then converted into the work of bringing the body to a stop.

d. Potential energy is converted to kinetic energy; the kinetic energy is then converted into the work of bringing the body to a stop.

Assuming that no obvious signs of intra-abdominal injury are present, which of the following injuries would MOST likely cause an injury to the liver or spleen to be overlooked? a. Pelvic fracture b. Lumbar spine fracture c. Femur fracture d. Shoulder fracture

d. Shoulder fracture

Peritonitis usually occurs when: a. bacteria or viruses invade the walls of the gastrointestinal tract. b. the vessels that supply the abdominal organs become inflamed. c. solid abdominal organs bleed secondary to penetrating trauma. d. hollow abdominal organs are damaged and spill their contents into the peritoneal cavity.

d. hollow abdominal organs are damaged and spill their contents into the peritoneal cavity.

During your assessment of a patient who experienced blunt trauma to the abdomen, you notice bruising around the umbilicus. This is a sign of: a. rupture of a hollow organ. b. a ruptured spleen. c. a severe liver laceration. d. intra-abdominal bleeding.

d. intra-abdominal bleeding.

Early bruising following abdominal trauma often manifests as: a. localized pain. b. gross distention. c. dark-purple marks. d. red areas of skin.

d. red areas of skin.

When caring for an occupant inside a motor vehicle equipped with an air bag that did not deploy upon impact, you should: Select one: a. realize that the air bag malfunctioned at the time of impact. b. recognize that the force of impact was most likely not severe. c. suspect that the patient may have experienced serious injuries. d. remember that it could still deploy and seriously injure you.

d. remember that it could still deploy and seriously injure you.

A 20-year-old waitress spilled hot coffee and burned her right forearm. When assessing the burn, you note a full-thickness burn surrounded by partial-thickness burns. In addition, she complains of pain in the surrounding partial-thickness burn, but not in the center where the full-thickness burn is located. As an EMT, you recognize that the patient is not localizing pain to the center of the burn because of:

damage to pain receptors in the dermis

A patient's short-term memory is MOST likely intact if he or she correctly answers questions regarding:

date and event

Significant clues to the possibility of severe injuries in motor vehicle collisions include:

death of a passenger

Air bags are designed to:

decrease the severity of deceleration injuries

Ischemic heart disease is MOST accurately defined as: A. decreased blood flow to the heart muscle due to coronary dilation. B. death of a portion of the heart muscle due to a decrease in oxygen. C. absent myocardial blood flow due to a blocked coronary artery. D. decreased blood flow to one or more portions of the myocardium.

decreased blood flow to one or more portions of the myocardium

Cyanosis of the skin is caused by

decreased blood oxygen.

Cardiogenic shock following AMI is caused by:

decreased pumping force of the heart muscle

Which of the following would NOT cause a decrease in tidal volume?

deep respirations

Which of the following signs is commonly observed in patients with right-sided heart failure?

dependent edema

You are dispatched to a residence for a 40-year-old female who fainted. Upon your arrival, the patient is conscious and alert, and states that she is fine. Her husband tells you that she fainted after receiving news that her sister was killed in a car crash. You offer oxygen to the patient, but she refuses to accept it. At this point, your primary concern should be to:

determine if she was injured when she fainted.

A 50-year-old male is found unconscious in his car. There were no witnesses to the event. When gathering medical history information for this patient, the EMT should:

determine if the patient has a medical alert bracelet or wallet card

A patient with profuse sweating is referred to as being

diaphoretic

Inhalation occurs when the:

diaphragm and intercostal muscles contract and cause a decrease in intrathoracic pressure

The thoracic cavity is separated from the abdominal cavity by the:

diaphram

A percutaneous transluminal coronary angioplasty (PTCA) restores blood flow to the ischemic myocardium by

dilating the affected coronary artery with a small inflatable balloon.

Nitroglycerin relieves cardiac-related chest pain by

dilating the coronary arteries and improving cardiac blood flow.

Which of the following medications blocks the release of histamines?

diphenhydramine (Benadryl)

A 66-year-old woman presents with a stabbing pain in the middle of her chest that radiates to her back. She tells you that the pain suddenly began about 30 minutes ago and has been severe since the onset. She has a history of hypertension, but admits to being noncompliant with her antihypertensive medications. When you assess her, you find that her blood pressure is significantly higher in her left arm than it is in her right arm. What are her signs and symptoms MOST indicative of?

dissecting aortic aneurysm

In nontrauma patients, an early indicator of internal bleeding is: Select one: A. a rapid, thready pulse. B. a decreasing blood pressure. C. dizziness upon standing. D. rapid, shallow breathing.

dizziness upon standing.

Energy that is available to cause injury ____ when an object's weight doubles, but ____ when its speed doubles

doubles, quadruples

Common side effects of epinephrine include all of the following, EXCEPT:

drowsiness.

Prior to attaching the AED to a cardiac arrest patient, the EMT should:

dry the chest off if it is wet

When assessing a​ patient, you note a bruise to his chest. On the prehospital care​ report, this injury is properly documented​ as:

ecchymosis.

The pulse oximeter is an assessment tool used to evaluate the:

effectiveness of oxygenation

Communicating with patients who are deaf or hard-of-hearing can be facilitated by doing all of the following, EXCEPT: A. placing yourself in a position to ensure that the patient can see your lips. B. elevating the tone of your voice and exaggerating word pronunciation. C. shining a light on your face when you are in a darkened environment. D. providing pen and paper if the patient prefers to write his or her response.

elevating the tone of your voice and exaggerating word pronunciation.

A 37-year-old male has an apparent foreign body airway obstruction. He is conscious and alert and is coughing forcefully. His skin is pink, warm, and moist. The MOST appropriate treatment for this patient includes:

encouraging him to cough and transporting.

Prior to defibrillating a patient with an AED, it is MOST important that you:

ensure that no one is touching the patient.

The purpose of the pin-indexing system that has been established for compressed gas cylinders is to:

ensure that the correct regulator is used for the cylinder.

When approaching a 32-year-old male who is complaining of traumatic neck pain, you should:

ensure that the patient can see you approaching him.

Upon arriving at a potentially unsafe scene, you should:

ensure that you are safe

The​ EMT's initial concern when treating a patient with a gunshot wound to the chest​ is:

ensuring an open airway.

Structures of the lower airway include all of the following, EXCEPT the:

epiglottis

The leaf-shaped structure located superior to the larynx is called the:

epiglottis.

Which of the following structures is contained within the mediastinum?

esophagus

A patient has suffered an open crush injury to his elbow. As you​ approach, you note the patient sitting upright holding a towel to his elbow. Although the towel is soaked with​ blood, the elbow does not appear to be actively bleeding. Your first action in caring for this patient should be​ to:

evaluate the​ patient's airway.

Medical direction has advised you to place a patient who is obese and extremely short of breath in a supine position. Knowing that doing so will cause the patient to become more short of breath, your best action would be to:

explain your concern to the physician and ask for clarification

Make sure to use ___________ when attempting to control vaginal bleeding

external pads

The process of exchanging oxygen and carbon dioxide between the alveoli and the blood of the capillaries is called:

external respiration

Anaphylaxis is MOST accurately defined as a(n):

extreme allergic reaction that may affect multiple body systems.

The MOST common error associated with the use of the AED is

failure of the EMT to ensure the battery is charged

Neurogenic shock occurs when:

failure of the nervous system causes widespread vasodilation.

Each ovary produces an ovum in alternating months and releases it into the:

fallopian tubes

A blood pressure cuff that is too small for a patient's arm will give a:

falsely high systolic and diastolic reading

Which of the following is of LEAST pertinence when obtaining medical history information from a patient complaining of chest discomfort?

family history of hypertension

Risk factors for AMI that cannot be controlled include:

family history.

The iliac arteries immediately subdivide into the

femoral arteries.

Blood that is ejected from the right ventricle

flows into the pulmonary arteries.

Blood that is ejected from the right ventricle:

flows into the pulmonary arteries.

A 39-year-old male sustained a large laceration to his leg during an accident with a chainsaw and is experiencing signs and symptoms of shock. You should first: Select one: A. follow appropriate standard precautions. B. place the patient on high-flow oxygen. C. perform a rapid head-to-toe assessment. D. apply direct pressure to the wound.

follow appropriate standard precautions.

You are called to the scene of a possible assault. Upon arrival, you are directed by police to a dark room where you find a 22-year-old woman who says she was sexually assaulted by a coworker this afternoon. The patient tells you that she would really like to be transported to the hospital but refuses a physical examination. You should:

follow your system's refusal of treatment policy and respect the patient's wishes without judgment.

The spinal cord exits the cranium through the:

foramen magnum.

A 67-year-old male presents with weakness, dizziness, and melena that began approximately 2 days ago. He denies a history of trauma. His blood pressure is 90/50 mm Hg and his pulse is 120 beats/min and thready. You should be MOST suspicious that this patient is experiencing: Select one: A. an aortic aneurysm. B. acute appendicitis. C. gastrointestinal bleeding. D. intrathoracic hemorrhaging.

gastrointestinal bleeding.

A 71-year-old female slipped on a rug and fell. She is conscious and alert and complains of severe pelvic pain. Her respirations are 22 breaths/min with adequate depth and her heart rate is 120 beats/min. Which of the following would NOT be appropriate for this patient?

gentle palpation of the pelvis

Whenever possible, a female sexual assault victim should be:

given the option of being treated by a female EMT.

Symptoms of ______ appear approximately 2 to 10 days after exposure.

gonorrhea

Medium-velocity penetrating injuries may be caused by a:

handgun

You should suspect that a patient is experiencing respiratory failure if he or she:

has bradycardia and diminished muscle tone

When a woman presents with abdominal pain or other vague symptoms, the EMT is often unable to determine the nature of the problem until he or she:

has gathered patient history information

Signs and symptoms of a hypertensive emergency would MOST likely be delayed in patients who:

have chronic hypertension.

CPAP is indicated for patients who:

have pulmonary edema and can follow verbal commands.

Patients with a subarachnoid hemorrhage typically complain of a sudden severe:

headache.

The cervical spine is MOST protected from whiplash-type injuries when the:

headrest is appropriately positioned.

Gastrointestinal bleeding should be suspected if a patient presents with: Select one: A. hematuria. B. hemoptysis. C. dyspnea. D. hematemesis.

hematemesis.

Chemicals that are responsible for the signs and symptoms of an allergic reaction to a bee sting include:

histamines and leukotrienes.

When assessing a patient with a medical complaint, which of the following would MOST likely reveal the cause of his or her problem?

history taking

Urticaria is the medical term for:

hives.

An effective and reliable communication system is essential to the EMT and the EMS system because

hospitals can better prepare for the arrival of a patient.

An effective and reliable communication system is essential to the EMT and the EMS system because:

hospitals can better prepare for the arrival of a patient.

The ability of a person's cardiovascular system to compensate for blood loss is MOST related to: Select one: A. his or her baseline blood pressure. B. the part of the body injured. C. how fast his or her heart beats. D. how rapidly he or she bleeds.

how rapidly he or she bleeds.

If ventilation is impaired, carbon dioxide levels in the bloodstream will increase. This condition is called:

hypercarbia.

Common side effects of nitroglycerin include all of the following, EXCEPT

hypertension.

Major risk factors for AMI include all of the following, EXCEPT:

hypoglycemia.

Which of the following signs or symptoms would you NOT expect to encounter in a patient with congestive heart failure?

hypotension and flat jugular veins

The MOST significant complication associated with oropharyngeal suctioning is:

hypoxia due to prolonged suction attempts

Advil,nuprin,and Motrin are brand(trade) names for the generic medication

ibuprofen

If you cannot palpate a pulse in an unresponsive patient, you should:

immediately begin cardiopulmonary resuscitation (CPR)

You respond to a residence for a 40 year old female who was assaulted by her husband, the scene has been secured by law enforcement. Upon your arrival, you find the patient lying supine on the floor in the kitchen. She is semiconscious with severely labored breath sounds. As your partner is supporting her ventilations, you should:

immediately request ALS support

After the AED has delivered a shock, the EMT should:

immediately resume CPR.

You respond to a call for a female pedestrian who has been struck by a car. As your partner maintains manual stabilization of her head, you perform a primary assessment. She is unconscious, has ineffective breathing, and has bloody secretions in her mouth. You should:

immediately suction her oropharynx.

A 45-year-old male was working on his roof when he fell approximately 12 feet, landing on his feet. He is conscious and alert and complains of an ache in his lower back. He is breathing adequately and has stable vital signs. You should: a. immobilize his spine and perform a focused secondary exam. b. obtain a Glasgow Coma Score value and give him oxygen. c. allow him to refuse transport if his vital signs remain stable. d. perform a rapid head-to-toe exam and immobilize his spine.

immobilize his spine and perform a focused secondary exam.

Without adequate oxygen, the body's cells:

incompletely convert glucose into energy, and lactic acid accumulates in the blood.

Which of the following would cause the greatest increase in cardiac output?

increased heart rate and increased stroke volume

Headache, vomiting, altered mental status, and seizures are all considered early signs of:

increased intracranial pressure.

As brain herniation​ occurs, the body responds​ by:

increasing blood pressure to perfuse the brain.

All of the following will help minimize the risk of gastric distention when ventilating an apneic patient with a bag-mask device, EXCEPT:

increasing the amount of delivered tidal volume.

Awareness of and concern for potentially serious obvious and underlying injuries is referred to as the:

index of suspicion

Activated charcoal is given to patients who have ingested certain substances because it:

induces vomiting and empties the stomach.

You are dispatched to a residence for a 66-year-old male who, according to family members, has suffered a massive stroke. Your primary assessment reveals that the patient is unresponsive, apneic, and pulseless. You should:

initiate CPR and attach an AED as soon as possible.

A patient sustained a crush injury after being struck with a baseball bat in the left upper quadrant of the abdomen. Assessment shows bruising to that area with instability noted to the rib cage. The​ EMT's highest concern regarding this injury should​ be:

injury to the spleen.

Elevation of the rib cage during inhalation occurs when: a. intrathoracic pressure decreases b. the diaphragm descends. c. the intercostal muscles contract. d. abdominal contents descend.

intercostal muscles contract

In contrast to inhalation, exhalation:

is a passive process caused by increased intrathoracic pressure.

A simple pneumothorax:

is commonly caused by blunt chest trauma

A driver involved in a rollover motor vehicle crash will MOST likely experience serious injuries or death if he or she:

is ejected or partially ejected.

If a woman with vaginal bleeding reports syncope, the EMT should assume that she:

is in shock

The mental status of a patient who has experienced a generalized seizure:

is likely to improve over a period of 5 to 30 minutes.

The cricoid cartilage:

is the only complete circular cartilage of the trachea.

The AED is MOST advantageous to the EMT because:

it delivers prompt defibrillation to patients with ventricular fibrillation.

Energy of a moving object

kinetic energy

Substance abuse is MOST accurately defined as:

knowingly misusing a substance to produce a desired effect.

You arrive at the scene of a major motor vehicle crash. The patient, a 50 year old female, was removed form her vehicle prior to your arrival. Bystanders who removed her state that she was not wearing a seatbelt. The patient is unresponsive, tachycardic, and diaphoretic. Your assessment reveals bilaterally clear and equal breath sounds, a midline trachea, and collapsed jugular veins. You should be MOST suspicious that this patient has experienced a:

laceration of the aorta

When using an auto-injector to give epinephrine, the primary injection site is the:

lateral portion of the thigh.

Rape is considered a _____ diagnosis, not a medical diagnosis

legal

A 40-year-old male presents with pain to the right upper quadrant of his abdomen. He is conscious and alert with stable vital signs. During your assessment, you note that his skin and sclera are jaundiced. You should suspect:

liver dysfunction

You are dispatched to a residence where a middle-aged man was found unconscious in his front yard. There are no witnesses who can tell you what happened. You find him in a prone position; his eyes are closed and he is not moving. Your FIRST action should be to:

log roll him as a unit to a supine position.

An unresponsive patient was ejected from a car as it rolled at a high rate of speed. The primary assessment is complete and you are providing positive pressure ventilation with​ high-concentration oxygen. The patient has a significant deformity to the right side of his​ head, and both pupils are equal but slow to react to light. He also has deformity to the forearms. Vital signs​ are: pulse​ 144, respirations 24 and​ inadequate, blood pressure​ 90/50 mmHg, and SpO2​ 97%. Based on this ​ presentation, the EMT​ should:

look for evidence of bleeding in an area of the body other than the brain.

At a construction​ site, an​ 18-year-old male was struck in the head by a pallet of concrete blocks as it was being moved by a crane. He has a large laceration to the left side of his head with noted instability to the skull beneath the wound. Moderate bleeding from the injury site is also noted. His airway is​ open, his breathing is​ adequate, and his pulse is strong and regular. Proper care for this patient should​ include:

loose dressing over the top of the laceration.

The hypoxic drive—the primary stimulus to breathe for patients with certain chronic respiratory diseases—is influenced by:

low blood oxygen levels.

A patient with atherosclerotic heart disease experiences chest pain during exertion because the:

lumen of the coronary artery is narrowed and cannot accommodate increased blood flow.

A student in a science lab has had an unknown chemical splashed into her eye. After assessing the airway, breathing, and circulation, the EMT should immediately:

lush the eye with copious amounts of tap water

When caring for a woman who is experiencing a gynecologic emergency, the EMT's main focus should be to:

maintain her ABCs and transport without delay.

A 48-year-old male is found unconscious in the garden by his wife. When you arrive at the scene and assess the man, you find that he is unresponsive, has severely labored breathing, and has hives over his entire trunk. You should:

maintain his airway and assist his ventilations.

The lower jawbone is called the:

mandible

End-tidal carbon dioxide (ETCO2) is defined as the:

maximal concentration of CO2 at the end of an exhaled breath.

When a person is lying supine at the end of exhalation, the diaphragm:

may rise as high as the nipple line

Your quick primary assessment of the patient and evaluation of the ____ can help to direct lifesaving care and provide critical information to the hospital staff

mechanism of injury

Each cell of the body combines nutrients and oxygen and produces energy and waste products through a process called:

metabolism.

The diastolic pressure represents the

minimum amount of pressure that is always present in the arteries.

You arrive at a local grocery store approximately 5 minutes after a 21-year-old female stopped seizing. She is confused and disoriented; she keeps asking you what happened and tells you that she is thirsty. Her brother, who witnessed the seizure, tells you that she takes valproate (Depakote) for her seizures, but has not taken it in a few days. He also tells you that she has diabetes. In addition to administering oxygen, you should:

monitor her airway and breathing status and assess her blood glucose level.

After administering 0.3 mg of epinephrine via auto-injector to a 22-year-old female with an allergic reaction, you note improvement in her breathing and dissipation of her hives. However, she is still anxious and tachycardic. You should:

monitor her closely but recall that anxiety and tachycardia are side effects of epinephrine.

When you are communicating with an older patient, it is important to remember that: A. most older people think clearly and are capable of answering questions. B. hostility and confusion should be presumed to be due to the patient's age. C. your questions should focus exclusively on the patient's obvious problem. D. speaking loudly and distinctly will ensure that the patient can hear you. Feedback

most older people think clearly and are capable of answering questions.

The chief complaint is MOST accurately defined as the:

most serious thing the patient is concerned about.

Which of the following statements regarding the one-person bag-mask device technique is correct?

mouth-to-mask technique with a one-way valve.

Tidal volume is defined as the volume of air that:

moves into or out of the lungs in a single breath.

Which of the following organs or tissues can survive the longest without oxygen?

muscle

If a patient with a chest injury is only able to inhale small amounts of air per breath, he or she:

must increase his or her respiratory rate to maintain adequate minute volume.

Angina pectoris most commonly occurs when:

myocardial oxygen demand exceeds the supply

An acute myocardial infarction (AMI) occurs when:

myocardial tissue dies secondary to an absence of oxygen.

During the normal wound-healing process, bleeding may occur from even a minor injury because:

new capiliaries that stem from intact capillaries are delicate and take time to become as stable as the preexisting capilleries

Bacterial vaginosis is a condition that occurs when:

normal bacteria in the vagina are replaced by an overgrowth of other bacterial forms.

In contrast to viral hepatitis, toxin-induced hepatitis

not a communicable disease

When providing a patient report via radio, you should protect the patient's privacy by: A. not disclosing his or her name. B. withholding medical history data. C. using coded medical language. D. refraining from objective statements.

not disclosing his or her name.

A young female pedestrian was hit by a car and thrown 15​ feet, striking her head against a metal guardrail. You start your secondary assessment and note a deep depression to the right parietal area of her head with intact skin overlying the area. You​ should:

note the injury and continue assessing the patient.

Which of the following is a severe burn in a 2-year- old child?

ny full-thickness burn, regardless of its location on the body

Before giving activated charcoal, you should:

obtain approval from medical control.

Prior to assisstin a patient with his or her prescribed nitroglycerin, the EMT should:

obtain authorization from medical control

In contrast to AMI, a dissecting aortic aneurysm:

often presents with pain that is maximal from the onset

Which of the following MOST accurately describes paradoxical movement of the chest wall?

only one section of the chest rises on inspiration while another area falls

A 40-year-old male crashed his motorcycle into a tree. He is semiconscious, has snoring respirations, and has a laceration to the forearm with minimal bleeding. You should:

open his airway with the jaw-thrust maneuver.

An injured patient is assigned a total score of 9 on the GCS. He is assigned a score of 2 for eye opening, a score of 3 for verbal response, and a score of 4 for motor response. Which of the following clinical findings is consistent with his GCS score?

opens eyes in response to pain, uses inappropriate words, withdraws from pain

When providing a lengthy radio report, the most important reason to pause every 30 seconds is so that:

other people can use the radio for an emergency transmission.

Airborne substances should be diluted with:

oxygen.

Early signs and symptoms of intra-abdominal bleeding include: Select one: A. significant hypotension. B. bruising only. C. widespread ecchymosis. D. pain and distention.

pain and distention.

In infants and small children, skin color should be assessed on the:

palms and soles

During your physical examination of a patient who was hit in the chest with a baseball​ bat, you notice that a small section of his ribcage sinks when he inhales and moves outward when he exhales. This is known​ as:

paradoxical movement.

A spinal cord injury at the level of C7 would MOST likely result in:

paralysis of the intercostal muscles

You have sealed the open chest wound of a 40 year old male who was stabbed in the anterior chest. Your assessment reveals that he is experiencing increasing respiratory distress and tachycardia, and is developing cyanosis. You should:

partially remove the dressing.

A full-body scan should be performed on:

patients with a significant MOI and unresponsive medical patients

The MOST serious complication associated with using a nasopharyngeal airway in a patient with trauma to the head or face is:

penetrating the cranium.

Possible causes of vaginal bleeding include all the following EXCEPT

peptic ulcer

Your EMS team is performing CPR on a 60-year-old male in cardiac arrest. You connect the AED, push the analyze button, and receive a "no shock advised" message. You should:

perform CPR for 2 minutes and reassess

The MOST effective way to determine whether your patient's problem is medical or traumatic in origin is to:

perform a careful and thorough assessment

You respond to the scene of a motor vehicle collision. Upon arrival, you find the driver, a young female, sitting on the curb. She is confused, is in obvious respiratory distress, and has pale, moist skin. As your partner manually stabilizes her head, you perform a primary assessment. After performing any immediate livesaving treatment, you should:

perform a rapid scan of her entire body and prepare for immediate transport.

A 71-year-old male is semiconscious following a sudden, severe headache. There is vomitus on his face and his respirations are slow and shallow. The EMT must immediately:

perform oropharyngeal suctioning.

An unrestrained​ middle-aged female was thrown from her car as it rolled into the median of an interstate. She is​ supine, appears​ unresponsive, has blood on her​ face, and has snoring respirations. Of the listed​ actions, the one that you would perform earliest in your treatment would be​ to:

perform the​ jaw-thrust maneuver.

Your patient is a young adult male with a gunshot wound to the left side of his head. He is unresponsive with snoring respirations. He is breathing 6 times per minute with a pulse rate of 52 and a blood pressure of​ 192/104 mmHg. His radial pulse is strong and his skin is​ cool, but not diaphoretic. In addition to​ in-line manual stabilization of the cervical​ spine, the first intervention for this patient would​ be:

performing a​ jaw-thrust maneuver.

Circulation of blood within an organ or tissue in adequate amounts to meet the cells' oxygen, nutritional, and waste-removal needs is termed _______. Select one: A. coagulation B. hypoperfusion C. perfusion D. hemorrhage

perfusion

A​ middle-aged male has been stabbed once in the left anterior chest. His airway is​ patent, respirations​ tachypneic, pulse weak and​ rapid, and skin cool and diaphoretic. Breath sounds are clear and equal bilaterally. The vital signs​ are: pulse​ 140, respirations​ 24, blood pressure​ 100/78 mmHg, and SpO2​ 96% on supplemental oxygen. Given this​ presentation, you would have a high index of suspicion​ for:

pericardial tamponade.

During your assessment of a 6-month-old male with vomiting and diarrhea, you note that his capillary refill time is approximately 4 seconds. From this information, you should conclude that his:

peripheral circulation is decreased.

The _______________ nerves control the diaphragm.Select one: a. costal b. vagus c. phrenic d. intercostal

phrenic

A 43-year-old man is experiencing a severe nosebleed. His blood pressure is 190/110 mm Hg and his heart rate is 90 beats/min and bounding. Preferred treatment for this patient includes: Select one: A. placing a rolled 4² × 4² dressing between his lower lip and gum. B. pinching the patient's nostrils and having him lean forward. C. having the patient pinch his own nostrils and then lie supine. D. packing both nostrils with gauze pads until the bleeding stops.

pinching the patient's nostrils and having him lean forward.

The EMT is by the side of a young female who was cut while washing dishes. She has a jagged laceration to her left forearm that is bleeding steadily and heavily. When attempting to control the​ bleeding, the EMT would​ first:

place a large sterile dressing over the site with hand pressure applied over the top of the dressing.

A female patient who suffered a straddle injury while trying to balance on top of a chain link fence continues to bleed from the vaginal area despite your initial attempt at hemorrhage control. At this time, you should a. gently put sterile gauze dressings into the opening of the vaginal canal. b. clean the area with sterile saline and reapply direct pressure. c. place another dressing over the first and maintain direct pressure. d. apply an ice pack wrapped in a towel to the genital area.

place another dressing over the first and maintain direct pressure.

If a patient with vaginal bleeding presents with a rapid pulse and pale or cool skin, you should:

place the patient in a supine position with her legs elevated.

When performing a physical exam on a victim of sexual assault, you should:

place the patient's clothing into a paper bag.

A young male was riding a motorcycle when he lost control and was ejected. The patient is unresponsive and receiving positive pressure ventilation. The secondary assessment reveals bright blood flowing from both nostrils. Management of this bleeding would​ include:

placing a loose dressing under the nose.

A 67-year-old female presents with difficulty breathing and chest discomfort that awakened her from her sleep. She states that she has congestive heart failure, has had two previous heart attacks, and has prescribed nitroglycerin. She is conscious and alert with adequate breathing. Her blood pressure is 94/64 mm Hg and her heart rate is 120 beats/min. Treatment for this patient includes:

placing her in an upright position.

In order to avoid exacerbating a patients injury, it is especially important to use extreme caution when providing positive-pressure ventilation to patients with a:

pneumothorax

You are assessing a 72-year-old man with abdominal pain. The patient is sitting in a chair; he is conscious, alert, and calm. As you are talking to the patient, your partner discreetly directs your attention to a handgun, which is located on a nearby table. You should:

position yourself in between the patient and the gun and ask your partner to request law enforcement assistance.

When performing a full-body scan on a supine patient, what part of the body is typically assessed last?

posterior

Product of mass, gravity, and height

potential energy

A 28 year old male was struck in the chest with a baseball bat during an altercation. He is conscious and alert and complains of severe chest pain. Your assessment reveals a large area of ecchymosis over the sternum and a rapid, irregular pulse. In addition to providing supplemental oxygen, you should:

prepare for immediate transport

A 50-year-old male presents with an altered mental status. His wife tells you that he had a "small stroke" 3 years ago but has otherwise been in good health. The patient is responsive but unable to follow commands. After administering oxygen, you should:

prepare for immediate transport.

A palpable pulse is created by:

pressure waves through the arteries caused by cardiac contraction.

A small metal rod was dislodged from a metal lathe and is now impaled in a​ man's inner thigh. Assessment indicates that there is no active bleeding from the​ site, but the surrounding tissues are ecchymotic. The paramedic asks you to stabilize the rod with bulky dressings. You recognize this action as important because it​ will:

prevent motion of the rod and further internal injury.

Immediately life-threatening chest injuries must be found and managed during the _________?

primary assessment

Immediately after giving an epinephrine injection, you should:

properly dispose of the syringe.

You are dispatched to the county jail for an inmate who is "sick." When you arrive, you find the patient, a 33-year-old male, unresponsive. His airway is patent and his respirations are rapid and shallow. Your initial action should be to:

provide assisted ventilation.

A patient with blunt chest wall trauma has a flail section to her chest. She is conscious and confused and breathing rapidly and shallowly. After manually stabilizing the flail section of the chest​ wall, the EMT best treats this condition​ by:

providing positive pressure ventilation.

EMTS treating a patient of a sexual assault may not only be dealing with medical issues but with _______ as well

psychological

A patient complaining of chest tightness, coughing up blood, and subcutaneous emphysema following an explosion may be suffering from a:

pulmonary blast injury

A man called EMS 12 hours after injuring his chest. Your assessment reveals a flail segment to the right side of the chest. The patient is experiencing respiratory distress and his oxygen saturations is 78%. His breath sounds are equal bilaterally and his jugular veins are normal. You should suspect:

pulmonary contusion

Which of the following blood vessels transports oxygenated blood?

pulmonary veins

During the primary assessment, circulation is evaluated by assessing:

pulse quality, external bleeding, and skin condition

Observations made when forming a general impression of a patient would include all of the following, EXCEPT:

pulse strength

Throughout a radio transmission to the dispatch center, the EMT periodically uses the term "break." You should recognize this to mean he is:

purposely pausing his transmission.

Pain that moves from its point of origin to another body location is said to be:

radiating

When assessing an unresponsive diabetic patient, the primary visible difference between hyperglycemia and hypoglycemia

rate and depth of breathing.

All of the following are considered types of motorcycle impacts EXCEPT:

rear collision

The AED has delivered a shock to an elderly male in cardiac arrest. Following 2 minutes of CPR, you reanalyze the patient's cardiac rhythm and receive a "no shock advised" message. After further resuscitation, you restore a palpable carotid pulse. Your next action should be to:

reassess airway and breathing and treat accordingly

You are transporting a 33-year-old male who was involved in a major motor vehicle crash. You have addressed all immediate and potentially life-threatening conditions and have stabilized his condition with the appropriate treatment. With an estimated time of arrival at the hospital of 20 minutes, you should:

reassess his condition in 5 minutes.

After assisting your patient with his or her nitroglycerin, you should

reassess the blood pressure within 5 minutes to detect hypotension.

A 50-year-old male was stung by a honeybee approximately 15 minutes ago. He presents with respiratory distress, facial swelling, and hypotension. After placing him on oxygen and administering his epinephrine via auto-injector, you note that his breathing has improved. Additionally, his facial swelling is resolving and his blood pressure is stable. Your next action should be to:

record the time and dose of the injection, and transport promptly.

You are ventilating a 40-year-old uninjured man who is apneic but has a pulse. When your partner reassesses his blood pressure, he notes that it has decreased significantly from previous readings. You elevate the patient's legs, but this action has no effect. You should:

reevaluate the rate and volume of your ventilations

A pulse with a consistent pattern is considered to be:

regular

The EMT knows that a function of the skin is​ to:

regulate temperature.

When assessing a patient with a closed soft-tissue injury, it is MOST important to

remain alert for more severe underlying injuries.

When a motor vehicle strikes a tree while traveling at 40 mph, the unrestrained occupant:

remains in motion until acted upon by an external force.

When caring for an occupant inside a motor vehicle equipped with an air bag that did not deploy upon impact, you should:

remember that it could still deploy and seriously injure you

You are ventilating an apneic woman with a bag-mask device. She has dentures, which are tight-fitting. Adequate chest rise is present with each ventilation, and the patient's oxygen saturation reads 96%. When you reassess the patency of her airway, you note that her dentures are now loose, although your ventilations are still producing adequate chest rise. You should:

remove her dentures, resume ventilations, and assess for adequate chest rise.

A 19-year-old female is found unconscious by her roommate. Your primary assessment reveals that her breathing is inadequate. As you insert an oropharyngeal airway, she begins to gag violently. You should:

remove the airway and be prepared to suction her oropharynx.

When using your portable radio, you must push the "press to talk" button and wait one second before speaking. This is essential to effective communication because your EMS system must use:

repeaters.

When interviewing a patient, you can show him or her that you understand the situation by:

repeating statements back to him or her

You have administered one dose of epinephrine to a 40-year-old female to treat an allergic reaction that she developed after being stung by a scorpion. Your reassessment reveals that she is still having difficulty breathing, has a decreasing mental status, and has a blood pressure of 80/50 mm Hg. You should:

request permission from medical control to give another dose of epinephrine.

In contrast to monophasic defibrillation, biphasic defibrillation:

requires a lower energy setting.

An unconscious patient found in a prone position must be placed in a supine position in case he or she:

requires cardiopulmonary resuscitation (CPR)

A patient who presents with profound cyanosis following a chest injury: a. requires prompt ventilation and oxygenation. b. has most likely experienced a ruptured aorta. c. should be placed in Trendelenburg position. d. is most likely experiencing severe blood loss.

requires prompt ventilation and oxygenation.

Prior to applying a nonrebreathing mask on a patient, you must ensure that the:

reservoir bag is fully inflated.

As you are wheeling your patient through the emergency department doors, you receive another call for a major motor vehicle crash. You should: A. place the patient in a high-visibility area and then respond to the call. B. respond only after giving a verbal patient report to a nurse or physician. C. inform the admissions clerk of the situation and then respond at once. D. leave a copy of the run form with a nurse and then respond to the call.

respond only after giving a verbal patient report to a nurse or physician.

The optic nerve endings are located within the:

retina

The systemic veins function by: Select one: A. delivering oxygen-poor blood to the capillaries. B. delivering deoxygenated blood to the capillaries. C. returning deoxygenated blood back to the heart. D. returning oxygen-rich blood back to the left atrium.

returning deoxygenated blood back to the heart.

Deoxygenated blood from the body returns to the

right atrium.

Defibrillator pads are placed on the patient's chest with one pad to the

right of the upper sternum and the other pad just to the left and below the left nipple.

A patient has suffered abrasions to the legs after falling on a moving treadmill. In relation to the function of the​ skin, the EMT should be most concerned​ about:

risk for infection.

You have inserted an oral airway and are ventilating an apneic woman with a bag-mask device. She suddenly begins regurgitating large amounts of vomit. You should:

roll her onto her side and remove the oral airway.

The white portion of the eye is called the

sclera

The stinger from a honeybee should be:

scraped away from the skin.

You are ventilating a patient with a stoma; however, air is escaping from the mouth and nose. To prevent this, you should:

seal the mouth and nose.

Shock due to severe infection is called ________.

septic shock

Which of the following situations or conditions warrants immediate transport?

severe chest pain and cool, pale skin

The amount of kinetic energy that is converted to do work on the body dictates the ____ of the injury.

severity

Which of the following factors will cause a decreased minute volume in an adult?

shallow breathing

Which of the following is NOT a possible cause of airway obstruction?

shallow breathing

when an electrical impulse reaches the AV node, it is slowed for a brief period of time so that :

sinoatrial (SA) node can reset and generate another impulse

The electrical impulse generated by the heart originates in the:

sinoatrial (SA) node.

Jugular venous distention suggests a problem with blood returning to the heart if the patient is:

sitting up at a 45° angle.

Clear fluid from the ears or nose indicate a​ possible:

skull fracture.

You could be sued for ___________ if your radio report to the hospital describes the patient in a manner that injures his or her reputation. A. libel B. assault C. slander D. negligence

slander

The diaphragm functions as an involuntary muscle when a person:

sleeps.

Central chemoreceptors located in the medulla provide feedback to increase the rate and depth of breathing when they sense:

slight elevations in carbon dioxide or a decrease in the pH of the cerebrospinal fluid.

In contrast to the sympathetic nervous system, the parasympathetic nervous system

slows the heart and respiratory rates.

Following blunt trauma to the abdomen, a 21-year-old female complains of diffuse abdominal pain and pain to the left shoulder. Your assessment reveals that her abdomen is distended and tender to palpation. On the basis of these findings, you should be MOST suspicious of injury to the: Select one: A. gallbladder. B. pancreas. C. spleen. D. liver

spleen

An 8-year-old boy fell forward off of a swing onto outstretched arms. He did not hit his head or lose consciousness and is complaining of left forearm pain. There is deformity to the left wrist, but no other injuries or life threats. His vital signs are pulse 132, respirations 20, blood pressure 108/62, and SpO2 100%. The proper care of this patient would be: Select one: a. apply high-flow oxygen, anatomically splint the forearm when the patient is immobilized to a long backboard. b. splint the forearm and apply a cold pack before transporting non-emergently. c. call ALS so that the patient can be given intravenous pain medications before splinting the forearm. d. immediately load and transport, splinting the injury en-route.

splint the forearm and apply a cold pack before transporting non-emergently.

Internal bleeding into a fractured extremity is MOST often controlled by: Select one: A. splinting the extremity. B. applying chemical ice pack. C. applying a tourniquet. D. keeping the patient warm.

splinting the extremity

During the secondary​ assessment, the EMT notes instability and a flail segment to a​ patient's lower left lateral chest. The EMT would​ immediately:

stabilize the segment with his hand.

While eating dinner, your partner suddenly grabs his throat and has a panicked look on his face. He has a weak cough, faint inspiratory stridor, and cyanosis around the lips. You should:

stand behind him and administer abdominal thrusts.

You respond to the residence of a 62-year-old male who is unresponsive. Your primary assessment reveals that he is pulseless and apneic. You should:

start CPR and attach the AED as soon as possible.

A young female patient involved in a motor vehicle collision responds to painful stimuli by extending her arms and legs. Her airway is open and breathing shallow and irregular. A radial pulse that is moderate in strength is palpated. Her pulse is 64 beats per​ minute, respirations are​ 8, and blood pressure is​ 210/110 mmHg with an SpO2​ 90%. She has noted deformity to her forehead and a left pupil that is dilated and does not respond to light. Manual​ in-line stabilization is being held by Emergency Medial Responders​ (EMR). At this​ time, the​ EMT's priority action is​ to:

start positive pressure ventilation at 20 breaths per minute.

An unrestrained​ 32-year-old female motorist hit a utility pole head on at 45 mph. The car sustained extensive damage. She has an open​ airway, breathing poorly at 32 breaths per​ minute, and is responsive to painful stimuli. The radial pulse is rapid and​ weak, and her skin is cool and cyanotic. Breath sounds are decreased on the right side. Emergency Medical Responders are maintaining manual​ in-line stabilization. Your next action would be​ to:

start positive pressure ventilation.

Assessment findings of a driver who was ejected from his vehicle in a​ rollover-type collision include unresponsiveness and bruising to the abdominal and pelvic​ areas, along with an open femur fracture. The patient has an open airway and is breathing at 32 times per minute. His skin is cool and diaphoretic and the radial pulses are weak. Manual​ in-line spinal stabilization is being maintained. The​ EMT's next action would be​ to:

start positive pressure ventilation.

A patient has been involved in a motorcycle crash and has a flail segment to the right lateral chest. His airway is open and he is breathing poorly at 24 breaths per minute with a decreasing SpO2. The EMT shows that he is appropriately caring for this injury when​ he:

starts positive pressure ventilation.

A 67-year-old female with severe chest pain becomes unresponsive, pulseless, and apneic during transport. You should:

stop the ambulance, begin CPR, and attach the AED as soon as possible

The purpose of defibrillation is to

stop the chaotic, disorganized contraction of the cardiac cells.

The purpose of defibrillation is to:

stop the chaotic, disorganized contraction of the cardiac cells.

The​ EMT's primary and first concern with any open injury to the chest​ is:

stopping air entry.

Which of the following sounds indicates swelling of the upper airway?

stridor

A crackling sound produced by air bubbles under the skin is called

subcutaneous emphysema

A collection of blood between the brain and the dura is called​ a/an:

subdural hematoma.

After performing a head tilt-chin lift maneuver to open the airway of an unresponsive patient, you should:

suction as needed and insert an airway adjunct.

You are caring for a 70-year-old female with signs and symptoms of an acute stroke. She is conscious, has secretions in her mouth, is breathing at a normal rate with adequate depth, and has an oxygen saturation of 96%. You should:

suction her oropharynx and transport immediately.

Proper technique for suctioning the oropharynx of an adult patient includes:

suctioning while withdrawing the catheter from the oropharynx.

If one or more occupants in the same vehicle are killed in a crash, the EMT should:

suspect that all living occupants experienced the same serious trauma.

Which of the following would the EMT most likely not perform on a responsive patient with a headache and no apparent life-threatening conditions?

systemic head-to-toe examination

The severity of bleeding should be based on all of the following findings, EXCEPT: Select one: A. the mechanism of injury. B. systolic blood pressure. C. clinical signs and symptoms. D. poor general appearance.

systolic blood pressure

When you use the palpation method to obtain a blood pressure, the measurement you obtain is the:

systolic blood pressure

The pressure exerted against the walls of the artery when the left ventricle contracts is called the:

systolic pressure

Signs and symptoms of a sympathomimetic drug overdose include:

tachycardia.

Patients with chest injuries will often present with _______?

tachypnea

Patients suffering from an open would to the neck may experience all of the following EXCEPT:

tension pneumothorax

Assessment findings on a​ 33-year-old female who has been shot in the right side of the chest indicate that she is suffering from a pneumothorax. As you care for this​ patient, your primary concern is monitoring the injury​ for:

tension pneumothorax.

If a patient has an open chest wound that allowed air to accumulate into the pleural cavity to the point at which the lung is totally collapsed and is starting to shift to the other​ side, this additional complication is known​ as:

tension pneumothorax.

When the myocardium requires more oxygen

the arteries supplying the heart dilate.

When the myocardium requires more oxygen:

the arteries supplying the heart dilate.

You are called for a​ 2-year-old boy who fell and cut his arm. While en route to the​ call, the dispatcher informs you that the patient has hemophilia​ (clotting disorder). As an​ EMT, you should recognize​ that:

the bleeding may be significant.

Hypovolemic shock occurs when: Select one: A. the patient's systolic blood pressure is less than 100 mm Hg. B. the body cannot compensate for rapid blood loss. C. at least 10% of the patient's blood volume is lost. D. the clotting ability of the blood is enhanced.

the body cannot compensate for rapid blood loss.

As time progresses following a significant injury:

the body's ability to compensate for shock decreases.

When treating a patient with chest pain, you should assume that he or she is having an AMI because:

the cause of the pain cannot be diagnosed in the field.

Large amounts of adenosine triphosphate (ATP) are generated when:

the cells function with adequate oxygen

You are caring for a 56-year-old male patient complaining of abdominal pain. Your service has recently switched to an electronic PCR system (ePCR). When completing the ePCR, it is important to be aware that: A. the ePCR is relatively unsecured and should not contain patient-specific information. B. a written patient care record will need to be completed on arrival at the hospital. C. the ePCR allows patient information to be transmitted directly to the receiving hospital's computers. D. the ePCR does not contain the same level of information as the written version and your verbal report should be expanded.

the ePCR allows patient information to be transmitted directly to the receiving hospital's computers.

The eyeball itself is referred to as the:

the globe

Airbags decrease injury to all of the following EXCEPT:

the heart

The danger of blood collecting inside the pericardial sac after blunt chest wall trauma is​ that:

the heart cannot contract as effectively.

When using the Glasgow Coma Score​ (GCS), the EMT understands​ that:

the higher score the better.

A dissecting aortic aneurysm occurs when:

the inner layers of the aorta become separated.

Ventricular tachycardia causes hypotension because:

the left ventricle does not adequately fill with blood

Patients with full-thickness (third-degree) bums generally do not complain of pain because:

the nerve endings have been destroyed

Prompt transport of a patient with a suspected AMI is important because:

the patient may be eligible to receive thrombolytic therapy.

Reassessment is performed to determine all of the following, EXCEPT:

the reason why the patient called EMS

What is the MOST common cause of airway obstruction in an unconscious patient?

the tongue

Whether you are using a commercial device or a stick and triangular bandage as a tourniquet, it is important to remember that: Select one: A. bulky dressings should be securely applied over the tourniquet to further assist in controlling the bleeding. B. you should try to control the bleeding by applying pressure to a proximal arterial pressure point first. C. the tourniquet should only be removed at the hospital because bleeding may return if the tourniquet is released. D. the tourniquet should be applied directly over a joint if possible because this provides better bleeding control.

the tourniquet should only be removed at the hospital because bleeding may return if the tourniquet is released.

The main advantage of the Venturi mask is:

the use of its fine adjustment capabilities in the long-term management of physiologically stable patients.

Children are often "belly breathers" because _________?

their intercostal muscles are not developed

A low ETCO2 reading, as measured by capnography, would MOST likely be observed if:

there is an absence or decrease in the level of CO2 in the lungs

Cardiac output may decrease if the heart beats too rapidly because

there is not enough time in between contractions for the heart to refill completely.

You are dispatched to a local nursery for a 39-year-old female who is sick. When you arrive, you find the patient lying on the floor. She is semiconscious, has copious amounts of saliva coming from her mouth, and is incontinent of urine. You quickly feel her pulse and note that it is very slow. Immediate management for this patient should include:

thoroughly suctioning her oropharynx.

A fall from more than _____ times the patient's height is considered to be significant.

three

Most patients are instructed by their physician to take up to _______ doses of nitroglycerin before calling EMS

three

When a patient's respirations are shallow:

tidal volume is markedly reduced

Your patient was replacing a roof on a hot summer day when he slipped and fell from the second story. Upon your​ arrival, you find that the patient is​ unconscious, fully​ immobilized, on O2 via NRB at 15 lpm with oxygen saturation of 98​ percent, and warm to the touch. The next intervention that you should consider​ is:

to passively cool the patient.

The descending aorta divides into the two iliac arteries at the level of the

umbilicus.

During an EMS call, you should take standard precautions:

upon exiting the ambulance, but before actual patient contact.

Which of the following negative effects of anaphylaxis will be the MOST rapidly fatal if not treated immediately?

upper airway swelling

Pelvic inflammatory disease (PID) typically does NOT affect the:

urinary bladder

If the victim of a toxicologic emergency vomits, an EMT should _________.

use appropriate personal protective equipment and examine the vomitus for pill fragments or other clues for patient care.

A patient has suffered a small but jagged laceration to her left hand. When cleaning the​ wound, it is important that the​ EMT:

use sterile gauze and wipe away from the site of injury.

You are providing care to a 61-year-old female complaining of chest pain that is cardiac in origin. Your service utilizes a multiplex communication system. You wish to transmit the patient's electrocardiogram to the hospital. In order to accomplish this, you must: A. wait until you reach the hospital to transmit the information. B. send the electrocardiogram from the back of the ambulance. C. use the multiplex system to transmit the information . D. be aware that only audio or data transmissions can be sent at any one time.

use the multiplex system to transmit the information

When documenting a patient's description of his or her chest pain or discomfort, the EMT should:

use the patient's own words.

Which of the following physiologic actions does epinephrine produce when given for an allergic reaction?

vasoconstriction and bronchodilation

A​ 24-year-old male has cut his arm with a table saw. Assessment reveals dark red blood flowing steadily from the laceration. As an​ EMT, you would identify this bleeding​ as:

venous.

You receive a call to a residence where a man found his wife unresponsive on the couch. The patient's respiratory rate is 8 breaths/min, her breathing is shallow, her heart rate is 40 beats/min, and her pulse is weak. The husband hands you an empty bottle of hydrocodone (Vicodin), which was refilled the day before. You should:

ventilate her with a BVM.

The physical act of moving air into and out of the lungs is called:

ventilation.

While evaluating a patient with chest pain, your partner tells you that the patient's blood pressure is 140/94 mm Hg. The lower number represents the pressure from the

ventricles relaxing

Sudden death following AMI is MOST often caused by:

ventricular fibrillation.

Which of the following cardiac arrhythmias has the greatest chance of deteriorating into a pulseless rhythm?

ventricular tachycardia

Effective therapeutic communication skills require _________. A. English-speaking patients or family members B. correct use of complex medical terminology C. supervision by advanced life support personnel D. verbal and nonverbal communication techniques

verbal and nonverbal communication techniques

Pleural fluid is contained between the: a. visceral pleura and the lung. b. parietal pleura and the chest wall. c. visceral and parietal pleurae. d. parietal pleura and the heart.

visceral and parietal pleurae.

In older patients, the first indicator of nontraumatic internal bleeding may be: Select one: A. weakness or dizziness. B. a heart rate over 120 beats/min. C. a low blood pressure. D. diaphoresis and pale skin.

weakness or dizziness

The two MOST common signs of anaphylaxis are:

wheezing and widespread urticaria.

The nasal cannula is MOST appropriately used in the prehospital setting:

when the patient cannot tolerate a nonrebreathing mask

When is it MOST appropriate to consider requesting additional ambulances at an accident scene?

when you determine there are multiple patients

Nitroglycerin is contraindicated in patients

who have experienced a head injury.

Distributive shock occurs when:

widespread dilation of the blood vessels causes blood to pool in the vascular beds.

Supplemental oxygen via nonrebreathing mask should be administered to patients:

with difficulty breathing and adequate tidal volume

A nasopharyngeal airway is inserted:

with the bevel facing the septum if inserted into the right nare

A patient with spontaneous respirations is breathing:

without assistance

Force acting over a distance

work

A patient has had part of his right thumb amputated in an industrial accident. Coworkers have retrieved the thumb and wrapped it in a towel. The EMT demonstrates appropriate handling of the amputated part when​ he:

wraps it in a dry sterile dressing.

You have been called for a 23-year-old male who is completely deaf. To best communicate with this patient you should:

write questions on a note pad.

Palpating the carotid pulse is not recommended in infants because:

you may inadvertently compress the trachea.

The EMT shows she understands​ "Do Not​ Resuscitate" (DNR) orders and living wills when she​ states: A. ​"If a patient has a valid​ DNR, the EMT should withhold all care and transport the patient to the hospital. If a living will is​ present, the EMT should do exactly what it​ says." B. ​"A living will is a form of​ DNR; only a living will states exactly how the patient wants to be​ resuscitated." C. ​"If the patient has a living​ will, it can legally be used as a substitute for a DNR order by the EMT as long as it is properly dated and​ signed." D. ​"A DNR applies to resuscitation issues​ only, such as when a patient stops breathing or the heart ceases to​ beat, whereas a living will pertains to​ long-term life support care and​ equipment."

​"A DNR applies to resuscitation issues​ only, such as when a patient stops breathing or the heart ceases to​ beat, whereas a living will pertains to​ long-term life support care and​ equipment."

Which statement shows that the EMT understands the use of a tourniquet in controlling​ bleeding?

​"A tourniquet should be applied tightly enough so that arterial blood flow distal to the tourniquet is completely​ stopped."

The EMT is properly assessing for sensory function in the hands when he tells or asks the​ patient:

​"Can you tell me what finger I am​ touching?"

Which statement made by an EMT demonstrates an understanding of chest​ trauma?

​"Closed chest injuries are caused by blunt trauma and can be just as serious as open chest​ injuries."

A​ 17-year-old female was thrown from a horse and is now confused. Assessment findings include an open​ airway, adequate​ breathing, and a strong radial pulse. Her vital signs are normal. Which question would be the MOST important to immediately ask the​ patient's mother, who was riding with​ her?

​"Did she lose​ consciousness?"

Which​ statement, made by the EMT shows an understanding of dressing and bandaging a​ wound?

​"I always leave the tips of the fingers or toes exposed when bandaging the arm or leg so future assessment of circulation can be​ made."

A male patient in his thirties fell 10 feet off of a loading​ dock, landing on his head. He has deformity and depression to the back of his head and is unresponsive with snoring respirations. Vital signs are pulse​ 132, respirations are​ agonal, blood pressure​ 136/64, and SpO2​ 89%. He responds to painful stimuli with purposeful activity. What instruction would you provide other caregivers at the​ scene?

​"I need someone to start positive pressure ventilation at 12 breaths a​ minute."

Which statement describes the BEST way to determine the severity of a​ patient's blood​ loss?

​"I use the​ patient's signs and symptoms to judge the severity of the blood​ loss."

On scene at a college football​ game, a wide receiver is unresponsive after colliding with another player and forcefully taking a helmet to the chest. The athletic trainer reports that the patient is pulseless and apneic. Given the mechanism of injury and​ patient's presentation, which instruction would be​ appropriate?

​"Let's apply the AED and follow all​ prompts."

A​ 28-year-old male was cutting limbs from a tree when he lost his footing and fell 20 feet. He is unresponsive and breathing extremely shallow at 28 times per minute. His radial pulse is weak and thready and his skin is cool to the touch. Emergency Medical Responders​ (EMRs) have placed him on a nonrebreather face mask and are holding manual​ in-line stabilization. A blanket has also been applied and covers the man. Based on these assessment​ findings, what instruction would you give to the​ EMRs?

​"Let's take off the oxygen mask and start positive pressure ventilation to assist his​ breathing."

While riding a​ bike, a young boy fell onto a stick that was protruding from the ground. The stick impaled him in the neck. The boy immediately pulled the stick out and ran​ home, where his mother called 911. Assessment reveals a gaping wound to the right neck. All hemorrhaging has clotted​ off, and manual​ in-line spinal stabilization is being maintained. Which instruction should you provide to the other EMTs on​ scene?

​"Place an occlusive dressing over the wound and tape it on all four​ sides."

Which statement indicates an understanding of the​ EMT's prehospital role in caring for the patient in​ shock?

​"Since shock is best treated in the​ hospital, the EMT should provide care to maintain perfusion to the vital organs and​ transport."

At an industrial​ complex, a young male was struck in the head by a large piece of steel thrown from a grinding machine. Assessment reveals a​ soft, painful depression underlying a tear in his​ scalp, which is covered with dried blood. There is also clear fluid coming from his right ear. He is conscious but confused and exhibits the following vital​ signs: pulse​ 84, respirations​ 16, blood pressure​ 142/76, and SpO2​ 92%. Emergency Medical Responders are on scene and are maintaining​ in-line manual stabilization of the cervical spine. What instruction would indicate the next step needed in this​ patient's care?

​"Will someone administer supplemental oxygen to the​ patient?

Which statement demonstrates an understanding of shock caused by internal​ bleeding?

​"You should always suspect internal bleeding in a patient with the signs and symptoms of shock but no external​ bleeding."

Which of the following describes the layers of the​ skin?

​Subcutaneous, epidermis, dermis

The EMT recognizes the parameters of the Glasgow Coma Score​ (GCS) as:

​eyes, motor, and verbal response.


Kaugnay na mga set ng pag-aaral

GOV 357M Constitutional Law Final exam

View Set

Exam 3 Chapter 12 Transportation (True/False)

View Set

Solving Trigonometric Equations Quiz

View Set

Chapter 3 - External Analysis: Industry Structure

View Set

Physics Chapter 3 Force and Motion & Chapter 4 Work and Energy

View Set